SlideShare a Scribd company logo
1 of 92
ĐẠI HỌC QUỐC GIA HÀ NỘI
TRƯỜNG ĐẠI HỌC KHOA HỌC TỰ NHIÊN
———–
NGUYỄN THỊ MINH THƯƠNG
LÝ THUYẾT ĐỒ THỊ
VỚI CÁC BÀI TOÁN PHỔ THÔNG
LUẬN VĂN THẠC SĨ KHOA HỌC
HÀ NỘI - 2015
ĐẠI HỌC QUỐC GIA HÀ NỘI
TRƯỜNG ĐẠI HỌC KHOA HỌC TỰ NHIÊN
———–
NGUYỄN THỊ MINH THƯƠNG
LÝ THUYẾT ĐỒ THỊ
VỚI CÁC BÀI TOÁN PHỔ THÔNG
Chuyên ngành: Phương pháp toán sơ cấp
Mã số: 60.46.01.13
LUẬN VĂN THẠC SĨ KHOA HỌC
Người hướng dẫn khoa học:
GS.TS Đặng Huy Ruận
HÀ NỘI - 2015
Mục lục
Lời nói đầu 3
1 Đại cương về đồ thị 4
1.1 Định nghĩa đồ thị . . . . . . . . . . . . . . . . . . . . . . 4
1.2 Một số dạng đồ thị đặc biệt . . . . . . . . . . . . . . . . 6
1.3 Bậc của đỉnh đồ thị . . . . . . . . . . . . . . . . . . . . . 8
1.3.1 Bậc của đỉnh . . . . . . . . . . . . . . . . . . . . 8
1.3.2 Nửa bậc . . . . . . . . . . . . . . . . . . . . . . . 8
1.3.3 Một số tính chất . . . . . . . . . . . . . . . . . . 9
1.4 Xích, chu trình, đường, vòng . . . . . . . . . . . . . . . . 13
1.4.1 Xích, chu trình . . . . . . . . . . . . . . . . . . . 13
1.4.2 Đường, vòng . . . . . . . . . . . . . . . . . . . . . 14
1.4.3 Một số tính chất . . . . . . . . . . . . . . . . . . 15
1.5 Đồ thị liên thông . . . . . . . . . . . . . . . . . . . . . . 16
1.5.1 Định nghĩa . . . . . . . . . . . . . . . . . . . . . 16
1.5.2 Tính chất . . . . . . . . . . . . . . . . . . . . . . 17
1.6 Số ổn định trong, số ổn định ngoài . . . . . . . . . . . . 18
1.6.1 Số ổn định trong . . . . . . . . . . . . . . . . . . 18
1.6.2 Số ổn định ngoài . . . . . . . . . . . . . . . . . . 19
1.6.3 Các thuật toán tìm số ổn định trong, số ổn định
ngoài. . . . . . . . . . . . . . . . . . . . . . . . . 20
1.7 Nhân của đồ thị và ứng dụng vào trò chơi . . . . . . . . 21
1.7.1 Định nghĩa . . . . . . . . . . . . . . . . . . . . . 21
1.7.2 Tính chất . . . . . . . . . . . . . . . . . . . . . . 22
1.7.3 Trò chơi Nim . . . . . . . . . . . . . . . . . . . . 23
1.7.4 Trò chơi bốc các vật . . . . . . . . . . . . . . . . 24
1.8 Cây và bụi . . . . . . . . . . . . . . . . . . . . . . . . . . 29
1.8.1 Định nghĩa . . . . . . . . . . . . . . . . . . . . . 29
1.8.2 Đặc điểm của cây và bụi . . . . . . . . . . . . . . 30
1
2 Một số bài toán đồ thị cơ bản 33
2.1 Bài toán về đường đi . . . . . . . . . . . . . . . . . . . . 33
2.1.1 Đường đi Euler - Chu trình Euler. . . . . . . . . . 33
2.1.2 Đường đi Hamilton - Chu trình Hamilton. . . . . 40
2.2 Bài toán tô màu đồ thị . . . . . . . . . . . . . . . . . . . 43
2.2.1 Định nghĩa . . . . . . . . . . . . . . . . . . . . . 43
2.2.2 Một số tính chất . . . . . . . . . . . . . . . . . . 43
2.2.3 Thuật toán tô màu đỉnh. . . . . . . . . . . . . . . 53
3 Ứng dụng lý thuyết đồ thị vào giải toán phổ thông. 54
3.1 Quy trình giải bài toán bằng phương pháp đồ thị. . . . . 54
3.1.1 Xây dựng đồ thị G mô tả các quan hệ. . . . . . . 54
3.1.2 Dựa vào các kết quả của lý thuyết đồ thị hoặc lý
luận trực tiếp suy ra đáp án của bài toán D. . . . 54
3.2 Bài toán về đỉnh - cạnh của đồ thị. . . . . . . . . . . . . 55
3.3 Bài toán về xích, chu trình, đường, vòng và tính liên thông
của đồ thị. . . . . . . . . . . . . . . . . . . . . . . . . . 58
3.4 Bài toán về tô màu đồ thị. . . . . . . . . . . . . . . . . 63
3.5 Bài toán liên quan đến số ổn định trong, số ổn định ngoài. 74
3.6 Bài toán liên quan đến đường đi. . . . . . . . . . . . . . 76
3.6.1 Bài toán tìm đường đi trong mê cung . . . . . . . 76
3.6.2 Bài toán liên quan đến đường và chu trình Euler . 80
3.6.3 Bài toán liên quan đến đường và chu trình Hamilton 82
3.7 Bài toán liên quan đến cây. . . . . . . . . . . . . . . . . 84
Kết luận 89
Tài liệu tham khảo 90
2
LỜI NÓI ĐẦU
Lý thuyết đồ thị là một trong những ngành khoa học ra đời khá sớm.
Lý thuyết đồ thị giúp mô tả hình học và giải quyết nhiều bài toán thực
tế phức tạp.
Khái niệm lý thuyết đồ thị được nhiều nhà khoa học độc lập nghiên
cứu và có nhiều đóng góp trong lĩnh vực toán học ứng dụng.
Năm 2001, Bộ Giáo Dục và Đào Tạo có quy định các chuyên đề bồi
dưỡng học sinh giỏi thống nhất trên toàn quốc, trong đó có chuyên đề
lý thuyết đồ thị. Như vậy, việc học chuyên đề Lý Thuyết Đồ Thị đối với
học sinh khá và giỏi đang là nhu cầu thực tế trong dạy học toán ở phổ
thông. Tuy nhiên, việc dạy học chuyên đề này còn tồn tại một số khó
khăn vì những lý do khác nhau. Một trong các lý do đó là sự mới mẻ,
độc đáo và khó của chủ đề kiến thức này.
Luận văn "Lý thuyết đồ thị với các bài toán phổ thông" đưa đến sự
sáng tạo trong cách nhìn nhận bài toán và lập luận cách giải dưới con
mắt của lý thuyết đồ thị.
Ngoài phần mở đầu và kết luận luận văn gồm 3 chương:
Chương 1 Đại cương về đồ thị.
Chương 2 Một số bài toán đồ thị cơ bản.
Chương 3 Ứng dụng lý thuyết đồ thị vào giải toán phổ thông.
Luận văn được hoàn thành dưới sự hướng dẫn, giúp đỡ tận tình của
GS.TS Đặng Huy Ruận, tác giả xin bày tỏ sự kính trọng và lòng biết ơn
sâu sắc tới thầy.
Tác giả cũng xin gửi lời cảm ơn chân thành đến Ban giám hiệu cùng các
thầy cô giáo khoa Toán - Cơ - Tin, Trường Đại học Khoa Học Tự Nhiên
- Đại Học Quốc Gia Hà Nội đã tạo điều kiện, dạy bảo và dìu dắt tác giả
trong những năm học vừa qua.
Xin chân thành cảm ơn sự giúp đỡ của bạn bè, người thân trong thời
gian học tập và làm luận văn.
Do khả năng nhận thức của bản thân tác giả, luận văn còn nhiều hạn
chế, thiếu sót. Tác giả kính mong các ý kiến chỉ bảo của quý thầy cô
cùng sự đóng góp của các bạn đọc.
Tác giả xin chân thành cảm ơn!
Hà Nội, tháng 6 năm 2015
3
Chương 1
Đại cương về đồ thị
1.1 Định nghĩa đồ thị
Tập hợp X = ∅ các đối tượng và bộ E các cặp sắp thứ tự và không
sắp thứ tự các phần tử của X được gọi là một đồ thị, đồng thời được ký
hiệu bằng G(X, E) (hoặc G = (X, E) hoặc G(X)).
Hình 1.1: Ví dụ về mô hình đồ thị
Các phần tử của X được gọi là các đỉnh. Cặp đỉnh không sắp thứ tự
được gọi là cạnh, cặp đỉnh sắp thứ tự được gọi là cạnh có hướng hay
cung.
Đồ thị chỉ chứa các cạnh được gọi là đồ thị vô hướng, còn đồ thị chỉ
chứa các cung được gọi là đồ thị có hướng. Nếu đồ thị chứa cả cạnh lẫn
cung thì nó được họi là đồ thị hỗn hợp hay đồ thị hỗn tạp.
Một cặp đỉnh có thể được nối với nhau bằng hai hoặc nhiều hơn hai
cạnh (hai hoặc nhiều hơn hai cung cùng một hướng). Các cạnh (cung)
này được gọi là các cạnh (cung) bội.
Một cung (hay một cạnh) có thể bắt đầu và kết thúc tại cùng một
đỉnh. Cung (cạnh) loại này được gọi là khuyên hay nút.
Cặp đỉnh x,y được nối với nhau bằng cạnh (cung) a và a được gọi là
cạnh (cung) thuộc đỉnh x, đỉnh y.
4
Nếu cung b xuất phát từ đỉnh u và đi vào đỉnh v thì u được gọi là
đỉnh đầu, v được gọi là đỉnh cuối của cung b.
Cặp đỉnh x, y được gọi là hai đỉnh kề nhau nếu x = y và là hai đầu
của cùng một cạnh hay một cung.
Đối với mọi đỉnh x dùng D(x) để chỉ tập đỉnh, mà mỗi đỉnh này được
nối với x bằng ít nhất một cạnh; D+
(x) để chỉ tập đỉnh mà mỗi đỉnh
này từ x có cung đi tới; D−
(x) để chỉ tập đỉnh mà mỗi đỉnh này có cung
đi tới x.
Hai cạnh (cung) a,b được gọi là kề nhau, nếu:
i) Chúng khác nhau.
ii) Chúng có đỉnh chung (nếu a, b là cung, thì không phụ thuộc vào
đỉnh chung đó là đỉnh đầu hay đỉnh cuối của cung a, đỉnh đầu hay đỉnh
cuối của cung b).
Ví dụ 1.1. Cho đồ thị hỗn hợp có khuyên G(X, E) với tập đỉnh
X = {x1, x2, x3, x4, x5, x6, x7},
tập cạnh và cung
E = {x1, x2; x2, x3; x4, x6; x5, x6; x3, x3; x1, x6; x5, x5}
= {a1 a2 a3 a4 a5 b1 b2},
trong đó a1, a2, a3, a4, a5 là các cạnh; b1, b2 là các cung.
Hình 1.2
5
1.2 Một số dạng đồ thị đặc biệt
Trong những trường hợp không cần phân biệt giữa cạnh và cung ta
quy ước dùng cạnh thay cho cả cung.
Đồ thị G = (X, E) không có khuyên và mỗi cặp đỉnh được nối với
nhau bằng không quá một cạnh, được gọi là đồ thị đơn hay đơn đồ thị
và thông thường được gọi là đồ thị.
Đồ thị G = (X, E) không có khuyên và có ít nhất một cặp đỉnh được
nối với nhau bằng từ hai cạnh trở lên được gọi là đa đồ thị.
Đồ thị G = (X, E) được gọi là vô hướng nếu các cạnh trong E là
không định hướng.
Đồ thị G = (X, E) được gọi là có hướng nếu các cạnh trong E là có
định hướng.
Hình 1.3
Đồ thị vô hướng (có hướng) G = (X, E) được gọi là đồ thị đầy đủ
nếu mỗi cặp đỉnh được nối với nhau bằng đúng một cạnh (một cung với
chiều tùy ý).
Hình 1.4: Đồ thị đầy đủ
Đa đồ thị vô hướng (có hướng) G = (X, E) được gọi là đồ thị k-đầy
đủ nếu mỗi cặp đỉnh được nối với nhau bằng đúng k cạnh (k cung với
6
chiều tùy ý).
Đồ thị (đa đồ thị) G = (X, E) được gọi là đồ thị (đa đồ thị) hai mảng
nếu tập đỉnh X của nó được phân thành hai tập con rời nhau X1, X2
(X1 X2 = X và X1 X2 = ∅) và mỗi cạnh đều có một đầu thuộc
X1 còn đầu kia thuộc X2.Khi đó G = (X, E) còn được ký hiệu bằng
G = (X1, X2, E).
Hình 1.5: Đồ thị hai mảng
Đồ thị (đa đồ thị) G = (X, E) được gọi là đồ thị (đa đồ thị) phẳng,
nếu nó có ít nhất một dạng biểu diễn hình học trải trên một mặt phẳng
nào đó, mà các cạnh của đồ thị chỉ cắt nhau ở đỉnh.
Đồ thị (đa đồ thị) G = (X, E) được gọi là hữu hạn, nếu số đỉnh của
nó hữu hạn, tức tập X có lực lượng hữu hạn.
Đồ thị (đa đồ thị) G = (X, E) được gọi là vô hạn, nếu số đỉnh của
nó là vô hạn.
Đồ thị (đa đồ thị) với số cạnh thuộc mỗi đỉnh đều hữu hạn được gọi
là đồ thị (đa đồ thị) hữu hạn địa phương.
Một đồ thị hay đa đồ thị hữu hạn thì nó cũng hữu hạn địa phương.
Cho Y ⊆ X, Y = ∅; H ⊆ E, F = E ∩ (Y × Y ) và V = (X × X)/E.
Đồ thị G1(Y, F) được gọi là đồ thị con, còn G2(X, H) là đồ thị bộ
phận của đồ thị G(X, E).
Đồ thị G (X, V ) được gọi là đồ thị bù của đồ thị G(X, E).
Đồ thị có hướng G(X, E) được gọi là đồ thị đối xứng nếu
∀x, y ∈ X, (x, y) ∈ E ⇒ (y, x) ∈ E
Trong đồ thị đối xứng tùy ý, hai đỉnh kề nhau luôn luôn được nối
bằng hai cung ngược chiều nhau. Để đơn giản, trong trường hợp này
người ta quy ước thay hai cung nói trên bằng một cạnh nối giữa x và y.
Đồ thị có hướng G(X, E) được gọi là đồ thị phản đối xứng nếu
∀x, y ∈ X, (x, y) ∈ E ⇒ (y, x) /∈ E
7
1.3 Bậc của đỉnh đồ thị
1.3.1 Bậc của đỉnh
Giả sử G = (X, E) là một đồ thị hay đa đồ thị có hướng hoặc không
có hướng. Số cạnh và cung thuộc đỉnh x được gọi là bậc của đỉnh x và
ký hiệu bằng m(x).
Đỉnh có bậc bằng 0 được gọi là đỉnh biệt lập.
Đỉnh có bậc bằng 1 được gọi là đỉnh treo.
Cạnh (cung) có ít nhất một đầu là đỉnh treo được gọi là cạnh (cung)
treo.
Hình 1.6
Ví dụ 1.2. Trong hình 1.6 ta có:
m(1) = 2, m(2) = 2, m(3) = 3, m(4) = 3, m(5) = 3, m(6) = 1, m(7) = 0
Đỉnh 6 là đỉnh treo, đỉnh 7 là đỉnh cô lập, g là cạnh treo.
1.3.2 Nửa bậc
Giả sử G = (X, E) là một đồ thị hay đa đồ thị có hướng. Số cung đi
vào đỉnh x được gọi là nửa bậc vào của đỉnh x và ký hiệu bằng m (x)
hoặc m−
(x). Số cung đi ra khỏi đỉnh x được gọi là nửa bậc ra của đỉnh
x và ký hiệu bằng m (x) hoặc m+
(x).
Ký hiệu tập cung đi vào đỉnh x bằng E−
(x), còn tập cung ra khỏi
đỉnh x bằng E+
(x).
8
Hình 1.7
Ví dụ 1.3. Trong hình 1.7 ta có:
m (1) = 1, m (2) = 2, m (3) = 2, m (4) = 0, m (5) = 1, m (6) = 1;
m (1) = 1, m (2) = 1, m (3) = 1, m (4) = 1, m (5) = 1, m (6) = 2;
E−
(4) = {∅}, E+
(4) = {g};
E−
(6) = {f}, E+
(6) = {e, d}.
1.3.3 Một số tính chất
Định lí 1.3.1. Trong đồ thị hay đa đồ thị tùy ý, tổng số bậc của tất cả
các đỉnh bao giờ cũng gấp đôi số cạnh.
Chứng minh.
Thật vậy, khi tính bậc của các đỉnh mỗi cạnh vô hướng hặc có hướng
đều được tính mỗi đầu đúng một lần, do đó tổng số bậc của tất cả các
đỉnh bao giờ cũng gấp đôi số cạnh.
Định lí 1.3.2. Trong đồ thị hay đa đồ thị tùy ý, số đỉnh bậc lẻ luôn luôn
là số chẵn.
Chứng minh.
Giả sử đồ thị (đa đồ thị) G = (X, E) có n đỉnh, m cạnh
X = {x1, x2, ..., xk, xk+1, ..., xn−1, xn},
Các đỉnh x1, x2, ..., xk bậc lẻ và xk+1, ..., xn−1, xn bậc chẵn.
9
Theo định lý 1.1 có đẳng thức:
m(x1) + m(x2) + ... + m(xk)
A
+ m(xk+1) + ... + m(xn−1) + m(xn)
B
= 2m
Vì B là tổng của các số chẵn nên B là số chẵn.
Do đó, A = 2m − B phải là số chẵn.
Số chẵn A là tổng của k số lẻ, nên k phải chẵn.
Bởi vậy, số đỉnh bậc lẻ trong đồ thị (đa đồ thị) bất kỳ phải là một số
chẵn.
Định lí 1.3.3. Trong một đồ thị với n đỉnh (n ≥ 2) có ít nhất hai đỉnh
cùng bậc.
Chứng minh.
Giả sử G = (X, E) là đồ thị tùy ý với |X| = n ≥ 2. Xét hai khả năng
sau:
1) Nếu đồ thị có đỉnh bậc 0 thì trong đồ thị không có đỉnh nào kề với
đỉnh này, nên mỗi đỉnh của đồ thị có bậc là một trong n − 1 số nguyên:
0, 1, 2, ..., n − 3, n − 2.
2) Nếu đồ thị có đỉnh bậc n − 1 thì đồ thị không có đỉnh bậc 0.
Bởi vậy, bậc của mỗi đỉnh thuộc đồ thị là một trong n − 1 số nguyên:
1, 2, ..., n − 2, n − 1.
Từ kết quả trên ta nhận thấy, đồ thị G = (X, E) với n đỉnh (n ≥ 2),
nhưng chỉ có không quá n − 1 loại bậc. Do đó, phải có ít nhất hai đỉnh
cùng bậc.
Khẳng định được chứng minh.
Định lí 1.3.4. Nếu đồ thị với n đỉnh (n ≥ 2) có đúng hai đỉnh cùng bậc,
thì hai đỉnh này không thể đồng thời có bậc 0 hoặc bậc n − 1.
Chứng minh.
Giả sử x,y là hai đỉnh cùng bậc của đồ thị G = (X, E) và đều có bậc
0 hoặc bậc n − 1. Loại x, y và tất cả các cạnh thuộc chúng khỏi đồ thị
G, ta được đồ thị con G1 có n − 2 đỉnh. Theo định lý 1.3 trong G1 có
hai đỉnh cùng bậc, chẳng hạn u,v.
1) Nếu x, y cùng bậc 0, thì u,v trong G không kề với x,y nên u,v trong
G đồng thời là hai đỉnh cùng bậc. Như vậy, đồ thị G phải có ít nhất hai
cặp đỉnh cùng bậc.
10
2) Nếu x, y đều bậc n − 1. Khi đó, mỗi đỉnh u, v đều kề đồng thời với
x, y nên trong đồ thị G các đỉnh u, v cũng cùng bậc. Như vậy, đồ thị G
phải có ít nhất hai cặp đỉnh cùng bậc.
Cả hai trường hợp có thể đều dẫn tới mâu thuẫn với tính chất: Đồ
thị G có duy nhất một cặp đỉnh cùng bậc, nên x, y không thể cùng bậc
0 hặc cùng bậc n − 1 .
Khẳng định được chứng minh.
Định lí 1.3.5. Số đỉnh bậc n − 1 trong đồ thị G với n đỉnh (n ≥ 4), mà
bốn đỉnh tùy ý có ít nhất một đỉnh kề với ba đỉnh còn lại, không nhỏ hơn
n − 3.
Chứng minh.
1) Nếu G là đồ thị đầy đủ, thì khẳng định là hiển nhiên.
2) Nếu G có cặp đỉnh duy nhất không kề nhau. Khi đó, trong G có
n − 2 đỉnh bậc n − 1
3) Nếu G có hai cặp đỉnh không kề nhau, thì chúng phải có đỉnh
chung.
Thật vậy, giải sử A, B; I, D là hai cặp đỉnh không kề nhau. Nếu hai
cặp đỉnh này không có đỉnh chung, thì trong 4 đỉnh A, B, I, D không có
đỉnh nào kề với ba đỉnh còn lại.
Như vậy, mâu thuẫn với giả thiết, nên hai cặp đỉnh A, B; I, D phải
có hai đỉnh trùng nhau, chẳng hạn B ≡ I.
Lấy đỉnh C tùy ý khác với A, B, D. Trong bộ bốn A, B, C, D đỉnh C
là đỉnh kề với cả ba đỉnh A, B, D.
Loại D ra khỏi bộ bốn và thay vào đó là đỉnh E tùy ý khác với A, B,
C, D. Trong bộ bốn A, B, C, E hoặc C hoặc E phải kề với ba đỉnh còn
lại. Nếu E kề với ba đỉnh còn lại, thì E cũng kề với C. Do đó C kề với
tất cả ba đỉnh A, B, E.
Do E là đỉnh tùy ý trong n − 4 đỉnh còn lại (khác với A, B, C) nên
C có bậc n − 1
C là đỉnh tùy ý trong n − 3 đỉnh bậc n − 1
Khẳng định được chứng minh.
Định lí 1.3.6. Với mọi số tự nhiên n (n > 2) luôn luôn tồn tại đồ thị n
đỉnh, mà ba đỉnh tùy ý của đồ thị đều không cùng bậc.
Chứng minh.
1) Với n = 3 đồ thị G3 gồm một đỉnh bậc 0 và hai đỉnh bậc 1.
11
2) Giả sử khẳng định đúng với đồ thị Gn có n đỉnh. Đồ thị Gn+1 có
n + 1 đỉnh được xây dựng như sau:
a. Nếu Gn có đỉnh bậc n − 1, thì không có đỉnh bậc 0, nên ta ghép
vào Gn đỉnh x bậc 0 và được đồ thị Gn+1 gồm n + 1 đỉnh. Việc ghép
thêm đỉnh x vẫn bảo toàn tính chất của Gn (tức là, ba đỉnh bất kỳ đều
không cùng bậc). Mặt khác, đồ thị Gn không có đỉnh bậc 0, nên trong
Gn+1 ba đỉnh bất kỳ đều không cùng bậc.
b. Nếu Gn không có đỉnh bậc n − 1. Khi đó, tất cả các đỉnh của Gn
đều có bậc không vượt quá n−2. Thêm vào Gn đỉnh x (không thuộc Gn)
và nối x với từng đỉnh thuộc Gn bằng một cạnh được đồ thị Gn+1 gồm
n + 1 đỉnh. Đỉnh x có bậc n, còn bậc của mỗi đỉnh thuộc Gn trong Gn+1
được tăng lên một đơn vị, nhưng đều không vượt quá n−1 và trong bậc
mới ba đỉnh bất kỳ của Gn vẫn không cùng bậc.
Khẳng định được chứng minh.
Định lí 1.3.7. Trong đồ thị G = (X, E) với ít nhất kn + 1 đỉnh, mỗi
đỉnh có bậc không nhỏ hơn (k − 1)n + 1 luôn tồn tại đồ thị con đầy đủ
gồm k + 1 đỉnh.
Chứng minh.
Ta sẽ chứng minh định lý bằng phương pháp quy nạp theo k.
1) Với k = 1 khẳng định hiển nhiên đúng.
2) Với k = 2 có thể làm chặt chẽ hơn giả thiết. Nếu đồ thị 2n + 1
đỉnh mà mỗi đỉnh có bậc không nhỏ hơn n, thì nó có đồ thị con 3 đỉnh
đầy đủ.
Thật vậy, xét đỉnh x tùy ý, còn y là một trong các đỉnh kề với x. Tổng
số đỉnh kề với x và y không nhỏ hơn 2n, nhưng số đỉnh khác x và y chỉ
là 2n − 1. Bởi vậy, phải có ít nhất một đỉnh z được tính hai lần. Khi đó,
x, y, z tạo thành một đồ thị con đầy đủ ba đỉnh.
3) Giả sử khẳng định trên đúng với k. Cần suy ra tính đúng đắn của
khẳng định đối với k + 1.
Theo giả thiết, thông đồ thị G gồm (k + 1)n + 1 đỉnh, số đỉnh kề
với đỉnh x tùy ý không nhỏ hơn kn + 1, nên số đỉnh không kề với x
sẽ không vượt quá n. Bởi vậy, mỗi đỉnh y kề với x thì nó kề với nhiều
nhất n đỉnh không kề với đỉnh x. Do đó, đỉnh y phải kề với ít nhất
kn + 1 − n = (k − 1)n + 1 đỉnh kề với đỉnh x. Xét đồ thị con G1 gồm các
đỉnh kề với x. Đồ thị con G1 có ít nhất kn + 1 đỉnh và mỗi đỉnh của nó
kề với ít nhất (k − 1)n + 1 đỉnh thuộc G1, nên theo giả thiết quy nạp,
12
trong G1 có đồ thị con đầy đủ G2 gồm k + 1 đỉnh. Vì đỉnh x kề với từng
đỉnh thuộc G2, nên đỉnh x kết hợp với các đỉnh thuộc G2 lập thành một
đồ thị con đầy đủ gồm k + 2 đỉnh thong đồ thị G.
Khẳng định được chứng minh.
1.4 Xích, chu trình, đường, vòng
1.4.1 Xích, chu trình
Giả sử G(X, E) là một đồ thị hay đa đồ thị vô hướng:
Dãy α các đỉnh của G(X, E):
α = [x1, x2, ..., xi, xi+1, ..., xn−1, xn]
được gọi là một xích hay một dây chuyền, nếu ∀i(1 ≤ i ≤ n − 1) cặp
đỉnh xi, xi+1 kề nhau.
Tổng số vị trí của tất cả các cạnh xuất hiện trong xích α được gọi là
độ dài của xích α, ký hiệu |α|.
Các đỉnh x1, xn được gọi là hai đỉnh đầu của xích α. Để chỉ rõ đỉnh
đầu và đỉnh cuối ta còn ký hiệu α bằng α[x1, xn].
Một xích có hai đầu trùng nhau được gọi là một chu trình.
Xích (chu trình) α được gọi là xích (chu trình) đơn (sơ cấp hay cơ
bản), nếu nó đi qua mỗi cạnh (mỗi đỉnh) không quá một lần.
Ví dụ 1.4. Cho đồ thị
Hình 1.8
13
α1 = [5, 1, 4, 2, 1] là một dây chuyền không sơ cấp.
α2 = [1, 2, 3, 4] là một dây chuyền sơ cấp.
α3 = [1, 5, 1] và α4 = [1, 2, 3, 4, 1] là các chu trình đơn và sơ cấp.
α5 = [1, 2, 4, 3, 2, 1] là chu trình đơn nhưng không sơ cấp.
1.4.2 Đường, vòng
Giả sử G(X, E) là một đồ thị hay đa đồ thị có hướng. Dãy đỉnh β
của G(X, E) :
β = [x1, x2, ..., xi, xi+1, ..., xm−1, xm]
được gọi là một đường hay một đường đi nếu ∀i(1 ≤ i ≤ m − 1), đỉnh
xi là đỉnh đầu, còn đỉnh xi+1 là đỉnh cuối của một cung nào đó.
Tổng số vị trí của tất cả các cung xuất hiện trong β được gọi là đồ
dài của đường β, ký hiệu: |β|.
Đỉnh x1 được gọi là đỉnh đầu còn xm là đỉnh cuối của đường β. Người
ta còn nói rằng, đường β xuất phát từ đỉnh x1 và đi tới xm. Đường β
còn được ký hiệu bằng β[x1, xm].
Một đường có đỉnh đầu và đỉnh cuối trùng nhau được gọi là một vòng.
Đường (vòng) β được gọi là đường (vòng) đơn (sơ cấp hay cơ bản),
nếu nó đi qua mỗi cạnh (mỗi đỉnh) không quá một lần.
Ví dụ 1.5. Cho đồ thị có hướng (hình 1.9):
β1 = [1, 2, 4, 3, 5, 1] là một vòng đơn và sơ cấp.
β2 = [1, 4, 3, 5] là một đường đơn và sơ cấp.
β3 = [1, 4, 2, 5] không phải là đường.
β4 = [1, 2, 4, 3, 2, 5] là một đường đơn nhưng không sơ cấp.
β5 = [1, 4, 2, 5, 1, 2, 5] không phải là đường đơn và cũng không phải là
đường sơ cấp.
β6 = [1, 2, 4, 3, 2, 5, 1] là một vòng đơn nhưng không là vòng sơ cấp.
14
Hình 1.9
1.4.3 Một số tính chất
Định lí 1.4.1. Trong một đồ thị vô hướng với n đỉnh (n ≥ 3) và các
đỉnh đều có bậc không nhỏ hơn 2 luôn tồn tại chu trình sơ cấp.
Chứng minh.
Vì đồ thị hữu hạn, mà xích sơ cấp qua từng đỉnh không quá một lần
nên số xích sơ cấp trong đồ thị G = (X, E) là một số hữu hạn. Bởi vậy,
luôn xác định được xích sơ cấp có độ dài cực đại trong đồ thị G = (X, E).
Giả sử α = [x1, x2, ..., xk−1, xk] là một trong những xích sơ cấp có độ
dài cực đại. Do bậc của mỗi đỉnh không nhỏ hơn 2, nên x1 phải kề với
một đỉnh y nào đó khác với x2.
Nếu y /∈ α, tức là y = xi, (3 ≤ i ≤ k), thì xích sơ cấp
α = [y, x1, x2, ..., xk−1, xk] có độ dài |α | = |α| + 1 > |α|. Ta đã đi
tới mâu thuẫn với tính chất độ dài cực đại của α. Bởi vậy, y ∈ α tức
y ≡ xi, (3 ≤ i ≤ k), nên trong đồ thị G = (X, E) có chu trình sơ cấp
β = [x1, x2, ..., xi, x1]
Khẳng định được chứng minh.
Định lí 1.4.2. Trong một đồ thị vô hướng với n đỉnh (n ≥ 4) và các
đỉnh đều có bậc không nhỏ hơn 3 luôn tồn tại chu trình sơ cấp độ dài
chẵn.
15
Chứng minh.
Giả sử α là một trong những xích sơ cấp có độ dài cực đại
α = [x1, x2, ..., xi−1, xi, xi+1, ..., xj−1, xj, xj+1, ..., xk−1, xk]
Vì α có độ dài cực đại, mà bậc của x1 không nhỏ hơn 3, nên x1 phải
kề với hai đỉnh khác thuộc α: xi, (3 ≤ i ≤ k), xj, (3 ≤ j ≤ k). Khi đó có
hai chu trình sơ cấp:
α1 = [x1, x2, ..., xi−1, xi, x1]
α2 = [x1, x2, ..., xi−1, xi, xi+1, ..., xj−1, xj, x1]
1) Nếu một trong hai chu trình α1, α2 có độ dài chẵn thì khẳng định
được chứng minh.
2) Nếu ngược lại, cả hai chu trình α1, α2 đề có độ dài lẻ.
Khi đó xích: α3 = [x1, x2, ..., xi−1, xi] có độ dài chẵn,
còn xích α4 = [xi, xi+1, ..., xj−1, xj, x1] có độ dài lẻ,
nên chu trình α5 = [x1, xi, xi+1, ..., xj−1, xj, x1] có độ dài chẵn.
Khẳng định được chứng minh.
1.5 Đồ thị liên thông
1.5.1 Định nghĩa
Hai đỉnh x, y của đồ thị G = (X, E) được gọi là cặp đỉnh liên thông
nếu hoặc giữa x và y có ít nhất một xích nối với nhau , hoặc tồn tại ít
nhất một đường đi từ x sang y hoặc từ y sang x.
Hình 1.10
Trong hình 1.10 cặp đỉnh x,y là liên thông
Đồ thị vô hướng G = (X, E) được gọi là đồ thị liên thông, nếu mọi
cặp đỉnh của nó đều liên thông.
Đồ thị có hướng G = (X, E) được gọi là đồ thị liên thông mạnh, nếu
mọi cặp đỉnh của nó đều liên thông.
16
Giả sử a là đỉnh bất kỳ của đồ thị G = (X, E). Dùng Ca để ký hiệu
tập con của các đỉnh thuộc G, gồm đỉnh a và tất cả các đỉnh liên thông
với a trong đồ thị G.
Đồ thị con của G có tập đỉnh Ca được gọi là một thành phần liên
thông của đồ thị G
Ví dụ 1.6. Cho đồ thị G có bốn thành phần liên thông:
Các đồ thị con G1, G3, G4 là liên thông
Đồ thị con G2 liên thông mạnh
Hình 1.11
1.5.2 Tính chất
Định lí 1.5.1. Đồ thị vô hướng tùy ý với n đỉnh (n ≥ 2), mà tổng bậc
của hai đỉnh tùy ý không nhỏ hơn n là đồ thị liên thông.
Chứng minh.
Giả sử đồ thị vô hướng G(X, E) có n đỉnh (n ≥ 2). Với mọi cặp đỉnh
a, b của đồ thị ta đều có:
m(a) + m(b) ≥ n (1)
Nhưng a, b không liên thông. Khi đó trong đồ thị G tồn tại hai thành
phần liên thông:
G1 chứa a và có n1 đỉnh, còn G2 chứa b và có n2 đỉnh.
17
Vì G1, G2 là các thành phần liên thông của G nên n1 + n2 ≤ n Khi
đó
m(a) + m(b) ≤ (n1 − 1) + (n2 − 1) = n1 + n2 − 2 ≤ n − 2 < n (2)
Như vậy, (1) và (2) mâu thuẫn nhau, nên đồ thị G phải liên thông.
Khẳng định được chứng minh.
Từ định lý trên suy ra hệ quả sau:
Hệ quả 1.5.1. Đồ thị, mà bậc của mỗi đỉnh không nhỏ hơn nửa số đỉnh,
là đồ thị liên thông.
Định lí 1.5.2. Nếu đồ thị có đúng hai đỉnh bậc lẻ, thì hai đỉnh này phải
liên thông.
Chứng minh.
Giả sử đồ thị G(X, E) có đúng hai đỉnh bậc lẻ và hai đỉnh đó là a và
b.
Giả sử a, b không liên thông với nhau.
Khi đó chúng phải thuộc hai thành phần liên thông khác nhau của đồ
thị G. Chẳng hạn G1 chứa đỉnh a, còn G2 chứa đỉnh b.
Bậc của đỉnh a trong G1 cũng chính là bậc của a trong G, nên trong
G1 đỉnh a vẫn có bậc lẻ. Điều này mâu thuẫn với định lý 1.2. Bởi vậy a,
b phải liên thông.
Khẳng định được chứng minh.
1.6 Số ổn định trong, số ổn định ngoài
1.6.1 Số ổn định trong
1. Tập ổn định trong
Giả sử có đồ thị G(X, E). Tập con A ⊆ X các đỉnh của đồ thị G được
gọi là tập ổn định trong, nếu mọi cặp đỉnh thuộc A đều không kề nhau
(không có cạnh hoặc cung nối với nhau).
Tập con B ⊆ X các đỉnh của đồ thị G được gọi là tập ổn định trong
cực đại, nếu B là tập ổn định trong và nếu thêm vào B một đỉnh tùy ý
x ∈ X, thì tập con nhận được B ∪ {x} sẽ không ổn định trong.
2. Tính chất
Nếu A là tập ổn định trong, thì mọi tập con của A đều phải ổn định
trong.
18
3. Số ổn định trong
Số phần tử của một trong những tập ổn định trong có lực lượng lớn
nhất được gọi là số ổn định trong của đồ thị G, đồng thời được ký hiệu
bằng α(G).
1.6.2 Số ổn định ngoài
1. Tập ổn định ngoài
Giả sử có đồ thị G(X, E). Tập con B ⊆ X các đỉnh của đồ thị G
được gọi là tập ổn định ngoài, nếu với mọi đỉnh x thuộc tập XB đều
tồn tại đỉnh y ∈ B, để hoặc từ x sang y có cung hoặc cặp đỉnh x, y được
nối bằng một cạnh.
2. Tính chất
Nếu B là tập ổn định ngoài, thì mọi tập chứa B đều ổn định ngoài.
3. Số ổn định ngoài
Số phần tử của một trong những tập ổn định ngoài có lực lượng bé
nhất được gọi là số ổn định ngoài của đồ thị G, đồng thời được ký hiệu
bằng β(G).
Ví dụ 1.7. Cho đồ thị G như hình 1.12. Hãy tìm tất cả các tập ổn định
trong,số ổn định trong và số ổn định ngoài của đồ thị G.
Các tập ổn định trong
Hình 1.12
19
- Vì đồ thị không có khuyên, nên mỗi đỉnh lập thành một tập ổn định
trong:
M1 = {x1}, M2 = {x2}, M3 = {x3}, M4 = {x4}, M5 = {x5},
M6 = {x6}, M7 = {x7}.
- Các tập ổn định trong gồm 2 đỉnh:
M8 = {x1, x3}, M9 = {x1, x4}, M10 = {x1, x6}, M11 = {x2, x5},
M12 = {x2, x7}, M13 = {x3, x5}, M14 = {x3, x6}, M15 = {x3, x7},
M16 = {x4, x7}, M17 = {x5, x6}.
- Các tập ổn định trong gồm 3 đỉnh:
M18 = {x1, x3, x6}, M19 = {x3, x6, x5}.
Số ổn định trong
Ta nhận thấy M18 = {x1, x3, x6}, M19 = {x3, x6, x5} là các tập ổn định
trong có lực lượng lớn nhất, nên số phần tử của nó chính là số ổn định
trong. Tức là, α(G) = |{x1, x3, x6}| = |{x3, x6, x5}| = 3.
Các tập ổn định ngoài
- Tập ổn định ngoài một đỉnh không có.
- Tập ổn định ngoài gồm 2 đỉnh, chẳng hạn N1 = {x1, x2}, N2 =
{x1, x4}.
Số ổn định ngoài
β(G) = |{x1, x2}| = 2.
1.6.3 Các thuật toán tìm số ổn định trong, số ổn định ngoài.
1.6.3.1. Thuật toán tìm số ổn định trong.
- Bước 1: Tìm các tập ổn định trong có 2 phần tử bằng cách xét tất
cả tổ hợp chập 2 của n phần tử (n số các đỉnh), kiểm tra những tập nào
mà phần tử tương ứng không kề nhau thì tập đó là ổn định trong;
- Bước 2: Duyệt từng tập có 2 phần tử và bổ sung thêm phẩn tử thứ
3 và kiểm tra từng cặp như bước 1, tập nào thỏa mãn ta được tập ổn
định trong 3 phần tử.
.........
- Bước k: Giả sử ta đã tìm được m tập con ổn định trong có k+1
phần tử
+ Duyệt từng tập và bổ sung vào các tập đó thêm 1 phần tử.
+ Nếu không có tập nào bổ sung được nữa thì dừng.
20
1.6.3.2. Thuật toán tìm số ổn định ngoài.
Xét G(X, E) với X = {x1, x2, ..., xn}
- Bước 1: Xác định các tập ∆(xi), i = 1, 2, ..., n
với ∆(xi) = {xi và các đỉnh kề với xi}
- Bước 2: Từ các tập ∆(x1), ∆(x2), ..., ∆(xn) ta tìm tập B = {xk1, xk2, ..., xkm}
sao cho ∆(xk1) ∪ ∆(xk2) ∪ ... ∪ ∆(xkm) = X.
Khi đó B là tập ổn định ngoài cực tiểu.
1.7 Nhân của đồ thị và ứng dụng vào trò chơi
1.7.1 Định nghĩa
Giả sử có đồ thị G(X, U). Tập đỉnh S ⊆ X được gọi là nhân của đồ
thị G, nếu nó vừa là tập ổn định trong lại vừa là tập ổn định ngoài.
Do S là tập ổn định trong nên nó không chứa khuyên. Mặt khác S ổn
định ngoài nên nó phải chứa tất cả các đỉnh biệt lập và các đỉnh không
có cung đi ra.
Ví dụ 1.8. Cho hai đồ thị như hình 1.13.
Đồ thị hình 1.13(a) có hai nhân là {1, 4} và {2, 3}
Đồ thị hình 1.13(b) không có nhân vì các tập ổn định trong chỉ gồm
1 đỉnh, còn các tập ổn định ngoài phải gồm ít nhất hai đỉnh.
Hình 1.13
21
1.7.2 Tính chất
Định lí 1.7.1. Nếu đồ thị G(X, U) có số ổn định trong nhỏ hơn số ổn
định ngoài thì nó không có nhân.
Chứng minh.
Giả sử trong đồ thị G(X, U),
α(G) < β(G) (1)
nhưng lại có nhân và S là một trong những nhân của đồ thị G. Khi đó,
theo định nghĩa:
α(G) = max{|A|A ∈ H(G)} ≥ |S| ≥ min{|B|B ∈ K(G)} = β(G) (2)
trong đó, H(G) là tập gồm các tập ổn định trong, còn K(G) là tập gồm
các tập ổn định ngoài của đồ thị G.
So sánh (1) và (2) đi tới mâu thuẫn, nên G không thể có nhân.
Định lý được chứng minh.
Định lí 1.7.2. Nếu S là nhân của đồ thị G(X, U), thì nó cũng là tập ổn
định trong cực đại.
Chứng minh.
Giả sử S là nhân của đồ thị G(X, U) và x là đỉnh tùy ý không thuộc
S. Xét tập S ∪ {x}. Vì S là nhân và x /∈ S, nên ∃y ∈ S, để x, y được nối
bằng một cạnh hoặc từ x sang y có cung. Bởi vậy, tập S ∪ {x} không ổn
định trong, nên S là tập ổn định trong cực đại.
Định lý được chứng minh.
Định lí 1.7.3. Trong đồ thị vô hướng không có khuyên mọi tập ổn định
trong cực đại đều là nhân.
Chứng minh.
Giả sử B là một tập ổn định trong cực đại của đồ thị vô hướng
G(X, E). Khi đó ∀x ∈ (XB) đều ∃y ∈ B để x, y có cạnh nối với nhau,
nên B đồng thời là tập ổn định ngoài.
Định lý được chứng minh.
Giả sử có đồ thị G(X, E) và A ⊆ X. Dùng D(A) để ký hiệu tập đỉnh,
mà mỗi đỉnh này có cạnh nối với ít nhất một đỉnh thuộc A. Còn D+
(A)
là tập đỉnh mà mỗi đỉnh này có ít nhất một đỉnh thuộc A có cung đi tới
nó. D−
(A) là tập đỉnh mà mỗi đỉnh này có cung đi tới ít nhất một đỉnh
thuộc A.
22
Hệ quả 1.7.1. Mọi đồ thị vô hướng không có khuyên đều có nhân.
Chứng minh.
Thật vậy, giả sử đồ thị vô hướng G(X, E) là đồ thị vô hướng không
có khuyên. Khi đó mỗi đỉnh đều lập thành một tập ổn định trong.
Xuất phát từ đỉnh tùy ý x0. Đặt S0 = {x0}, sau đó chọn đỉnh tùy ý
x1 /∈ D(x0) và đặt S1 = S0 ∪ {x1} = {x0, x1}.
Tiếp theo, chọn đỉnh tùy ý x2 /∈ D(S1)... Vì đồ thị G hữu hạn, nên
sớm hay muộn quá trình phải dừng lại, tức là có số tự nhiên n để
D(Sn) = XSn. Với cách chọn này Sn là tập ổn định trong cực đại, nên
theo định lý 1.16, nó là nhân của đồ thị G.
Hệ quả được chứng minh.
1.7.3 Trò chơi Nim
Giữa hai đấu thủ, được ký hiệu là A và B, có một đồ thị G(X, E) cho
phép xác định một trò chơi nào đó. Trong trò chơi này mỗi thế là một
đỉnh của đồ thị.
Đỉnh khởi đầu x0 được chọn bằng cách gắp thăm và các đấu thủ lần
lượt đi: Đầu tiên đấu thủ A chọn đỉnh x1 trong tập D(x0) ∪ D+
(x0); sau
đó đấu thủ B chọn đỉnh x2 trong tập D(x1) ∪ D+
(x1); tiếp theo đấu thủ
A chọn đỉnh x3 trong tập D(x2) ∪ D+
(x2),...Nếu một trong hai đấu thủ
chọn được đỉnh xk, mà D(xk) ∪ D+
(xk) = ∅, thì ván đó kết thúc. Đấu
thủ nào chọn được đỉnh cuối cùng thì thắng cuộc và đấu thủ kia thua
cuộc.
Để kỷ niệm trò tiêu khiển quen thuộc mà Nim đã tổng quát hóa,
người ta gọi trò chơi mô tả ở trên là trò chơi Nim và dùng ngay đồ thị
G(X, E) xác định nó để ký hiệu cho trò chơi này.
Định lí 1.7.4. Nếu đồ thị G(X, E) có nhân S và nếu một đấu thủ đã
chọn được một đỉnh trong nhân S, thì việc chọn này bảo đảm cho đấu
thủ đó thắng hoặc hòa.
Chứng minh.
Thật vậy, nếu đấu thủ A chọn được đỉnh x1 ∈ S, thì hoặc
D(x1) ∪ D+
(x1) = ∅ tức là A thắng cuộc, hoặc D(x1) ∪ D+
(x1) = ∅, thì
đối phương B buộc phải chọn đỉnh x2 ∈ (X − S). Khi đó đến lượt mình
đấu thủ A lại có thể chọn x3 ∈ S và cứ như thế mãi.
23
Vì đồ thị G có hữu hạn đỉnh, nên đến một lúc nào đó một trong hai
đấu thủ bằng cách chọn dược đỉnh xk ∈ S, mà D(xk) ∪ D+
(xk) = ∅.
Theo cách chọn trên, thì đấu thủ A đến lượt chọn xk, nên A chính là
người thắng cuộc.
Định lý được chứng minh.
1.7.4 Trò chơi bốc các vật
1. Trò chơi
Trên bàn có một đống gồm m vật. Hai đấu thủ A, B thực hiện trò
chơi bốc các vật theo nguyên tắc:
1) Người đi đầu xác định ngẫu nhiên (bằng gắp thăm hoặc gieo đòng
tiền).
2) Với k(1 ≤ k < m) mỗi người đến lượt phải bốc ít nhất một vật và
không được bốc quá k vật.
3) Người bốc được vật cuối cùng thắng(thua)cuộc.
Khi tham gia cuộc chơi mỗi người đều phải tìm cách thực hiện để
chiến thắng.
Mỗi bước chơi đều có vai trò quyết định của nó. Song bước một có ý
nghĩa quyết định hơn cả. Bởi thế người đi đầu có phần chủ động hơn.
Nếu người đi đầu có thuật toán chơi đúng, thì nhất định chiến thắng.
Bởi vậy cần đưa ra thuật toán chơi đúng cho người đi đầu.
2. Thuật toán chơi dựa vào nhân đồ thị
a.Trường hợp bốc được vật cuối cùng thắng cuộc
1)Xây dựng đồ thị xác định trò chơi:
Cần xác định đỉnh và cung của đồ thị tương ứng với số lượng vật có
thể có là 0, 1, 2, ..., i, i + 1, ..., m − 1, m. Dùng ngay số lượng vật để ghi
trên các điểm tương ứng.
i) Đối với mỗi đỉnh x ≥ k có cung đi tới từng đỉnh thuộc tập
D+
(x) = {x − 1, x − 2, ..., x − k + 1, x − k}
.
ii) Đối với mỗi đỉnh y(1 ≤ y < k) có cung đi tới từng đỉnh thuộc tập
D+
(y) = {0, 1, 2, ..., y − 1}
.
2)Xác định nhân của đồ thị:
24
Vì từng cặp đỉnh thuộc tập
M = {0, k + 1, 2(k + 1), ...,
m
k + 1
(k + 1)}
không kề nhau và mỗi đỉnh i ∈ M đều có cung đi tới đỉnh
i
k + 1
(k + 1),
nên tập M là nhân của đồ thị G.
3) Thuật toán:
Giả sử A là người được đi đầu. Khi đó A bốc
m −
m
k + 1
(k + 1)
vật, tức đi theo cung
m,
m
k + 1
(k + 1)
để đến đỉnh
m
k + 1
(k + 1) ∈ M.
Đến lượt mình, giả sử B bốc t(1 ≤ t ≤ k) vật. Tiếp theo A bốc k+1−t
vật, tức xuất phát từ đỉnh
m
k + 1
(k + 1) − t
đi theo cung
m
k + 1
(k + 1) − t,
m
k + 1
k
để đến đỉnh
m
k + 1
k ∈ M.
Cứ tiếp tục như vậy đấu thủ B chỉ có thể đạt được đỉnh ngoài nhân
M, còn đấu thủ A lần lượt đạt được các đỉnh
m
k + 1
(k + 1),
m
k + 1
− 1 (k + 1), ...
Cuối cùng A đạt được đỉnh 0, tức là A bốc được vật cuối cùng nên thắng
cuộc.
25
Ví dụ 1.9. ([4]Lý thuyết đồ thị và các bài toán không mẫu mực)
Có 13 viên bi. Hai em A và B thực hiện trò chơi bốc bi theo nguyên tắc
sau:
1) Mỗi người đến lượt chỉ có thể bốc từ 1 đến 3 viên.
2) Ai bốc được viên bi cuối cùng thì thắng cuộc.
Nếu A được đi đầu, thì A phải có cách bốc bi như thế nào để đảm bảo
thắng cuộc?
Giải
1)Xây dựng đồ thị G xác định trò chơi (hình 1.14).
Nhân của đồ thị M = {0, 4, 8, 12}
Hình 1.14
2)Thuật toán:
Để đảm bảo chiến thắng thì A phải bốc 1 viên bi để số bi còn lại là
12 (12 ∈ M). Tiếp sau, nếu B bốc s(1 ≤ s ≤ 3) viên bi, thì A phải bốc
4 − s viên bi để số bi còn lại là 8 (8 ∈ M),nếu B bốc t(1 ≤ t ≤ 3) viên
bi, thì A phải bốc 4−t viên bi để số bi còn lại là 4 (4 ∈ M). Tiếp theo B
chỉ có thể bốc từ 1 đến 3 viên bi, nên số bi còn lại sẽ không ít hơn 1 và
không quá 3 viên nên A được bốc cả số bi còn lại (tức là đạt được đỉnh
0 thuộc M). Do đó A thắng.
b.Trường hợp bốc được vật cuối cùng thua cuộc
1)Xây dựng đồ thị xác định trò chơi:
Cần xác định đỉnh và cung của đồ thị tương ứng với số lượng vật có
thể có là 0, 1, 2, ..., i, i + 1, ..., m − 1, m. Dùng ngay số lượng vật để ghi
26
trên các điểm tương ứng.
i) Đối với mỗi đỉnh x ≥ k có cung đi tới từng đỉnh thuộc tập
D+
(x) = {x − 1, x − 2, ..., x − k + 1, x − k}.
ii) Đối với mỗi đỉnh y(1 ≤ y < k) có cung đi tới từng đỉnh thuộc tập
D+
(y) = {0, 1, 2, ..., y − 1}.
2)Xác định nhân của đồ thị:
Vì từng cặp đỉnh thuộc tập
N = {1, k + 2, 2(k + 1) + 1, ...,
m
k + 1
(k + 1) + 1}
không kề nhau và mỗi đỉnh i /∈ N đều có cung đi tới đỉnh
i
k + 1
(k + 1) + 1,
nên tập N là nhân của đồ thị con không chứa đỉnh 0.
3) Thuật toán:
Giả sử A là người được đi đầu. Khi đó A bốc
m −
m
k + 1
(k + 1) − 1
vật, tức đi theo cung
m,
m
k + 1
(k + 1) + 1
để đến đỉnh
m
k + 1
(k + 1) + 1 ∈ N.
Đến lượt mình, giả sử B bốc t(1 ≤ t ≤ k) vật. Tiếp theo A bốc k+1−t
vật, tức xuất phát từ đỉnh
m
k + 1
(k + 1) + 1 − t
đi theo cung
m
k + 1
(k + 1) + 1 − t,
m
k + 1
− 1 (k + 1) + 1
27
để đến đỉnh
m
k + 1
− 1 (k + 1) + 1 ∈ N.
Cứ tiếp tục như vậy đấu thủ B chỉ có thể đạt được đỉnh ngoài nhân
N, còn đấu thủ A lần lượt đạt được các đỉnh
(
m
k + 1
(k + 1) + 1 − t,
m
k + 1
− 1 (k + 1) + 1, ...
Cuối cùng A đạt được đỉnh 1 ∈ N, tức là sau khi đấu thủ A bốc lần cuối
trên bàn còn đúng 1 vật. Khi đó, B phải bốc vật cuối cùng, nên thua
cuộc.
Ví dụ 1.10. ([4]Lý thuyết đồ thị và các bài toán không mẫu mực)
Có 14 que diêm trên bàn. Hai em A và B thực hiện trò chơi bốc diêm
theo nguyên tắc sau:
1) Mỗi người đến lượt chỉ có thể bốc từ 1 đến 3 que diêm.
2) Ai bốc phải que diêm cuối cùng thì thua cuộc.
Nếu A được đi đầu, thì A phải có cách bốc diêm như thế nào để đảm
bảo thắng cuộc?
Giải
Hình 1.15
1)Xây dựng đồ thị G xác định trò chơi (hình 1.15).
Nhân của đồ thị N = {1, 5, 9, 13}
2)Thuật toán:
28
Để đảm bảo chiến thắng thì A phải bốc 1 que diêm để số diêm còn
lại là 13 (13 ∈ N). Tiếp sau, nếu B bốc s(1 ≤ s ≤ 3) que diêm, thì A
phải bốc 4 − s que diêm để số diêm còn lại là 9 que (9 ∈ N), nếu B bốc
t(1 ≤ t ≤ 3) que diêm, thì A phải bốc 4 − t que để số diêm còn lại là 5
(5 ∈ N). Nếu B bốc u(1 ≤ u ≤ 3) que diêm, thì A phải bốc 4 − u que
diêm để số diêm còn lại là 1 (1 ∈ N). Chỉ còn lại một que diêm nên B
phải bốc nốt. Do đó A thắng.
1.8 Cây và bụi
1.8.1 Định nghĩa
Một đồ thị vô hướng liên thông, không có chu trình và có ít nhất hai
đỉnh được gọi là một cây (hình 1.16)
Hình 1.16
Đồ thị hữu hạn có hướng G = (X, U) là một bụi gốc x1 ∈ X, nếu nó
có ít nhất hai đỉnh và thỏa mãn ba điều kiện sau:
1. Mỗi đỉnh khác x1 là điểm cuối của một cung duy nhất.
2. Đỉnh x1 không là điểm cuối của bất kỳ một cung nào.
3. Đồ thị G = (X, U) không có vòng. (Hình 1.17)
Hình 1.17
29
1.8.2 Đặc điểm của cây và bụi
Định lí 1.8.1. Giả sử H là một đồ thị vô hướng với n đỉnh n > 1. Để
đặc trưng cho một cây thì sáu tính chất sau đây là tương đương:
1. H liên thông và không có chu trình;
2. H không có chu trình và có n − 1 cạnh;
3. H liên thông và có n − 1 cạnh;
4. H không có chu trình và nếu thêm một cạnh nối giữa hai đỉnh bất
kì không kề nhau thì đồ thị nhận được H’ có một chu trình (và chỉ một
mà thôi);
5. H liên thông và khi bớt một cạnh bất kì thì đồ thị mất tính liên
thông;
6. Mọi cặp đỉnh của H đều được nối với nhau bằng một xích và chỉ
một xích mà thôi.
Chứng minh.
Định lý được chứng minh theo phương pháp vòng tròn.
Ký hiệu số cạnh của đồ thị H bằng m. Dùng p để ký hiệu số thành
phần liên thông và V(H) là số chu trình của đồ thị H.
(1) ⇒ (2): Theo tính chất (1): p = 1, V (H) = m − n + 1 = 0, nên
m = n − 1.
(2) ⇒ (3): Theo tính chất (2): m = n − 1, V (H) = 0, nên:
V (H) = m − n + p = n − 1 − n + p = 0 ⇒ p = 1
Bởi vậy, H liên thông và có n − 1 cạnh.
(3) ⇒ (4): Theo tính chất (3): p = 1, m = n − 1, nên
V (H) = m − n + p = n − 1 − n + 1 = 0
Tức là H không có chu trình; ngoài ra, nếu thêm vào một cạnh nối giữa
hai đỉnh không kề nhau, thì đồ thị H’ nhận được sẽ có chu số:
V (H ) = m + 1 − n + 1 = n − 1 + 1 − n + 1 = 1
nên đồ thị H’ có một chu trình và chỉ một mà thôi.
(4) ⇒ (5): Lấy hai đỉnh bất kỳ x, y của đồ thị H. Theo tính chất (4):
nếu thêm vào cạnh (x,y) thì đồ thị mới nhận được H’ có chu trình, điều
đó chứng tỏ giữa x, y đã có xích nối với nhau, tức H đã liên thông.
Giả sử bớt đi một cạnh nào đó, chẳng hạn (u,v) mà đồ thị nhận được
vẫn liên thông. Điều này chứng tỏ trong đồ thị H giữa các đỉnh u, v
30
ngoài cạnh (u,v) còn một xích nữa nối giữa chúng, tức là trong H có ít
nhất một chu trình đi qua u, v. Ta đi tới mâu thuẫn với tính chất (4):
Đồ thị H không có chu trình. Bởi vậy, nếu bớt đi một cạnh tùy ý thì đồ
thị nhận được từ H sẽ không liên thông.
(5) ⇒ (6): Giả sử trong H tồn tại cặp đỉnh nào đó, chẳng hạn x, y
được nối với nhau từ hai xích trở lên. Khi đó, nếu ta bỏ đi một cạnh
nào đó thuộc một trong hai xích này, thì xích còn lại vẫn bảo đảm cho
x, y liên thông. Như vậy, ta đã đi tới mâu thuẫn với tính chất (5). Do
đó, mọi cặp đỉnh của H đều được nối với nhau bằng một xích và chỉ một
mà thôi.
(6) ⇒ (1): Giả sử H không liên thông. Khi đó có ít nhất một cặp đỉnh
không có xích nối với nhau, nên mâu thuẫn với tính chất (6).
Giả sử H có chu trình. Khi đó có ít nhất một cặp đỉnh nằm trên chu
trình này được nối với nhau bằng ít nhất hai xích. Như vậy, ta cũng đi
đến mâu thuẫn với tính chất (6). Bởi vậy, đồ thị H có tính chất (1).
Định lý được chứng minh.
Định lí 1.8.2. Một cây có ít nhất hai đỉnh treo.
Chứng minh.
Giả sử cây H chỉ có không quá một đỉnh treo. Ta tưởng tượng có một
khách bộ hành đi theo đồ thị đó, xuất phát từ một đỉnh tùy ý (trong
trường hợp đồ thị không có đỉnh treo) hay từ đỉnh treo (trong trường
hợp đồ thị có một đỉnh treo): Nếu hành khách tự cấm mình không đi
qua một cạnh hai lần, khi đó không thể gặp một đỉnh hai lần (do đồ thị
không có chu trình). Mặt khác, khi tới một đỉnh người đó luôn luôn có
thể đi ra bằng một cạnh mới (vì mỗi đỉnh khác đỉnh xuất phát đều có ít
nhất hai cạnh). Như vậy khách bộ hành sẽ đi mãi không bao giờ dừng
lại. Đó là điều không thể xảy ra, vì đồ thị H có hữu hạn đỉnh. Vậy đồ
thị H không thể có ít hơn hai đỉnh treo.
Định lý được chứng minh.
Định lí 1.8.3. Mọi bụi khi bỏ định hướng các cạnh đều trở thành cây.
Chứng minh.
Giả sử bụi H = (X, U) có gốc là x1 và đồ thị vô hướng G = (X, E)
nhận được từ bụi H sau khi bỏ định hướng các cung.
1. Đồ thị G liên thông: Do điều kiện (1) mỗi đỉnh x = x1 đều có
đường từ x1 đi tới. Thật vậy, giả sử x = x1 và từ x1 không có đường đi
tới x.
31
Nếu x là đỉnh biệt lập, thì nó không thể là đỉnh cuối của một cung
nào, còn nếu x không phải là đỉnh biệt lập, thì phải có đỉnh y là điểm
xuất phát của một đường đi tới x. Nhưng do từ x1 không có đường đi tới
x nên y = x1; mà nó cũng không là điểm cuối của bất kỳ cung nào. Như
vậy, ta đã đi tới mâu thuẫn với điều kiện (1). Do đó mọi đỉnh x = x1 từ
x1 đều có đường đi tới nó, nên trong G mọi đỉnh x đều có xích nối với
đỉnh x1. Bởi vậy, G liên thông.
2. Đồ thị G không có chu trình. Thật vậy, giả sử G có chu trình, thì
trong H dãy cung tương ứng với các cạnh thuộc chu trình này sẽ hoặc
lập thành một vòng hoặc có ít nhất hai cung có chung điểm cuối. Như
vậy, ta đã đi tới mâu thuẫn với điều kiện (1) hoặc điều kiện (3). Nên đồ
thị G không có chu trình và liên thông. Do đó G là một cây.
Định lý được chứng minh.
32
Chương 2
Một số bài toán đồ thị cơ bản
2.1 Bài toán về đường đi
2.1.1 Đường đi Euler - Chu trình Euler.
2.1.1.1. Bài toán mở đầu :
Bài toán 7 cây cầu ở K¨onigsberg: Thành phố K¨onigsberg thuộc Phổ
(bây giờ gọi là Kaliningrad thuộc Cộng hòa Liên bang Nga) được chia
thành bốn vùng bằng các nhánh sông Pregel. Các vùng này gồm 2 vùng
bên bờ sông, đảo Kneiphof và một miền nằm giữa 2 nhánh của sông
Pregel. Vào thế kỷ thứ XVIII, người ta đã xây 7 cây cầu nối các vùng
lại với nhau như sơ đồ sau:
Hình 2.1
Vào chủ nhật, người dân ở đây thường đi bộ dọc theo các vùng trong
thành phố. Họ tự hỏi “Liệu có thể xuất phát tại một địa điểm nào đó
trong thành phố, đi qua tất cả 7 cây cầu, qua mỗi cây một lần, rồi trở
33
về điểm xuất phát được không?”
Nhà toán học Thụy Sĩ Leonard Euler đã nghiên cứu giải bài toán này.
Lời giải của ông được công bố năm 1736. Bài toán này có thể được coi
là một trong những ứng dụng đầu tiên của lý thuyết đồ thị.
Ta có thể xây dựng đồ thị G = (V, E) mô tả bài toán như sau:
+ Đỉnh: Lấy các điểm trên mặt phẳng hay trong không gian tương
ứng với các vùng đất trong sơ đồ. Đối tượng của bài toán ở đây là một
vùng đất trong sơ đồ. Vậy, mỗi đỉnh biểu diễn cho một vùng đất. Đồ thị
G sẽ có 4 đỉnh A, B, C, D tương ứng với 4 vùng đất.
+ Cạnh: Trong đồ thị G các đỉnh được nối với nhau bằng các cạnh
mà mỗi cạnh đại diện cho một chiếc cầu nối giữa hai vùng đất. Đồ thị
G sẽ có 7 cạnh tương ứng với 7 chiếc cầu nối giữa các vùng đất trong sơ
đồ.
Euler đã nghiên cứu bài toán này, mô hình nó bằng một đa đồ thị,
bốn vùng được biểu diễn bằng 4 đỉnh, các cầu là các cạnh như đồ thị
sau:
Hình 2.2
Bài toán tìm đường đi qua tất cả các cầu mỗi cầu không quá một lần
có thể được phát biểu lại bằng ngôn ngữ đồ thị như sau: “Trong đa đồ
thị G = (V, E) tồn tại hay không một chu trình đơn có chứa tất cả 7
cạnh?”
2.1.1.2. Định nghĩa
1. Chu trình Euler (Đồ thị Euler)
Cho G = (V, E) là một đa đồ thị liên thông. Chu trình đơn chứa tất
cả các cạnh của đồ thị G được gọi là chu trình Euler. Đồ thị có chứa
một chu trình Euler được gọi là đồ thị Euler.
34
2. Đường đi Euler
Cho G = (V, E) là một đa đồ thị liên thông. Đường đi Euler trong G
là đường đi đơn chứa tất cả các cạnh của đồ thị G.
Hình 2.3
Ví dụ 2.1. .
Đồ thị ở hình 2.3a có chu trình Euler: a, b, e, d, c, e, a.
Đồ thị ở hình 2.3b không có chu trình Euler nhưng có đường đi Euler:
a, c, d, a, b, e, d, b.
Đồ thị ở hình 2.3c không có chu trình Euler và đường đi Euler.
2.1.1.3. Chu trình và đường đi Euler trong đồ thị vô hướng
Khi giải bài toán cầu K¨onigsberg, Euler đã phát hiện ra các tiêu chuẩn
để khẳng định một đa đồ thị có chu trình và đường đi Euler hay không?
1. Định lý về chu trình Euler
Một đa đồ thị liên thông G =(V, E) có chu trình Euler khi và chỉ khi
mỗi đỉnh của nó đều có bậc chẵn.
Chứng minh.
(⇒) Ta sẽ chứng minh: Nếu đồ thị G có chu trình Euler, thì mọi đỉnh
của G đều có bậc chẵn. Thật vậy, trước tiên giả sử G có chu trình Euler
bắt đầu từ đỉnh a và tiếp tục bằng cạnh liên thuộc với a, chẳng hạn ab,
khi đó cạnh ab góp một đơn vị vào deg (a). Mỗi lần khi chu trình đi qua
mỗi đỉnh, nó tăng thêm 2 đơn vị cho bậc của đỉnh đó. Vì chu trình đi
vào mỗi đỉnh bằng một cạnh liên thuộc và rời khỏi đỉnh này bằng một
cạnh liên thuộc khác. Cuối cùng chu trình kết thúc ở đỉnh mà nó xuất
35
phát. Do đó nó tăng thêm một đơn vị vào deg (a). Nghĩa là deg (a) phải
là một số chẵn. Đỉnh khác a cũng có bậc chẵn vì chu trình góp 2 đơn vị
vào bậc của nó mỗi lần đi qua đỉnh này. Vậy, mỗi đỉnh của G đều có bậc
chẵn.
(⇐) Giả sử mọi đỉnh của đa đồ thị liên thông G đều có bậc chẵn. Ta
sẽ chứng minh tồn tại một chu trình Euler trong G.
Thật vậy, ta sẽ xây dựng một chu trình đơn bắt đầu từ đỉnh a tùy ý
của G. Gọi x0 = a; Trước tiên, ta chọn tùy ý cạnh x0x1, x1x2, ..., xn−1xn
càng dài càng tốt. Ví dụ, trong đồ thị G sau:
Hình 2.4
Ta bắt đầu tại a và chọn các cạnh liên tiếp ab, bc, cf, fa. Đường đi
mà ta chọn sẽ kết thúc vì đồ thị có hữu hạn đỉnh. Đường đi này bắt đầu
tại a và kết thúc tại a.
Điều này luôn xảy ra vì mỗi lần đường đi qua một đỉnh bậc chẵn,
nó chỉ dùng một cạnh để vào đỉnh này và còn ít nhất một đỉnh để ra
khỏi đỉnh này. Đường đi vừa nói trên có thể đi qua tất cả các cạnh hoặc
có thể không. Nếu tất cả các cạnh được sử dụng, thì ta nhận được chu
trình Euler. Nếu không, ta gọi H là đồ thị con nhận được từ G bằng
cách xóa các cạnh đã dùng và các đỉnh không liên thuộc với các cạnh
còn lại. Chẳng hạn với đồ thị trên, khi xóa đi chu trình a, b, c, f, a khỏi
đồ thị này, ta nhận được đồ thị con H.
Vì G là liên thông nên H có ít nhất một đỉnh chung với chu trình vừa
bị xóa. Gọi w là đỉnh đó (trong ví dụ trên là đỉnh c). Mỗi đỉnh của H
có bậc chẵn vì tất cả các đỉnh của G có bậc chẵn và với mỗi đỉnh ta đã
xóa đi từng cặp liên thuộc để tạo ra H. Lưu ý rằng H có thể không liên
thông. Bắt đầu từ đỉnh w ta xây dựng một đường đi đơn bằng cách chọn
càng nhiều đỉnh càng tốt như ta đã làm trong G. Đường này phải kết
thúc tại w. Ví dụ trong đồ thị H nêu trên ta có chu trình con: c, d, e, c.
Sau đó, ta tạo một chu trình trong G bằng cách ghép chu trình trong H
36
với chu trình ban đầu trong G (điều này thực hiện được vì 2 chu trình
có chung đỉnh w). Tiếp tục quá trình này cho đến khi tất cả các đỉnh
được sử dụng. Quá trình này phải kết thúc vì đồ thị có hữu hạn đỉnh.
Do đó, ta đã xây dựng được một chu trình Euler.
Định lý được chứng minh.
Bây giờ, trở lại bài toán 7 cây cầu ở K¨onigsberg: Ta đã thấy đa đồ
thị G = (V, E) biểu diễn các cầu ở K¨onigsberg có 4 đỉnh bậc lẻ. Do đó,
theo định lý trên sẽ không có chu trình Euler trong đa đồ thị này. Điều
này cũng có nghĩa là bài toán 7 cây cầu ở K¨onigsberg không có lời giải.
Hay nói cách khác, không thể từ một địa điểm nào đó đi qua cả 7 cây
cầu, qua mỗi cầu đúng một lần rồi trở về được nơi xuất phát.
2. Thuật toán Fleury tìm chu trình Euler
Để tìm một chu trình Euler trong một đa đồ thị có tất cả các đỉnh
đều bậc chẵn, ta có thể sử dụng thuật toán Fleury như sau:
Xuất phát từ một đỉnh bất kỳ của đồ thị G và tuân theo hai qui tắc
sau:
• Qui tắc 1: Mỗi khi đi qua một cạnh nào thì xóa cạnh đó đi, sau đó
xóa đỉnh cô lập (nếu có).
• Qui tắc 2: Không bao giờ đi qua một cầu (cạnh duy nhất nối giữa
hai thành phần liên thông), trừ khi không còn cách đi nào khác để di
chuyển.
Ví dụ 2.2. Tìm một chu trình Euler trong đồ thị G (Hình 2.5)
Hình 2.5
Xuất phát từ đỉnh A, giả sử ta chọn cạnh AB, BC, CF. Sau đó xóa 3
cạnh này, ta được đồ thị G1
37
Đến đây, ta không thể chọn FG vì GF là một cầu, cho nên ta chọn
FD, DC, CE, EF. Đến đây, ta được đồ thị G2.
Ta không còn cách chọn nào khác, nên phải chọn FG, GH, HB, BG,
GA.
Như vậy, ta có chu trình Euler sau: A, B, C, F, D, C, E, F, G, H,
B, G, A.
3. Định lý về đường đi Euler
Đa đồ thị liên thông G = (V, E) có đường đi Euler, nhưng không có
chu trình Euler khi và chỉ khi nó có đúng hai đỉnh bậc lẻ.
Chứng minh.
(⇒) Giả sử đa đồ thị G có đường đi Euler, nhưng không có chu trình
Euler. Ta sẽ chứng minh G có đúng 2 đỉnh bậc lẻ.
Thật vậy, giả sử G có đường đi Euler từ a đến b, nhưng không có chu
trình Euler. Cạnh đầu tiên của đường đi góp một đơn vị vào deg (a).
Sau đó mỗi lần đường đi qua a lại góp thêm 2 đơn vị vào deg (a).
Chắc chắn đường đi không thể kết thúc tại a, cho nên deg(a) là số lẻ.
Cạnh cuối cùng của đường đi góp một đơn vị vào deg(b) và mỗi lần đi
qua b, nó cũng góp 2 đơn vị vào deg(b). Do đó, deg(b) là số lẻ. Các đỉnh
trung gian đều có bậc chẵn vì mỗi lần đường đi tới rồi lại đi nên tăng
hai đơn vị cho bậc của đỉnh đó. Vậy, đồ thị đã cho có đúng 2 đỉnh bậc
lẻ.
(⇐) Giả sử đa đồ thị liên thông G có đúng 2 đỉnh bậc lẻ. Ta sẽ chứng
minh G có đường đi Euler.
Thật vậy, giả sử G có đúng 2 đỉnh bậc lẻ là a và b. Thêm vào cạnh
nối giữa a và b ta được đồ thị G’.Khi đó trong đồ thị mới G = G ∪ ab
, tất cả các đỉnh đều có bậc chẵn. Do đó, theo định lý Euler, tồn tại
một chu trình Euler trong G’. Trong chu trình này bỏ cạnh ab, ta được
đường đi Euler trong G.
Như vậy, trong một đa đồ thị liên thông có 2 đỉnh bậc lẻ thì đường
đi Euler trong đồ thị đó sẽ nhận 2 đỉnh bậc lẻ làm các điểm đầu mút.
Định lý được chứng minh.
38
2.1.1.4. Chu trình và đường đi Euler đối với đồ thị có hướng
1. Định lý về chu trình Euler
Đồ thị G = (V, E) có chứa một chu trình Euler khi và chỉ khi G là
liên thông và mọi đỉnh của nó đều có bậc chẵn.
Ví dụ 2.3. Trên hình 2.6 ta có:
Hình 2.6
Đồ thị G1 có chu trình Euler: a, b, c, a, d, c, a.
Đồ thị G2 không có chu trình Euler.
2. Định lý về đường đi Euler
Cho G = (V, E) là một đa đồ thị. G có một đường đi Euler từ A đến
B khi và chỉ khi G là liên thông và mọi đỉnh của nó đều có bậc chẵn, chỉ
trừ A và B có bậc lẻ.
Ví dụ 2.4. Trên hình 2.7
Hình 2.7
Đồ thị G có đường đi Euler: a, b, c, a, d, c.
39
2.1.2 Đường đi Hamilton - Chu trình Hamilton.
2.1.2.1. Trò chơi Hamilton.
Năm 1857 W. R. Hamilton đưa ra trò chơi sau đây:
Trên mỗi đỉnh trong số 20 đỉnh của khối đa diện ngũ giác đều 12 mặt
ghi tên một thành phố trên thế giới Hãy tìm cách đi bằng các cạnh của
khối đa diện để qua tất cả các thành phố, mỗi thành phố đúng một lần.
Hình 2.8
Để có được đáp án cho trò chơi như hình 2.8 ta cần nghiên cứu lý
thuyết về chu trình Hamilton.
2.1.2.2. Định nghĩa.
Đường đi trong đồ thị vô hướng G = (V, E) được gọi là đường đi
Hamilton nếu nó đi qua tất cả các đỉnh của G và qua mỗi đỉnh đúng
một lần.
Một chu trình sơ cấp đi qua tất cả các đỉnh của đồ thị G = (V, E)
(đi qua mỗi đỉnh đúng một lần) được gọi là chu trình Hamilton. Đồ thị
G = (V, E) có chứa chu trình Hamilton được gọi là đồ thị Hamilton.
Ví dụ 2.5. Cho hai đồ thị như trên hình 2.9
Đồ thị hình 2.9a có chu trình Hamilton là: a, b, c, d, e, a.
Đồ thị hình 2.9b không có chu trình Hamilton vì mọi chu trình chứa
mọi đỉnh của đồ thị đều phải đi qua cạnh ab 2 lần.
40
Hình 2.9
2.1.2.3. Điều kiện tồn tại chu trình Hamilton.
1. Bổ đề 2.1.2.1.
Đồ thị vô hướng n đỉnh liên thông (n ≥ 3), thuần nhất bậc 2 có chu
trình Hamilton.
Chứng minh. Giả sử đồ thị vô hướng G = (X, E)(|X| = n ≥ 3), liên
thông thuần nhất bậc 2.
Đồ thị G hữu hạn, nên số xích sơ cấp trong G cũng hữu hạn.
Giả sử α = (x1, x2, ..., xk−1, xk) là một trong những xích sơ cấp có độ
dài dài nhất (|α|max)
Nếu k < n thì tồn tại đỉnh y /∈ α. Do G liên thông nên phải có ít
nhất một đỉnh thuộc α kề với đỉnh y, chẳng hạn xi.
Nếu i = 2, 3, ..., k − 1, thì m(xi) > 2 nên vô lý.
Nếu i = 1 hoặc k, thì xích (y, x2, ..., xk−1, xk) hoặc xích (x1, x2, ..., xk−1, xk, y)
có độ dài bằng |α| + 1 > |α|max, nên cũng dẫn đến mâu thuẫn.
Do đó k = n, tức ta được xích Hamilton.
Vì m(x1) = m(xn) = 2 nên x1 phải kề với xn (không thể kề với
xi(3 ≤ i ≤ n−1) vì bậc của mỗi đỉnh này đã bằng 2). Ta được chu trình
Hamilton.
2. Bổ đề 2.1.2.2.
Đồ thị vô hướng G = (X, E) có chu trình Hamilton khi và chỉ khi nó
có một đồ thị bộ phận liên thông và thuần nhất bậc 2.
Chứng minh. .
1) Điều kiện cần là hiển nhiên. Bởi vì, nếu đồ thị G có chu trình
Hamilton, thì mỗi chu trình Hamilton của G là một đồ thị bộ phận liên
thông thuần nhất bậc 2.
41
2) Điều kiện đủ.
Giả sử đồ thị G = (X, E) có đồ thị bộ phận G1 = (X, E1) liên thông
và thuần nhất bậc 2. Theo bổ đề 2.1.2.1 đồ thị G1 có chu trình Hamilton
α. Khi đó, α cũng chính là chu trình Hamilton trong đồ thị G
3. Định lý Rédei
Trong đồ thị có hướng đầy đủ luôn luôn tồn tại đường Hamilton.
Chứng minh. (Thuật toán tìm đường Hamilton trong đồ thị đầy đủ có
hướng.)
Giả sử G = (X, E) là đồ thị có hướng đầy đủ nào đó và:
α = [x1, x2, ..., xi, xi+1, ...xk−1, xk]
là một đường sơ cấp bất kì trong đồ thị G.
1) Nếu α đã đi qua tất cả các đỉnh của G, thì nó là một đường
Hamilton của G.
2) Nếu trong G còn có đỉnh nằm ngoài α, thì ta có thể bổ sung dần
dần các đỉnh này vào α và các đường sơ cấp nhận được tiếp theo để cuối
cùng nhận được đường Hamilton.
Thật vậy, giả sử x là đỉnh tùy ý không nằm trên α.
a) Nếu cung nối x với x1 xuất phát từ x, thì bổ sung x và đầu của
đường α, để được α1, mà |α1| = |α| + 1
α1 = [x, x1, x2, ..., xk−1, xk]
b) Nếu tồn tại chỉ số i(1 ≤ i ≤ k − 1) mà từ xi có cung đi tới x và từ
x có cung đi tới xi+1 thì ta chen x vào giữa xi và xi+1 để được đường sơ
cấp α2 mà |α2| = |α| + 1,
α2 = [x1, x2, ..., xi, x, xi+1, ..., xk−1, xk]
c) Nếu cả hai khả năng trên đều không xảy ra nghĩa là ∀i(1 ≤ i ≤ k)xi
đều có cung đi tới x. Khi đó bổ sung x vào cuối của đường α và được α3
mà |α3| = |α| + 1,
α3 = [x1, x2, ..., xk−1, xk, x]
Nếu đồ thị G có n đỉnh thì sau n − k bổ sung ta sẽ được đường
Hamilton.
42
2.2 Bài toán tô màu đồ thị
2.2.1 Định nghĩa
Tô màu đỉnh của một đồ thị là phép gán các màu cho các đỉnh, sao
cho hai đỉnh kề nhau bất kỳ có màu khác nhau.
Sắc số của đồ thị là số màu ít nhất cần dùng để tô trên các đỉnh của
đồ thị, sao cho hai đỉnh kề nhau tùy ý được tô bằng hai màu khác nhau.
Ví dụ 2.6. ([4]Lý thuyết đồ thị và các bài toán không mẫu mực)
Đồ thị trên hình 2.10 có sắc số bằng 3:
Hình 2.10
Sắc lớp là số màu ít nhất cần dùng để tô trên các cạnh của đồ thị,
sao cho hai cạnh kề nhau (có đỉnh chung) tùy ý đều có màu khác nhau.
2.2.2 Một số tính chất
Định lí 2.2.1. Một chu trình độ dài lẻ luôn có sắc số bằng 3.
Chứng minh.
Giả sử α là một chu trình độ dài lẻ tùy ý. Khi đó tồn tại số tự nhiên
n để |α| = 2n + 1.
Ký hiệu các đỉnh của α một cách trực tiếp bằng x1, x2, ..., x2n, x2n+1.
Ta sẽ chứng minh khẳng định trên bằng phương pháp quy nạp theo
n.
1) Với n = 1, chu trình α gồm 3 đỉnh x1, x1, x3.
Do mỗi đỉnh xi, (1 ≤ i ≤ 3) đều kề với hai đỉnh còn lại, nên ta phải
dùng đúng 3 màu khác nhau thì mới đủ tô trên mỗi đỉnh một màu, để
hai đỉnh kề nhau tùy ý đều có màu khác nhau.
2) Giả sử khẳng định đã đúng với n ≤ k, tức là đối với chu trình
α1 tùy ý với độ dài 2n + 1, (1 ≤ n ≤ k) đều có sắc số bằng 3. Cần chỉ
ra rằng với n = k + 1 khẳng định vẫn đúng, tức là chu trình α tùy ý
43
có độ dài bằng 2(k + 1) + 1 và có tập đỉnh được đánh số liên tiếp là
x1, x2, ..., xk, xk+1, ..., x2k+2, x2k+3.
Hình 2.11
Nối đỉnh x1 với đỉnh x2k+1 ta được chu trình α1 với độ dài 2k + 1.
Theo giả thiết quy nạp, sắc số của α1 bằng 3, đồng thời x1 và x2k+1 có
màu khác nhau. Chẳng hạn, x1 được tô màu M1 và x2k+1 được tô bằng
màu M2. Khi đó để tô đỉnh x2k+2 ta có thể dùng lại màu M1 và tô đỉnh
x2k+3 màu M2. Tức là không phải thêm màu mới. Vậy sắc số của α bằng
3.
Khẳng định được chứng minh.
Lớp đồ thị có chu trình tam giác cùng màu
Để phục vụ cho việc giải quyết một số bài toán nào đó ta cần xét
những dãy số đặc biệt và đưa ra các khẳng định thích hợp, chẳng hạn,
để xây dựng những lớp đồ thị có chu trình tam giác cùng màu người ta
đưa ra các dãy số nguyên dương:
a1 = 2, a2 = 5, ..., an+1 = (n + 1)an + 1
u2 = 3, u3 = 6, ..., un+1 = (un − 1)n + 2
và có định lý sau:
Định lí 2.2.2. a. Một đồ thị đầy đủ và vô hướng với an + 1 đỉnh, các
cạnh được tô bằng n màu luôn có chu trình tam giác cùng màu.
b. Một đồ thị đầy đủ vô hướng un+1 đỉnh, các cạnh được tô bằng n
màu luôn có chu trình tam giác cùng màu.
Chứng minh.
44
a. Chứng minh bằng quy nạp theo n
1) Với n = 1 đồ thị đầy đủ tương ứng gồm a1 +1 = 2+1 = 3 đỉnh lập
thành một chu trình tam giác. Các cạnh của đồ thị này được tô bằng
một màu, nên chu trình tam giác lập nên G1 cùng màu.
2) Giả sử khẳng định đã đúng với n = k, tức là đồ thị đầy đủ bất kỳ
Gk gồm ak + 1 đỉnh với các cạnh được tô bằng k màu đã có chu trình
tam giác cùng màu. Cần chứng minh rằng khẳng định cũng đúng với
n = k + 1.
Xét đồ thị đầy đủ tùy ý Gk+1 với ak+1 + 1 đỉnh và các cạnh được tô
bằng k + 1 màu, nên xuất phát từ đỉnh P phải có ít nhất ak + 1 cạnh
được tô bằng cùng một màu. Giả sử màu này là màu đỏ và các cạnh
PA1, PA2, ..., PAak
, PAak+1 được tô bằng màu đỏ. Có hai khả năng xảy
ra:
i) Nếu một trong các cạnh nối giữa các đỉnh Ai, Aj, (1 ≤ i, j ≤ ak +1)
được tô màu đỏ, chẳng hạn cạnh A1, A2 màu đỏ. Khi đó chu trình tam
giác A1PA2 màu đỏ, nên đồ thị Gk+1 có chu trình tam giác màu đỏ.
ii) Ngược lại, nếu không có cạnh nào trong các cạnh Ai, Aj, (1 ≤
i, j ≤ ak + 1) được tô màu đỏ. Khi đó đồ thị con đầy đủ Gk với tập
đỉnh A1, A2, ..., Aak
, Aak+1 có các cạnh được tô bằng không quá k màu,
nên theo giả thiết quy nạp Gk đã có chu trình tam giác cùng màu, do
đó Gk+1 có chu trình tam giác cùng màu.
Khẳng định được chứng minh.
b. Chứng minh tương tự như phần a.
Định lí 2.2.3. Đồ thị đầy đủ có un+1 − 1 đỉnh (n ≥ 2) với n màu cạnh
(các cạnh được tô bằng n màu), sao cho không có tam giác cùng màu
nào, luôn luôn có 5 hình cạnh với các cạnh cùng màu và các đường chéo
được tô bằng màu khác.
Chứng minh. 1. Với n = 2 đồ thị tương ứng G2 đầy đủ có u3 − 1 = 5
đỉnh và 2 màu cạnh (xanh, đỏ) không có đồ thị con K3 cùng màu. Khi
đó G2 có thể biểu diễn ở dạng hình 5 cạnh với cạnh cùng màu đỏ và
đường chéo màu xanh.
Thật vậy, do G2 đầy đủ nên mỗi đỉnh xuất phát đúng 4 cạnh được tô
bằng 2 màu. Chính xác hơn, tại từng đỉnh, mỗi màu được tô trên đúng
2 cạnh. Giả sử ngược lại, tại đỉnh A màu đỏ được tô trên 3 cạnh là AB,
AC và AD. Khi đó một trong 3 cạnh BC, BD, CD màu đỏ, đồ thị có tam
giác đỏ. Ngược lại cả 3 cạnh đều màu xanh, đồ thị có tam giác xanh.
Như vậy mâu thuẫn với giả thiết.
45
Giả sử A có các cạnh đỏ là AB và AC (đường liền nét), còn AD và
AE là xanh (đường nét đứt). Khi đó cạnh BC phải xanh và ED là đỏ
(hình 2.12a).
Hai cạnh BE và CE không thể cùng xanh. Giả sử BE đỏ, thì CE xanh.
Suy ra CD đỏ và BD xanh. Vậy ta được hình 5 cạnh với cạnh màu đỏ
và đường chéo màu xanh (hình 2.12b).
Hình 2.12
2. Giả sử khẳng định đúng với n = k. Xét đồ thị Gk+1 đầy đủ với
u(k+1)+1 − 1 đỉnh, k + 1 màu cạnh và không có đồ thị con K3 cùng màu.
Mỗi đỉnh của Gk+1 xuất phát (u(k+1)+1 − 1)(k + 1) cạnh với k + 1
màu, nên phải có ít nhất uk+1 − 1 cạnh cùng màu. Giả sử tại đỉnh A có
uk+1 − 1 cạnh cùng được tô bởi màu m1. Khi đó trong các đỉnh đối của
A không có cặp đỉnh nào được nối với nhau bởi cạnh màu m1 (trái lại
thì có K3 cùng màu m1). Xét đồ thị con đầy đủ Gk lập nên từ uk+1 − 1
đỉnh đối với A có cạnh chỉ tô bởi k màu (trừ màu m1) và không có K3
cùng màu, nên theo giả thiết quy nạp, trong Gk có hình 5 cạnh với cạnh
cùng một màu và đường chéo là các màu khác (tất cả đều không là màu
m1). Vậy trong Gk+1 có điều cần khẳng định.
Định lí 2.2.4. Đồ thị đầy đủ gồm 6 đỉnh và được tô bằng không quá hai
màu cạnh thì luôn có 2 chu trình tam giác cùng màu.
Chứng minh. Trước hết ta chứng minh tồn tại tam giác cùng màu thứ
nhất.
Gọi các đỉnh của đồ thị là A1, A2, A3, A4, A5, A6. Giả sử các cạnh của
đồ thị được tô bởi màu xanh hoặc đỏ. Xét 5 cạnh xuất phát từ đỉnh A1.
Theo nghuyên lý Dirichlet tồn tại ít nhất 3 cạnh cùng màu, không giảm
tính tổng quát nếu ta giả sử các cạnh đó là A1A2, A1A3, A1A4 và có màu
đỏ (nét liền).
46
Hình 2.13a
Nếu tam giác A2A3A4 có các cạnh đều là màu xanh suy ra tồn tại
tam giác cùng màu.
Nếu tam giác A2A3A4 có ít nhất một cạnh màu đỏ, giả sử cạnh đó là
A2A3 suy ra tam giác A1A2A3 là tam giác cùng màu.
Chứng minh sự tồn tại của tam giác cùng màu thứ hai.
Giả sử tam giác cùng màu thứ nhất là A1A2A3 và có màu đỏ. Xét các
khả năng sau:
• Trường hợp 1: Các cạnh A1A4, A1A5, A1A6 đều có màu đỏ. Xét tam
giác A4A5A6:
a. Nếu tam giác A4A5A6 có cả ba cạnh đều màu xanh thì ta có tam
giác cùng màu thứ hai.
b. Nếu tam giác A4A5A6 có ít nhất một cạnh màu đỏ, chẳng hạn
A4A5 thì ta có tam giác cùng màu thứ hai là A1A4A5.
• Trường hợp 2: Trong 3 cạnh A1A4, A1A5, A1A6 có đúng hai cạnh
màu đỏ và một cạnh màu xanh. Không giảm tổng quát ta giả sử hai
cạnh đỏ là A1A4, A1A5 và cạnh có màu xanh là A1A6. Xét tam giác
A3A4A5:
a. Nếu tam giác có cả 3 cạnh đều màu xanh thì ta có tam giác cùng
màu thứ hai.
b. Nếu tam giác này có ít nhất một cạnh đỏ, chẳng hạn A4A5 thì ta
có tam giác cùng màu thứ hai là A1A4A5.
• Trường hợp 3: Trong 3 cạnh A1A4, A1A5, A1A6 có đúng một cạnh
màu đỏ và hai cạnh màu xanh. Giả sử cạnh màu đỏ là A1A4 và hai cạnh
màu xanh là A1A5, A1A6.
a. Nếu A2A4 hoặc A3A4 có màu đỏ hoặc A5A6 có màu xanh thì ta có
tam giác cùng màu thứ hai.
b. Nếu A2A4, A3A4 màu xanh còn A5A6 màu đỏ. Ta xét tam giác
47
Hình 2.13b
Hình 2.13c
A4A5A6 :
-) Nếu A4A5, A4A6 đều có màu đỏ thì ta có tam giác cùng màu thứ
hai là A4A5A6.
-) Nếu trong hai cạnh A4A5, A4A6 có ít nhất một cạnh có màu xanh.
Giả sử A4A5 có màu xanh. Xét tam giác A2A3A5:
+, Nếu tam giác này có A3A5, A2A5 cùng màu đỏ thì ta có tam giác
cùng màu thứ hai.
+) Nếu A3A5 có màu xanh thì ta cũng có tam giác cùng màu thứ hai
là A3A4A5.
+) Nếu A2A5 có màu xanh thì ta cũng có tam giác cùng màu thứ hai
là A2A4A5.
• Trường hợp 4: Cả 3 cạnh A1A4, A1A5, A1A6 cùng có màu xanh.
Xét tam giác A4A5A6:
-) Nếu cả ba cạnh của tam giác đều có màu đỏ thì ta có tam giác
cùng màu thứ hai.
48
Hình 2.13d
-) Nếu trong 3 cạnh của tam giác có ít nhất một cạnh có màu xanh,
giả sử cạnh đó là A5A6 thì ta có tam giác cùng màu thứ 2 là A1A5A6
Định lí 2.2.5. Đồ thị đầy đủ G gồm n đỉnh (n ≥ 6) và được tô bằng
không quá hai màu cạnh luôn có ít nhất (n − 4) tam giác cùng màu.
Chứng minh. Chứng minh bằng phương pháp quy nạp.
Khẳng định đúng với n=6.
Giả sử định lý đúng với n = k, k ≥ 6. Nghĩa là đồ thị đầy đủ Gk gồm
có k đỉnh được tô bởi không quá hai màu cạnh đã có ít nhất (k − 4)
tam giác cùng màu. Cần chứng minh đồ thị đầy đủ Gk+1 có k + 1 đỉnh
và được tô bởi không quá hai màu cạnh thì có ít nhất (k − 3) tam giác
cùng màu.
Gọi các đỉnh của đồ thị Gk+1 là A1, A2, A3, ..., Ak+1. Vì Gk+1 có đồ
thị con Gk nên luôn có tam giác cùng màu. Không giảm tổng quát ta
gọi tam giác cùng màu đó là A1A2A3. Loại đỉnh A1 và các cạnh xuất
phát từ nó ra khỏi đồ thị Gk+1 thì ta có đồ thị đầy đủ Gk có ít nhất
(k − 4) tam giác cùng màu. Khôi phục lại đỉnh A1 và các cạnh của nó
ta có thêm ít nhất một tam giác cùng màu nữa là A1A2A3. Vậy đồ thị
đầy đủ Gk+1 có ít nhất k − 4 + 1 = k − 3 tam giác cùng màu.
Định lí 2.2.6. Đồ thị đầy đủ G gồm 9 đỉnh được tô bằng hai màu cạnh
xanh, đỏ thì luôn có đồ thị con đầy đủ K3 xanh hoặc đồ thị con đầy đủ
K4 đỏ (hoặc ngược lại nếu ta đổi hai màu cho nhau)
Chứng minh. Gọi các đỉnh của đồ thị là x1, x2, ..., x9.
• Trường hợp 1: Đồ thị có đỉnh là đầu mút của ít nhất 4 cạnh màu
xanh (nét đứt đoạn). Giả sử đỉnh x1 là đầu mút của ít nhất 4 cạnh màu
xanh và các cạnh đó là x1x2, x1x3, x1x4, x1x5. Xét các cạnh nối hai đỉnh
49
trong số các đỉnh x2, x3, x4, x5. Nếu có ít nhất một cạnh có màu xanh
thì ta có tam giác xanh. Nếu không có cạnh nào có màu xanh, thì ta có
tứ giác đỏ (hình 2.14a).
Hình 2.14a
• Trường hợp 2: Không có đỉnh nào là đầu mút của nhiều hơn 3 cạnh
màu xanh. Khi đó có ít nhất một đỉnh là đầu mút của ít hơn 3 cạnh
màu xanh.
Thật vậy, nếu tất cả các đỉnh là đầu mút của đúng 3 cạnh màu xanh,
thì tổng số cạnh có màu xanh là
3.9
2
. Điều này vô lý vì số cạnh phải
nguyên.
Hình 2.14b
Giả sử x1 là mút của không quá 2 cạnh xanh suy ra x1 là mút của ít
nhất 6 cạnh đỏ.
50
Gọi các cạnh màu đỏ là x1x2, x1x3, x1x4, x1x5, x1x6, x1x7 (hình 2.14b).
Xét đồ thị con đầy đủ G1 có 6 đỉnh x2, x3, x4, x5, x6, x7 với hai màu cạnh.
Theo định lý 2.2.4 thì G1 có tam giác cùng màu.
Nếu tam giác này có màu xanh thì định lý được chứng minh. Nếu
tam giác này có màu đỏ thì tam giác này cùng với đỉnh x1 tạo thành tứ
giác cùng màu đỏ.
Định lí 2.2.7. Đồ thị đầy đủ K14 gồm 14 đỉnh được tô bằng hai màu
cạnh xanh, đỏ, thì luôn có tam giác xanh hoặc ngũ giác đỏ (hoặc ngược
lại nếu ta đổi hai màu cho nhau.)
Chứng minh. Gọi các đỉnh của đồ thị là x1, x2, ..., x14.
• Trường hợp 1: Đồ thị có đỉnh là đầu mút của ít nhất 5 cạnh màu
xanh (nét đứt đoạn). Giả sử đỉnh x1 là đầu mút của ít nhất 5 cạnh màu
xanh và các cạnh đó là x1x2, x1x3, x1x4, x1x5, x1x6.
Nếu trong các cạnh có đầu mút là hai trong 5 điểm x2, x3, x4, x5, x6
có cạnh màu xanh. Chẳng hạn x2x5 thì ta có tam giác xanh x1x2x5. Nếu
không ta có ngũ giác đỏ x2x3x4x5x6.
Hình 2.15
• Trường hợp 2: Không có đỉnh nào là đầu mút của nhiều hơn 4 cạnh
màu xanh. Suy ra đỉnh x1 là đầu mút của ít nhất 9 cạnh màu đỏ. Giả
sử các cạnh đó là x1x2, x1x3, x1x4, x1x5, x1x6, x1x7, x1x8, x1x9, x1x10. Xét
đồ thị con đầy đủ G1 gồm các đỉnh x2, x3, x4, x5, x6, x7, x8, x9, x10. Theo
định lý 2.2.6 thì G1 có tam giác xanh hoặc tứ giác đỏ. Nếu tồn tại tam
giác xanh ta có điều phải chứng minh. Nếu tồn tại tứ giác đỏ, giả sử tứ
giác đó là x2x3x4x5 thì ta có ngũ giác đỏ x1x2x3x4x5.
51
Nếu tam giác này có màu xanh thì định lý được chứng minh. Nếu
tam giác này có màu đỏ thì tam giác này cùng với đỉnh x1 tạo thành tứ
giác cùng màu đỏ.
Định lí 2.2.8. Đồ thị 3 mảng G(X, E) với lực lượng mỗi mảng đều bằng
n(n ≥ 1). Mỗi đỉnh được nối với từng đỉnh thuộc hai mảng còn lại bằng
các cạnh tô màu xanh hoặc màu đỏ, sao cho số cạnh đỏ xuất phát từ mỗi
đỉnh bằng đúng (n+1). Khi đó đồ thị G(X, E) có chu trình tam giác đỏ.
Chứng minh. Trước hết chọn một đỉnh của G(X, E) có số cạnh đỏ nối
với một trong hai mảng đỉnh còn lại lớn nhất. Giả sử đỉnh chọn được
là A thuộc mảng thứ nhất và mảng có nhiều đỉnh nhất nối với A bằng
một cạnh đỏ là mảng thứ hai gồm k đỉnh: B1, B2, ..., Bk . Khi đó số đỉnh
thuộc mảng thứ 3 nối với A bằng một cạnh có màu đỏ là: n+k −1 đỉnh.
Hình 2.16
Xét đỉnh C thuộc mảng thứ 3 và nối với đỉnh A bằng một cạnh có màu
đỏ. Đỉnh C phải nối được với ít nhất một trong các đỉnh B1, B2, ..., Bk
bằng một cạnh màu đỏ. Thật vậy, nếu ngược lại thì số đỉnh thuộc mảng
thứ hai nối với C bằng một cạnh màu đỏ ít hơn n−k, suy ra số đỉnh thuộc
mảng thứ nhất nối với C bằng một cạnh màu đỏ lớn hơn n+1−(n−k) =
52
k + 1. Điều này mâu thuẫn với việc chọn điểm A, vậy tồn tại đỉnh Bi,
sao cho CBi có màu đỏ. Suy ra ta có tam giác đỏ ABiC.
2.2.3 Thuật toán tô màu đỉnh.
1, Lập danh sách các đỉnh đồ thị theo thứ tự bậc giảm dần.
Đặt i := 1
2, Tô màu i cho đỉnh đầu tiên trong danh sách. Duyệt lần lượt các
đỉnh tiếp theo và tô màu i cho đỉnh không kề đỉnh đã được tô màu i.
3, Nếu tất cả các đỉnh đã được tô màu thì kết thúc: Đồ thị đã được
tô bằng i màu. Ngược lại sang bước 4.
4, Loại khỏi tập đỉnh các đỉnh đã tô màu, đặt i := i + 1, và quay lại
bước 2.
53
Chương 3
Ứng dụng lý thuyết đồ thị vào giải
toán phổ thông.
3.1 Quy trình giải bài toán bằng phương pháp đồ
thị.
Để giải bài toán T bằng phương pháp đồ thị ta cần thực hiện lần lượt
hai bước sau:
3.1.1 Xây dựng đồ thị G mô tả các quan hệ.
• Đỉnh: Lấy các điểm trên mặt phẳng hoặc trong không gian tương
ứng với các đối tượng đã cho trong bài toán. Dùng ngay các ký hiệu đối
tượng để ghi trên điểm tương ứng.
• Cạnh: Hai đỉnh x, y tùy ý được nối với nhau bằng một cạnh (cung)
với "đặc điểm t", khi và chỉ khi các đối tượng x, y có quan hệ "t" với
nhau. Khi đó bài toán T đã được chuyển về bài toán D trên đồ thị.
3.1.2 Dựa vào các kết quả của lý thuyết đồ thị hoặc lý luận
trực tiếp suy ra đáp án của bài toán D.
Nếu đáp án của bài toán D còn dưới dạng "ngôn ngữ đồ thị", thì căn
cứ vào phép đặt tương ứng khi xây dựng đồ thị mà diễn đạt thành đáp
án bằng ngôn ngữ thông thường (tức là đáp án của bài toán T).
54
3.2 Bài toán về đỉnh - cạnh của đồ thị.
Bài toán 3.2.1. (Olympic Toán Mỹ 1982) Sống trong một ký túc xá có
1982 người. Cứ bốn người trong đó bao giờ cũng chọn được ít nhất một
người quen với cả ba người còn lại. Có ít nhất bao nhiêu người mà mỗi
người quen với tất cả những người trong ký túc xá.
Giải:
1. Xây dựng đồ thị G.
Đỉnh: Lấy mỗi đỉnh trong đồ thị tương ứng với mỗi người sống trong
ký túc xá.
Cạnh: Trong đồ thị G hai đỉnh x,y bất kỳ có cạnh được nối với nhau
khi và chỉ khi hai người x, y quen biết nhau.
Đồ thị G mô tả toàn bộ quan hệ quen biết nhau trong ký túc xá mà
ta xét.
2. Đáp án bài toán bằng ngôn ngữ đồ thị.
Với cách xây dựng trên, bậc của mỗi đỉnh thuộc G bằng đúng số người
quen của người tương ứng với đỉnh này.
Theo định lý 1.3.5 ta suy ra có ít nhât 1982−3 = 1979 người mà mỗi
người quen với tất cả những người trong ký túc xá.
Bài toán 3.2.2. Trường THPT Tân Dân có 1101 học sinh. Biết rằng
mỗi học sinh quen ít nhất 1001 học sinh. Chứng minh với mỗi học sinh
của trường luôn tìm được 11 học sinh khác để tạo thành một nhóm gồm
12 học sinh, sao cho hai học sinh bất kỳ trong nhóm đều quen nhau.
Giải:
1. Xây dựng đồ thị G.
Đỉnh: Lấy 1101 đỉnh trên mặt phẳng tương ứng với 1101 học sinh.
Cạnh: Trong đồ thị G hai đỉnh x,y bất kỳ được nối với nhau bằng
một cạnh khi và chỉ khi hai em học sinh x, y quen nhau nhau.
2. Đáp án bài toán bằng ngôn ngữ đồ thị.
Với cách xây dựng trên, bậc của mỗi đỉnh thuộc G bằng đúng số người
quen của người tương ứng với đỉnh này.
Theo định lý 1.3.7 số đỉnh của đồ thị G là
kn + 1 = 1101 (∗)
mỗi đỉnh có bậc không nhỏ hơn
(k − 1)n + 1 = 1001 (∗∗)
55
Do đó, luôn tồn tại đồ thị con đầy đủ gồm k + 1 đỉnh.
Từ (∗), (∗∗) ta có n = 100, k = 11 . Như vậy, sẽ tồn tại một nhóm
gồm k+1 = 11+1 = 12 em học sinh mà hai học sinh bất kỳ trong nhóm
đều quen nhau.
Bài toán 3.2.3. Liệu có thể có nhóm 9 người, mà trong đó mỗi người
chỉ quen biết đúng 5 người khác được không?
Giải:
1. Xây dựng đồ thị G.
Đỉnh: Lấy 9 điểm tương ứng với 9 người trong nhóm làm đỉnh của đồ
thị và đánh số từ 1 đến 9.
Cạnh: Trong đồ thị G hai đỉnh x,y bất kỳ được nối với nhau bằng
một cạnh khi và chỉ khi hai người x, y quen nhau.
Đồ thị G mô tả quan hệ quen nhau của 9 người trong nhóm.
Hình 3.1
2. Đáp án bài toán bằng ngôn ngữ đồ thị.
Với cách xây dựng trên, bậc của mỗi đỉnh thuộc G bằng đúng số người
quen của người tương ứng với đỉnh này.
Nếu theo đúng giả thiết, tất cả các đỉnh của đồ thị đều có bậc là 5
thì theo định lý 1.3.1 số cạnh của đồ thị là:
9.5
2
= 22, 5 /∈ N
56
không là số nguyên nên vô lý. Do đó, không thể có nhóm 9 người trong
đó mỗi người chỉ quen biết 5 người khác.
Bài toán 3.2.4. ([4]Lý thuyết đồ thị và các bài toán không mẫu mực)
Chứng minh rằng: Nếu trong một tập số nguyên dương tùy ý M gồm ít
nhất 3 số, mà có đúng 2 số có số số đồng dư bằng nhau, thì các số này
không thể đồng thời không đồng dư với một số nào hoặc đồng thời đồng
dư với tất cả các số còn lại thuộc tập M.
Chứng minh. .
1. Xây dựng đồ thị G.
Đỉnh: Lấy các điểm tương ứng với các số nguyên dương thuộc M làm
đỉnh của đồ thị. Dùng ngay các số thuộc M để ghi trên các đỉnh tương
ứng.
Cạnh: Trong đồ thị G hai đỉnh x,y bất kỳ được nối với nhau bằng
một cạnh khi và chỉ khi hai số x, y có số số đồng dư bằng nhau.
Đồ thị G mô tả quan hệ của các số nguyên thuộc tập M, mà số số
đồng dư của các số này là bằng nhau.
2. Đáp án bài toán bằng ngôn ngữ đồ thị.
Theo định lý 1.3.4 đồ thị G với n đỉnh có đúng 2 đỉnh cùng bậc thì
hai đỉnh này không thể đồng thời bậc 0 hoặc bậc n − 1 từ đó ta suy ra
ngay điều phải chứng minh.
Bài toán 3.2.5. Trong hội thi đấu cờ vua của khối các trường trung
học phổ thông thuộc huyện Phú Xuyên có 10 em học sinh đại diện cho
các trường tham gia thi đấu. Thể lệ cuộc thi là mỗi em phải đấu một
trận với các em khác. Chứng minh rằng bất kỳ lúc nào cũng có 2 em đã
đấu được một số trận như nhau.
Giải:
1. Xây dựng đồ thị G.
Đỉnh: Lấy mỗi điểm tương ứng với mỗi em học sinh tham gia thi đấu
làm đỉnh của đồ thị. Dùng ngay tên của mỗi em để ghi trên đỉnh tương
ứng.
Cạnh: Trong đồ thị G hai đỉnh x,y bất kỳ được nối với nhau bằng
một cạnh khi và chỉ khi hai 2 em học sinh x, y đã thi đấu cờ vua với
nhau.
Đồ thị G mô tả toàn bộ quan hệ thi đấu giữa các em học sinh.
2. Đáp án bài toán bằng ngôn ngữ đồ thị.
57
Với cách xây dựng trên, bậc của mỗi đỉnh thuộc G bằng đúng số trận
đấu mà em học sinh tương ứng với đỉnh này đã thi đấu.
Theo định lý 1.3.3 đồ thị G luôn có ít nhất 2 đỉnh cùng bậc. Tức bất
kỳ lúc nào cũng có 2 em đã đấu được một số trận như nhau.
Bài toán 3.2.6. Cho một khối đa diện lồi A1, A2, ..., An. Gọi m1, m2, ..., mn
lần lượt là số cạnh xuất phát từ các đỉnh A1, A2, ..., An và c là số cạnh
của khối đa diện. Khi đó ta có m1 + m2 + ... + mn = 2c.
Giải:
1. Xây dựng đồ thị G.
Đỉnh: Lấy mỗi điểm trong đồ thị tương ứng với mỗi đỉnh của khối đa
diện lồi.
Cạnh: Trong đồ thị G hai đỉnh x,y bất kỳ được nối với nhau bằng
một cạnh khi và chỉ khi cặp đỉnh x, y tạo thành cạnh của khối đa diện.
2. Đáp án bài toán bằng ngôn ngữ đồ thị.
Với cách xây dựng trên, bậc của mỗi đỉnh thuộc G bằng đúng số cạnh
xuất phát từ đỉnh tương ứng của khối đa diện.
Theo định lý 1.3.1 tổng số bậc của tất cả các đỉnh luôn gấp đôi số
cạnh. Do đó ta có m1 + m2 + ... + mn = 2c
3.3 Bài toán về xích, chu trình, đường, vòng và tính
liên thông của đồ thị.
Bài toán 3.3.1. ([4]Lý thuyết đồ thị và các bài toán không mẫu mực)
Một thôn có ít nhất 4 gia đình, mỗi gia đình thân với ít nhất 3 gia đình
khác. Chứng minh rằng có thể xếp một số chẵn gia đình làm nhà xung
quanh một cái hồ để mỗi gia đình sống giữa hai gia đình mà họ thân.
Giải:
1. Xây dựng đồ thị G.
Đỉnh: Đối với mỗi gia đình trong thôn lấy một điểm tương ứng làm
đỉnh của đồ thị và dùng ngay tên chủ hộ để ghi trên đỉnh tương ứng.
Cạnh: Trong đồ thị G hai đỉnh x,y bất kỳ được nối với nhau bằng
một cạnh khi và chỉ khi hai gia đình x, y thân nhau.
Đồ thị G mô tả toàn bộ quan hệ thân nhau giữa các gia đình trong
thôn.
2. Đáp án bài toán bằng ngôn ngữ đồ thị.
58
Với cách xây dựng trên, bậc của mỗi đỉnh thuộc G bằng đúng số gia
đình thân của gia đình tương ứng với đỉnh này.
Theo cách xác định cạnh, mỗi đỉnh x có số cạnh đúng bằng số gia
đình thân với gia đình x. Bởi vậy, mỗi đỉnh của đồ thị G đều có bậc
không nhỏ hơn 3. Do đó theo định lý 1.4.2, trong đồ thị G tồn tại chu
trình sơ cấp α độ dài chẵn. Khi đó dựa theo α mà sắp xếp chẵn gia đình
tương ứng làm nhà xung quanh hồ theo thứ tự các đỉnh thì mỗi gia đình
trong các gia đình này sẽ sống giữa hai gia đình mà họ thân.
Bài toán 3.3.2. ([3]Lý thuyết đồ thị và ứng dụng) Một tập số nguyên
dương M gồm ít nhất ba số. Mỗi số đều có ước chung với ít nhất hai số
khác. Chứng minh rằng luôn luôn có thể ghi một nhóm gồm ít nhất ba
số thuộc tập hợp lên một vòng tròn, để mỗi số đều đứng giữa hai số mà
nó có ước chung.
Giải:
1. Xây dựng đồ thị G.
Đỉnh: Đối với mỗi số nguyên dương thuộc tập M lấy một điểm tương
ứng làm đỉnh của đồ thị và ghi các số thuộc tập M lên các đỉnh tương
ứng.
Cạnh: Trong đồ thị G hai đỉnh x,y bất kỳ được nối với nhau bằng
một cạnh khi và chỉ khi hai số x, y có ước chung.
Đồ thị G mô tả toàn bộ quan hệ có chung ước với nhau giữa các số
trong tập hợp M.
2. Đáp án bài toán bằng ngôn ngữ đồ thị.
Vì mối số thuộc M có ước chung với ít nhất hai số khác, nên theo
định lý 1.4.1 trong G có chu trình sơ cấp. Khi đó dựa theo một trong
những chu trình sơ cấp của G mà ghi các số tương ứng lên một vòng
tròn, thì mỗi số đều đứng giữa hai số mà nó có ước chung.
Bài toán 3.3.3. (IMO 1991) Giả sử G là một đồ thị liên thông có k
cạnh. Chứng minh rằng: Có thể đánh nhãn được các cạnh 1, 2, 3,...,k
theo cách mà mỗi đỉnh thuộc vào hai hoặc nhiều hơn hai cạnh, ước số
chung lớn nhất của các số nguyên đánh nhãn các cạnh này là 1.
Giải:
Ta bắt đầu tại một đỉnh v0 nào đó. Tưởng tượng rằng ta đang đi dọc
theo các cạnh phân biệt của đồ thị, vừa đi vừa đánh số chúng như ta
đang đếm: 1,2,3,... cho đến khi ta không thể đi xa hơn được nữa vì nếu
muốn đi thêm phải dùng lại một cạnh đã đi qua.
59
Luận văn: Lý thuyết đồ thị với các bài toán phổ thông, HAY, 9đ
Luận văn: Lý thuyết đồ thị với các bài toán phổ thông, HAY, 9đ
Luận văn: Lý thuyết đồ thị với các bài toán phổ thông, HAY, 9đ
Luận văn: Lý thuyết đồ thị với các bài toán phổ thông, HAY, 9đ
Luận văn: Lý thuyết đồ thị với các bài toán phổ thông, HAY, 9đ
Luận văn: Lý thuyết đồ thị với các bài toán phổ thông, HAY, 9đ
Luận văn: Lý thuyết đồ thị với các bài toán phổ thông, HAY, 9đ
Luận văn: Lý thuyết đồ thị với các bài toán phổ thông, HAY, 9đ
Luận văn: Lý thuyết đồ thị với các bài toán phổ thông, HAY, 9đ
Luận văn: Lý thuyết đồ thị với các bài toán phổ thông, HAY, 9đ
Luận văn: Lý thuyết đồ thị với các bài toán phổ thông, HAY, 9đ
Luận văn: Lý thuyết đồ thị với các bài toán phổ thông, HAY, 9đ
Luận văn: Lý thuyết đồ thị với các bài toán phổ thông, HAY, 9đ
Luận văn: Lý thuyết đồ thị với các bài toán phổ thông, HAY, 9đ
Luận văn: Lý thuyết đồ thị với các bài toán phổ thông, HAY, 9đ
Luận văn: Lý thuyết đồ thị với các bài toán phổ thông, HAY, 9đ
Luận văn: Lý thuyết đồ thị với các bài toán phổ thông, HAY, 9đ
Luận văn: Lý thuyết đồ thị với các bài toán phổ thông, HAY, 9đ
Luận văn: Lý thuyết đồ thị với các bài toán phổ thông, HAY, 9đ
Luận văn: Lý thuyết đồ thị với các bài toán phổ thông, HAY, 9đ
Luận văn: Lý thuyết đồ thị với các bài toán phổ thông, HAY, 9đ
Luận văn: Lý thuyết đồ thị với các bài toán phổ thông, HAY, 9đ
Luận văn: Lý thuyết đồ thị với các bài toán phổ thông, HAY, 9đ
Luận văn: Lý thuyết đồ thị với các bài toán phổ thông, HAY, 9đ
Luận văn: Lý thuyết đồ thị với các bài toán phổ thông, HAY, 9đ
Luận văn: Lý thuyết đồ thị với các bài toán phổ thông, HAY, 9đ
Luận văn: Lý thuyết đồ thị với các bài toán phổ thông, HAY, 9đ
Luận văn: Lý thuyết đồ thị với các bài toán phổ thông, HAY, 9đ
Luận văn: Lý thuyết đồ thị với các bài toán phổ thông, HAY, 9đ
Luận văn: Lý thuyết đồ thị với các bài toán phổ thông, HAY, 9đ
Luận văn: Lý thuyết đồ thị với các bài toán phổ thông, HAY, 9đ

More Related Content

What's hot

201-bai-tap-phuong-trinh-vi-phan
 201-bai-tap-phuong-trinh-vi-phan 201-bai-tap-phuong-trinh-vi-phan
201-bai-tap-phuong-trinh-vi-phanSơn DC
 
Gt khong gian_metric Nguyen Hoang
Gt khong gian_metric Nguyen HoangGt khong gian_metric Nguyen Hoang
Gt khong gian_metric Nguyen HoangBui Loi
 
chuong 5. do thi (phan 1)
chuong 5. do thi (phan 1)chuong 5. do thi (phan 1)
chuong 5. do thi (phan 1)kikihoho
 
Các phương pháp đếm nâng cao
Các phương pháp đếm nâng caoCác phương pháp đếm nâng cao
Các phương pháp đếm nâng caoThế Giới Tinh Hoa
 
Hệ phương trình vi phân tuyến tính
Hệ phương trình vi phân tuyến tínhHệ phương trình vi phân tuyến tính
Hệ phương trình vi phân tuyến tínhThế Giới Tinh Hoa
 
Topo daicuong1[1]
Topo daicuong1[1]Topo daicuong1[1]
Topo daicuong1[1]Bui Loi
 
Bài toán số học liên quan tới lũy thữa
Bài toán số học liên quan tới lũy thữaBài toán số học liên quan tới lũy thữa
Bài toán số học liên quan tới lũy thữaThế Giới Tinh Hoa
 
Giáo trình Phân tích và thiết kế giải thuật - CHAP 1
Giáo trình Phân tích và thiết kế giải thuật - CHAP 1Giáo trình Phân tích và thiết kế giải thuật - CHAP 1
Giáo trình Phân tích và thiết kế giải thuật - CHAP 1Nguyễn Công Hoàng
 
Các phương pháp giải mũ. logarit
Các phương pháp giải mũ. logaritCác phương pháp giải mũ. logarit
Các phương pháp giải mũ. logaritThế Giới Tinh Hoa
 
Phương pháp nhánh cận
Phương pháp nhánh cậnPhương pháp nhánh cận
Phương pháp nhánh cậnDiên Vĩ
 
Hướng dẫn giải bài tập chuỗi - Toán cao cấp
Hướng dẫn giải bài tập chuỗi - Toán cao cấpHướng dẫn giải bài tập chuỗi - Toán cao cấp
Hướng dẫn giải bài tập chuỗi - Toán cao cấpVan-Duyet Le
 
Số phức-5-Dạng lượng giác của số phức và ứng dụng-pages 47-61
Số phức-5-Dạng lượng giác của số phức và ứng dụng-pages 47-61Số phức-5-Dạng lượng giác của số phức và ứng dụng-pages 47-61
Số phức-5-Dạng lượng giác của số phức và ứng dụng-pages 47-61lovestem
 
Bài Giảng Và Ngân Hàng Đề Thi OTOMAT
Bài Giảng Và Ngân Hàng Đề Thi OTOMATBài Giảng Và Ngân Hàng Đề Thi OTOMAT
Bài Giảng Và Ngân Hàng Đề Thi OTOMATHiệp Mông Chí
 
kỹ thuật giải phương trình hàm
kỹ thuật giải phương trình hàmkỹ thuật giải phương trình hàm
kỹ thuật giải phương trình hàmljmonking
 
Hinh hoc so cap va thuc hanh giai toan
Hinh hoc so cap va thuc hanh giai toanHinh hoc so cap va thuc hanh giai toan
Hinh hoc so cap va thuc hanh giai toanTamPhan59
 
Phương pháp tính giới hạn dãy số
Phương pháp tính giới hạn dãy sốPhương pháp tính giới hạn dãy số
Phương pháp tính giới hạn dãy sốThế Giới Tinh Hoa
 

What's hot (20)

201-bai-tap-phuong-trinh-vi-phan
 201-bai-tap-phuong-trinh-vi-phan 201-bai-tap-phuong-trinh-vi-phan
201-bai-tap-phuong-trinh-vi-phan
 
Gt khong gian_metric Nguyen Hoang
Gt khong gian_metric Nguyen HoangGt khong gian_metric Nguyen Hoang
Gt khong gian_metric Nguyen Hoang
 
Dãy số và giới hạn
Dãy số và giới hạnDãy số và giới hạn
Dãy số và giới hạn
 
Luận văn: Các bài toán về hệ thức lượng trong tam giác, HOT, 9đ
Luận văn: Các bài toán về hệ thức lượng trong tam giác, HOT, 9đLuận văn: Các bài toán về hệ thức lượng trong tam giác, HOT, 9đ
Luận văn: Các bài toán về hệ thức lượng trong tam giác, HOT, 9đ
 
chuong 5. do thi (phan 1)
chuong 5. do thi (phan 1)chuong 5. do thi (phan 1)
chuong 5. do thi (phan 1)
 
Các phương pháp đếm nâng cao
Các phương pháp đếm nâng caoCác phương pháp đếm nâng cao
Các phương pháp đếm nâng cao
 
Hệ phương trình vi phân tuyến tính
Hệ phương trình vi phân tuyến tínhHệ phương trình vi phân tuyến tính
Hệ phương trình vi phân tuyến tính
 
Topo daicuong1[1]
Topo daicuong1[1]Topo daicuong1[1]
Topo daicuong1[1]
 
Bài toán số học liên quan tới lũy thữa
Bài toán số học liên quan tới lũy thữaBài toán số học liên quan tới lũy thữa
Bài toán số học liên quan tới lũy thữa
 
Giáo trình Phân tích và thiết kế giải thuật - CHAP 1
Giáo trình Phân tích và thiết kế giải thuật - CHAP 1Giáo trình Phân tích và thiết kế giải thuật - CHAP 1
Giáo trình Phân tích và thiết kế giải thuật - CHAP 1
 
Các phương pháp giải mũ. logarit
Các phương pháp giải mũ. logaritCác phương pháp giải mũ. logarit
Các phương pháp giải mũ. logarit
 
Phương pháp nhánh cận
Phương pháp nhánh cậnPhương pháp nhánh cận
Phương pháp nhánh cận
 
Hướng dẫn giải bài tập chuỗi - Toán cao cấp
Hướng dẫn giải bài tập chuỗi - Toán cao cấpHướng dẫn giải bài tập chuỗi - Toán cao cấp
Hướng dẫn giải bài tập chuỗi - Toán cao cấp
 
Bồi dưỡng học sinh giỏi Toán Tiểu Học
Bồi dưỡng học sinh giỏi Toán Tiểu HọcBồi dưỡng học sinh giỏi Toán Tiểu Học
Bồi dưỡng học sinh giỏi Toán Tiểu Học
 
Đệ Quy, Quay Lui, Nhánh Cận
Đệ Quy, Quay Lui, Nhánh CậnĐệ Quy, Quay Lui, Nhánh Cận
Đệ Quy, Quay Lui, Nhánh Cận
 
Số phức-5-Dạng lượng giác của số phức và ứng dụng-pages 47-61
Số phức-5-Dạng lượng giác của số phức và ứng dụng-pages 47-61Số phức-5-Dạng lượng giác của số phức và ứng dụng-pages 47-61
Số phức-5-Dạng lượng giác của số phức và ứng dụng-pages 47-61
 
Bài Giảng Và Ngân Hàng Đề Thi OTOMAT
Bài Giảng Và Ngân Hàng Đề Thi OTOMATBài Giảng Và Ngân Hàng Đề Thi OTOMAT
Bài Giảng Và Ngân Hàng Đề Thi OTOMAT
 
kỹ thuật giải phương trình hàm
kỹ thuật giải phương trình hàmkỹ thuật giải phương trình hàm
kỹ thuật giải phương trình hàm
 
Hinh hoc so cap va thuc hanh giai toan
Hinh hoc so cap va thuc hanh giai toanHinh hoc so cap va thuc hanh giai toan
Hinh hoc so cap va thuc hanh giai toan
 
Phương pháp tính giới hạn dãy số
Phương pháp tính giới hạn dãy sốPhương pháp tính giới hạn dãy số
Phương pháp tính giới hạn dãy số
 

Similar to Luận văn: Lý thuyết đồ thị với các bài toán phổ thông, HAY, 9đ

Graph Theory - (Lý thuyết đồ thị) ĐHKHTN
Graph Theory - (Lý thuyết đồ thị) ĐHKHTNGraph Theory - (Lý thuyết đồ thị) ĐHKHTN
Graph Theory - (Lý thuyết đồ thị) ĐHKHTNFumthsAe
 
Tom tat bai giang ly thuyet do thi - nguyen ngoc trung
Tom tat bai giang   ly thuyet do thi - nguyen ngoc trungTom tat bai giang   ly thuyet do thi - nguyen ngoc trung
Tom tat bai giang ly thuyet do thi - nguyen ngoc trungPhi Phi
 
[Math educare.com] giai tich ham nhieu bien-phep tinh vi phan ham nhieu bien_...
[Math educare.com] giai tich ham nhieu bien-phep tinh vi phan ham nhieu bien_...[Math educare.com] giai tich ham nhieu bien-phep tinh vi phan ham nhieu bien_...
[Math educare.com] giai tich ham nhieu bien-phep tinh vi phan ham nhieu bien_...Nguyen Vietnam
 
Tiểu luận đường đi HAMILTON
Tiểu luận đường đi HAMILTON Tiểu luận đường đi HAMILTON
Tiểu luận đường đi HAMILTON nataliej4
 
he-thong-cac-khai-niem-co-ban-va-dinh-ly-hinh-hoc-thcs-hinh-hoc-phang.pdf
he-thong-cac-khai-niem-co-ban-va-dinh-ly-hinh-hoc-thcs-hinh-hoc-phang.pdfhe-thong-cac-khai-niem-co-ban-va-dinh-ly-hinh-hoc-thcs-hinh-hoc-phang.pdf
he-thong-cac-khai-niem-co-ban-va-dinh-ly-hinh-hoc-thcs-hinh-hoc-phang.pdfcholacha
 
ứng dụng của tích phân
ứng dụng của tích phânứng dụng của tích phân
ứng dụng của tích phânOanh MJ
 
Luận văn: Phương trình liên hợp và ứng dụng của nó, HOT - Gửi miễn phí qua za...
Luận văn: Phương trình liên hợp và ứng dụng của nó, HOT - Gửi miễn phí qua za...Luận văn: Phương trình liên hợp và ứng dụng của nó, HOT - Gửi miễn phí qua za...
Luận văn: Phương trình liên hợp và ứng dụng của nó, HOT - Gửi miễn phí qua za...Dịch vụ viết bài trọn gói ZALO: 0909232620
 

Similar to Luận văn: Lý thuyết đồ thị với các bài toán phổ thông, HAY, 9đ (20)

Luận văn: Định lý bézout và chiều ngược lại, HAY, 9đ
Luận văn: Định lý bézout và chiều ngược lại, HAY, 9đLuận văn: Định lý bézout và chiều ngược lại, HAY, 9đ
Luận văn: Định lý bézout và chiều ngược lại, HAY, 9đ
 
Luận văn: Mô hình đồ thị luồng và mối quan hệ với đồ thị, HAY
Luận văn: Mô hình đồ thị luồng và mối quan hệ với đồ thị, HAYLuận văn: Mô hình đồ thị luồng và mối quan hệ với đồ thị, HAY
Luận văn: Mô hình đồ thị luồng và mối quan hệ với đồ thị, HAY
 
Luận văn: Phân tích tai của đồ thị và đồ thị series parallel, HAY
Luận văn: Phân tích tai của đồ thị và đồ thị series parallel, HAYLuận văn: Phân tích tai của đồ thị và đồ thị series parallel, HAY
Luận văn: Phân tích tai của đồ thị và đồ thị series parallel, HAY
 
Graph Theory - (Lý thuyết đồ thị) ĐHKHTN
Graph Theory - (Lý thuyết đồ thị) ĐHKHTNGraph Theory - (Lý thuyết đồ thị) ĐHKHTN
Graph Theory - (Lý thuyết đồ thị) ĐHKHTN
 
Tom tat bai giang ly thuyet do thi - nguyen ngoc trung
Tom tat bai giang   ly thuyet do thi - nguyen ngoc trungTom tat bai giang   ly thuyet do thi - nguyen ngoc trung
Tom tat bai giang ly thuyet do thi - nguyen ngoc trung
 
Luận văn: Phương trình tích phân kỳ dị với dịch chuyển và phản xạ
Luận văn: Phương trình tích phân kỳ dị với dịch chuyển và phản xạLuận văn: Phương trình tích phân kỳ dị với dịch chuyển và phản xạ
Luận văn: Phương trình tích phân kỳ dị với dịch chuyển và phản xạ
 
[Math educare.com] giai tich ham nhieu bien-phep tinh vi phan ham nhieu bien_...
[Math educare.com] giai tich ham nhieu bien-phep tinh vi phan ham nhieu bien_...[Math educare.com] giai tich ham nhieu bien-phep tinh vi phan ham nhieu bien_...
[Math educare.com] giai tich ham nhieu bien-phep tinh vi phan ham nhieu bien_...
 
Tiểu luận đường đi HAMILTON
Tiểu luận đường đi HAMILTON Tiểu luận đường đi HAMILTON
Tiểu luận đường đi HAMILTON
 
Luận văn: Bất đẳng thức trong lớp hàm siêu việt, HAY, 9đ
Luận văn: Bất đẳng thức trong lớp hàm siêu việt, HAY, 9đLuận văn: Bất đẳng thức trong lớp hàm siêu việt, HAY, 9đ
Luận văn: Bất đẳng thức trong lớp hàm siêu việt, HAY, 9đ
 
Luận án: Giải bất đẳng thức biến phân trên tập điểm bất động, HAY
Luận án: Giải bất đẳng thức biến phân trên tập điểm bất động, HAYLuận án: Giải bất đẳng thức biến phân trên tập điểm bất động, HAY
Luận án: Giải bất đẳng thức biến phân trên tập điểm bất động, HAY
 
Luận văn: Mặt cực tiểu kiểu đồ thị trong không gian R ×ω R 2, HAY
Luận văn: Mặt cực tiểu kiểu đồ thị trong không gian R ×ω R 2, HAYLuận văn: Mặt cực tiểu kiểu đồ thị trong không gian R ×ω R 2, HAY
Luận văn: Mặt cực tiểu kiểu đồ thị trong không gian R ×ω R 2, HAY
 
Luận văn: Phương pháp xây dựng độ đo và tích phân, HOT, 9đ
Luận văn: Phương pháp xây dựng độ đo và tích phân, HOT, 9đLuận văn: Phương pháp xây dựng độ đo và tích phân, HOT, 9đ
Luận văn: Phương pháp xây dựng độ đo và tích phân, HOT, 9đ
 
he-thong-cac-khai-niem-co-ban-va-dinh-ly-hinh-hoc-thcs-hinh-hoc-phang.pdf
he-thong-cac-khai-niem-co-ban-va-dinh-ly-hinh-hoc-thcs-hinh-hoc-phang.pdfhe-thong-cac-khai-niem-co-ban-va-dinh-ly-hinh-hoc-thcs-hinh-hoc-phang.pdf
he-thong-cac-khai-niem-co-ban-va-dinh-ly-hinh-hoc-thcs-hinh-hoc-phang.pdf
 
ứng dụng của tích phân
ứng dụng của tích phânứng dụng của tích phân
ứng dụng của tích phân
 
[Vnmath.com] skkn-bui thi hang
[Vnmath.com] skkn-bui thi hang[Vnmath.com] skkn-bui thi hang
[Vnmath.com] skkn-bui thi hang
 
Luận văn: Phương trình liên hợp và ứng dụng, HAY, 9đ
Luận văn: Phương trình liên hợp và ứng dụng, HAY, 9đLuận văn: Phương trình liên hợp và ứng dụng, HAY, 9đ
Luận văn: Phương trình liên hợp và ứng dụng, HAY, 9đ
 
Luận văn: Phương trình liên hợp và ứng dụng của nó, HOT - Gửi miễn phí qua za...
Luận văn: Phương trình liên hợp và ứng dụng của nó, HOT - Gửi miễn phí qua za...Luận văn: Phương trình liên hợp và ứng dụng của nó, HOT - Gửi miễn phí qua za...
Luận văn: Phương trình liên hợp và ứng dụng của nó, HOT - Gửi miễn phí qua za...
 
Ltdt
LtdtLtdt
Ltdt
 
Luận văn: Giải số phương trình vi phân đại số bằng đa bước, 9đ
Luận văn: Giải số phương trình vi phân đại số bằng đa bước, 9đLuận văn: Giải số phương trình vi phân đại số bằng đa bước, 9đ
Luận văn: Giải số phương trình vi phân đại số bằng đa bước, 9đ
 
Luận văn: Một số phương pháp giải phương trình hàm, HOT, 9đ
Luận văn: Một số phương pháp giải phương trình hàm, HOT, 9đLuận văn: Một số phương pháp giải phương trình hàm, HOT, 9đ
Luận văn: Một số phương pháp giải phương trình hàm, HOT, 9đ
 

More from Dịch vụ viết bài trọn gói ZALO: 0909232620

Danh Sách 200 Đề Tài Tiểu Luận Chuyên Viên Chính Về Bảo Hiểm Xã Hội Mới Nhất
Danh Sách 200 Đề Tài Tiểu Luận Chuyên Viên Chính Về Bảo Hiểm Xã Hội Mới NhấtDanh Sách 200 Đề Tài Tiểu Luận Chuyên Viên Chính Về Bảo Hiểm Xã Hội Mới Nhất
Danh Sách 200 Đề Tài Tiểu Luận Chuyên Viên Chính Về Bảo Hiểm Xã Hội Mới NhấtDịch vụ viết bài trọn gói ZALO: 0909232620
 
Danh Sách 200 Đề Tài Báo Cáo Thực Tập Luật Phòng, Chống Hiv, Mới Nhất, Điểm Cao
Danh Sách 200 Đề Tài Báo Cáo Thực Tập Luật Phòng, Chống Hiv, Mới Nhất, Điểm CaoDanh Sách 200 Đề Tài Báo Cáo Thực Tập Luật Phòng, Chống Hiv, Mới Nhất, Điểm Cao
Danh Sách 200 Đề Tài Báo Cáo Thực Tập Luật Phòng, Chống Hiv, Mới Nhất, Điểm CaoDịch vụ viết bài trọn gói ZALO: 0909232620
 

More from Dịch vụ viết bài trọn gói ZALO: 0909232620 (20)

Danh Sách 200 Đề Tài Tiểu Luận Chuyên Viên Chính Về Bảo Hiểm Xã Hội Mới Nhất
Danh Sách 200 Đề Tài Tiểu Luận Chuyên Viên Chính Về Bảo Hiểm Xã Hội Mới NhấtDanh Sách 200 Đề Tài Tiểu Luận Chuyên Viên Chính Về Bảo Hiểm Xã Hội Mới Nhất
Danh Sách 200 Đề Tài Tiểu Luận Chuyên Viên Chính Về Bảo Hiểm Xã Hội Mới Nhất
 
Danh Sách 200 Đề Tài Luận Văn Thạc Sĩ Quản Trị Nguồn Nhân Lực, 9 Điểm
Danh Sách 200 Đề Tài Luận Văn Thạc Sĩ Quản Trị Nguồn Nhân Lực, 9 ĐiểmDanh Sách 200 Đề Tài Luận Văn Thạc Sĩ Quản Trị Nguồn Nhân Lực, 9 Điểm
Danh Sách 200 Đề Tài Luận Văn Thạc Sĩ Quản Trị Nguồn Nhân Lực, 9 Điểm
 
Danh Sách 200 Đề Tài Luận Văn Thạc Sĩ Quản Lý Văn Hóa Giúp Bạn Thêm Ý Tưởng
Danh Sách 200 Đề Tài Luận Văn Thạc Sĩ Quản Lý Văn Hóa Giúp Bạn Thêm Ý TưởngDanh Sách 200 Đề Tài Luận Văn Thạc Sĩ Quản Lý Văn Hóa Giúp Bạn Thêm Ý Tưởng
Danh Sách 200 Đề Tài Luận Văn Thạc Sĩ Quản Lý Văn Hóa Giúp Bạn Thêm Ý Tưởng
 
Danh Sách 200 Đề Tài Báo Cáo Thực Tập Quản Lý Giáo Dục Dễ Làm Điểm Cao
Danh Sách 200 Đề Tài Báo Cáo Thực Tập Quản Lý Giáo Dục Dễ Làm Điểm CaoDanh Sách 200 Đề Tài Báo Cáo Thực Tập Quản Lý Giáo Dục Dễ Làm Điểm Cao
Danh Sách 200 Đề Tài Báo Cáo Thực Tập Quản Lý Giáo Dục Dễ Làm Điểm Cao
 
Danh Sách 200 Đề Tài Báo Cáo Thực Tập Quan Hệ Lao Động Từ Sinh Viên Giỏi
Danh Sách 200 Đề Tài Báo Cáo Thực Tập Quan Hệ Lao Động Từ Sinh Viên GiỏiDanh Sách 200 Đề Tài Báo Cáo Thực Tập Quan Hệ Lao Động Từ Sinh Viên Giỏi
Danh Sách 200 Đề Tài Báo Cáo Thực Tập Quan Hệ Lao Động Từ Sinh Viên Giỏi
 
Danh Sách 200 Đề Tài Báo Cáo Thực Tập Nuôi Trồng Thủy Sản Dễ Làm Nhất
Danh Sách 200 Đề Tài Báo Cáo Thực Tập Nuôi Trồng Thủy Sản Dễ Làm NhấtDanh Sách 200 Đề Tài Báo Cáo Thực Tập Nuôi Trồng Thủy Sản Dễ Làm Nhất
Danh Sách 200 Đề Tài Báo Cáo Thực Tập Nuôi Trồng Thủy Sản Dễ Làm Nhất
 
Danh Sách 200 Đề Tài Báo Cáo Thực Tập Luật Sư, Mới Nhất, Điểm Cao
Danh Sách 200 Đề Tài Báo Cáo Thực Tập Luật Sư, Mới Nhất, Điểm CaoDanh Sách 200 Đề Tài Báo Cáo Thực Tập Luật Sư, Mới Nhất, Điểm Cao
Danh Sách 200 Đề Tài Báo Cáo Thực Tập Luật Sư, Mới Nhất, Điểm Cao
 
Danh Sách 200 Đề Tài Báo Cáo Thực Tập Luật Phòng, Chống Hiv, Mới Nhất, Điểm Cao
Danh Sách 200 Đề Tài Báo Cáo Thực Tập Luật Phòng, Chống Hiv, Mới Nhất, Điểm CaoDanh Sách 200 Đề Tài Báo Cáo Thực Tập Luật Phòng, Chống Hiv, Mới Nhất, Điểm Cao
Danh Sách 200 Đề Tài Báo Cáo Thực Tập Luật Phòng, Chống Hiv, Mới Nhất, Điểm Cao
 
Danh Sách 200 Đề Tài Báo Cáo Thực Tập Luật Phá Sản, Mới Nhất
Danh Sách 200 Đề Tài Báo Cáo Thực Tập Luật Phá Sản, Mới NhấtDanh Sách 200 Đề Tài Báo Cáo Thực Tập Luật Phá Sản, Mới Nhất
Danh Sách 200 Đề Tài Báo Cáo Thực Tập Luật Phá Sản, Mới Nhất
 
Danh Sách 200 Đề Tài Báo Cáo Thực Tập Luật Nhà Ở, Điểm Cao
Danh Sách 200 Đề Tài Báo Cáo Thực Tập Luật Nhà Ở, Điểm CaoDanh Sách 200 Đề Tài Báo Cáo Thực Tập Luật Nhà Ở, Điểm Cao
Danh Sách 200 Đề Tài Báo Cáo Thực Tập Luật Nhà Ở, Điểm Cao
 
Danh Sách 200 Đề Tài Báo Cáo Thực Tập Luật Ngân Hàng, Mới Nhất
Danh Sách 200 Đề Tài Báo Cáo Thực Tập Luật Ngân Hàng, Mới NhấtDanh Sách 200 Đề Tài Báo Cáo Thực Tập Luật Ngân Hàng, Mới Nhất
Danh Sách 200 Đề Tài Báo Cáo Thực Tập Luật Ngân Hàng, Mới Nhất
 
Danh Sách 200 Đề Tài Báo Cáo Thực Tập Luật Môi Trường, Mới Nhất
Danh Sách 200 Đề Tài Báo Cáo Thực Tập Luật Môi Trường, Mới NhấtDanh Sách 200 Đề Tài Báo Cáo Thực Tập Luật Môi Trường, Mới Nhất
Danh Sách 200 Đề Tài Báo Cáo Thực Tập Luật Môi Trường, Mới Nhất
 
Danh Sách 200 Đề Tài Báo Cáo Thực Tập Luật Hộ Tịch, Điểm Cao
Danh Sách 200 Đề Tài Báo Cáo Thực Tập Luật Hộ Tịch, Điểm CaoDanh Sách 200 Đề Tài Báo Cáo Thực Tập Luật Hộ Tịch, Điểm Cao
Danh Sách 200 Đề Tài Báo Cáo Thực Tập Luật Hộ Tịch, Điểm Cao
 
Danh Sách 200 Đề Tài Báo Cáo Thực Tập Luật Hình Sự , Dễ Làm Điểm Cao
Danh Sách 200 Đề Tài Báo Cáo Thực Tập Luật Hình Sự , Dễ Làm Điểm CaoDanh Sách 200 Đề Tài Báo Cáo Thực Tập Luật Hình Sự , Dễ Làm Điểm Cao
Danh Sách 200 Đề Tài Báo Cáo Thực Tập Luật Hình Sự , Dễ Làm Điểm Cao
 
Danh Sách 200 Đề Tài Báo Cáo Thực Tập Luật Hành Chính, Dễ Làm Điểm Cao
Danh Sách 200 Đề Tài Báo Cáo Thực Tập Luật Hành Chính, Dễ Làm Điểm CaoDanh Sách 200 Đề Tài Báo Cáo Thực Tập Luật Hành Chính, Dễ Làm Điểm Cao
Danh Sách 200 Đề Tài Báo Cáo Thực Tập Luật Hành Chính, Dễ Làm Điểm Cao
 
Danh Sách 200 Đề Tài Báo Cáo Thực Tập Luật Giáo Dục, Điểm Cao
Danh Sách 200 Đề Tài Báo Cáo Thực Tập Luật Giáo Dục, Điểm CaoDanh Sách 200 Đề Tài Báo Cáo Thực Tập Luật Giáo Dục, Điểm Cao
Danh Sách 200 Đề Tài Báo Cáo Thực Tập Luật Giáo Dục, Điểm Cao
 
Danh Sách 200 Đề Tài Báo Cáo Thực Tập Luật Đấu Thầu, Từ Sinh Viên Khá Giỏi
Danh Sách 200 Đề Tài Báo Cáo Thực Tập Luật Đấu Thầu, Từ Sinh Viên Khá GiỏiDanh Sách 200 Đề Tài Báo Cáo Thực Tập Luật Đấu Thầu, Từ Sinh Viên Khá Giỏi
Danh Sách 200 Đề Tài Báo Cáo Thực Tập Luật Đấu Thầu, Từ Sinh Viên Khá Giỏi
 
Danh Sách 200 Đề Tài Báo Cáo Thực Tập Luật Đầu Tư, Dễ Làm Điểm Cao
Danh Sách 200 Đề Tài Báo Cáo Thực Tập Luật Đầu Tư, Dễ Làm Điểm CaoDanh Sách 200 Đề Tài Báo Cáo Thực Tập Luật Đầu Tư, Dễ Làm Điểm Cao
Danh Sách 200 Đề Tài Báo Cáo Thực Tập Luật Đầu Tư, Dễ Làm Điểm Cao
 
Danh Sách 200 Đề Tài Báo Cáo Thực Tập Luật Đầu Tư Công, Dễ Làm Điểm Cao
Danh Sách 200 Đề Tài Báo Cáo Thực Tập Luật Đầu Tư Công, Dễ Làm Điểm CaoDanh Sách 200 Đề Tài Báo Cáo Thực Tập Luật Đầu Tư Công, Dễ Làm Điểm Cao
Danh Sách 200 Đề Tài Báo Cáo Thực Tập Luật Đầu Tư Công, Dễ Làm Điểm Cao
 
Danh Sách 200 Đề Tài Báo Cáo Thực Tập Luật Đất Đai, Từ Sinh Viên Khá Giỏi
Danh Sách 200 Đề Tài Báo Cáo Thực Tập Luật Đất Đai, Từ Sinh Viên Khá GiỏiDanh Sách 200 Đề Tài Báo Cáo Thực Tập Luật Đất Đai, Từ Sinh Viên Khá Giỏi
Danh Sách 200 Đề Tài Báo Cáo Thực Tập Luật Đất Đai, Từ Sinh Viên Khá Giỏi
 

Recently uploaded

Đề thi tin học HK2 lớp 3 Chân Trời Sáng Tạo
Đề thi tin học HK2 lớp 3 Chân Trời Sáng TạoĐề thi tin học HK2 lớp 3 Chân Trời Sáng Tạo
Đề thi tin học HK2 lớp 3 Chân Trời Sáng Tạowindcances
 
TUYỂN TẬP ĐỀ THI GIỮA KÌ, CUỐI KÌ 2 MÔN VẬT LÍ LỚP 11 THEO HÌNH THỨC THI MỚI ...
TUYỂN TẬP ĐỀ THI GIỮA KÌ, CUỐI KÌ 2 MÔN VẬT LÍ LỚP 11 THEO HÌNH THỨC THI MỚI ...TUYỂN TẬP ĐỀ THI GIỮA KÌ, CUỐI KÌ 2 MÔN VẬT LÍ LỚP 11 THEO HÌNH THỨC THI MỚI ...
TUYỂN TẬP ĐỀ THI GIỮA KÌ, CUỐI KÌ 2 MÔN VẬT LÍ LỚP 11 THEO HÌNH THỨC THI MỚI ...Nguyen Thanh Tu Collection
 
26 Truyện Ngắn Sơn Nam (Sơn Nam) thuviensach.vn.pdf
26 Truyện Ngắn Sơn Nam (Sơn Nam) thuviensach.vn.pdf26 Truyện Ngắn Sơn Nam (Sơn Nam) thuviensach.vn.pdf
26 Truyện Ngắn Sơn Nam (Sơn Nam) thuviensach.vn.pdfltbdieu
 
Tử Vi Là Gì Học Luận Giải Tử Vi Và Luận Đoán Vận Hạn
Tử Vi Là Gì Học Luận Giải Tử Vi Và Luận Đoán Vận HạnTử Vi Là Gì Học Luận Giải Tử Vi Và Luận Đoán Vận Hạn
Tử Vi Là Gì Học Luận Giải Tử Vi Và Luận Đoán Vận HạnKabala
 
Giáo trình xây dựng thực đơn. Ths Hoang Ngoc Hien.pdf
Giáo trình xây dựng thực đơn. Ths Hoang Ngoc Hien.pdfGiáo trình xây dựng thực đơn. Ths Hoang Ngoc Hien.pdf
Giáo trình xây dựng thực đơn. Ths Hoang Ngoc Hien.pdf4pdx29gsr9
 
60 CÂU HỎI ÔN TẬP LÝ LUẬN CHÍNH TRỊ NĂM 2024.docx
60 CÂU HỎI ÔN TẬP LÝ LUẬN CHÍNH TRỊ NĂM 2024.docx60 CÂU HỎI ÔN TẬP LÝ LUẬN CHÍNH TRỊ NĂM 2024.docx
60 CÂU HỎI ÔN TẬP LÝ LUẬN CHÍNH TRỊ NĂM 2024.docxasdnguyendinhdang
 
bài thi bảo vệ nền tảng tư tưởng của Đảng.docx
bài thi bảo vệ nền tảng tư tưởng của Đảng.docxbài thi bảo vệ nền tảng tư tưởng của Đảng.docx
bài thi bảo vệ nền tảng tư tưởng của Đảng.docxTrnHiYn5
 
Danh sách sinh viên tốt nghiệp Đại học - Cao đẳng Trường Đại học Phú Yên năm ...
Danh sách sinh viên tốt nghiệp Đại học - Cao đẳng Trường Đại học Phú Yên năm ...Danh sách sinh viên tốt nghiệp Đại học - Cao đẳng Trường Đại học Phú Yên năm ...
Danh sách sinh viên tốt nghiệp Đại học - Cao đẳng Trường Đại học Phú Yên năm ...hoangtuansinh1
 
Trắc nghiệm CHƯƠNG 5 môn Chủ nghĩa xã hội
Trắc nghiệm CHƯƠNG 5 môn Chủ nghĩa xã hộiTrắc nghiệm CHƯƠNG 5 môn Chủ nghĩa xã hội
Trắc nghiệm CHƯƠNG 5 môn Chủ nghĩa xã hộiNgocNguyen591215
 
Giới Thiệu Về Kabala | Hành Trình Thấu Hiểu Bản Thân | Kabala.vn
Giới Thiệu Về Kabala | Hành Trình Thấu Hiểu Bản Thân | Kabala.vnGiới Thiệu Về Kabala | Hành Trình Thấu Hiểu Bản Thân | Kabala.vn
Giới Thiệu Về Kabala | Hành Trình Thấu Hiểu Bản Thân | Kabala.vnKabala
 
Bài học phòng cháy chữa cháy - PCCC tại tòa nhà
Bài học phòng cháy chữa cháy - PCCC tại tòa nhàBài học phòng cháy chữa cháy - PCCC tại tòa nhà
Bài học phòng cháy chữa cháy - PCCC tại tòa nhàNguyen Thi Trang Nhung
 
các nội dung phòng chống xâm hại tình dục ở trẻ em
các nội dung phòng chống xâm hại tình dục ở trẻ emcác nội dung phòng chống xâm hại tình dục ở trẻ em
các nội dung phòng chống xâm hại tình dục ở trẻ emTrangNhung96
 
xemsomenh.com-Vòng Thái Tuế và Ý Nghĩa Các Sao Tại Cung Mệnh.pdf
xemsomenh.com-Vòng Thái Tuế và Ý Nghĩa Các Sao Tại Cung Mệnh.pdfxemsomenh.com-Vòng Thái Tuế và Ý Nghĩa Các Sao Tại Cung Mệnh.pdf
xemsomenh.com-Vòng Thái Tuế và Ý Nghĩa Các Sao Tại Cung Mệnh.pdfXem Số Mệnh
 
30 ĐỀ PHÁT TRIỂN THEO CẤU TRÚC ĐỀ MINH HỌA BGD NGÀY 22-3-2024 KỲ THI TỐT NGHI...
30 ĐỀ PHÁT TRIỂN THEO CẤU TRÚC ĐỀ MINH HỌA BGD NGÀY 22-3-2024 KỲ THI TỐT NGHI...30 ĐỀ PHÁT TRIỂN THEO CẤU TRÚC ĐỀ MINH HỌA BGD NGÀY 22-3-2024 KỲ THI TỐT NGHI...
30 ĐỀ PHÁT TRIỂN THEO CẤU TRÚC ĐỀ MINH HỌA BGD NGÀY 22-3-2024 KỲ THI TỐT NGHI...Nguyen Thanh Tu Collection
 
ĐỀ KIỂM TRA CUỐI KÌ 2 BIÊN SOẠN THEO ĐỊNH HƯỚNG ĐỀ BGD 2025 MÔN TOÁN 10 - CÁN...
ĐỀ KIỂM TRA CUỐI KÌ 2 BIÊN SOẠN THEO ĐỊNH HƯỚNG ĐỀ BGD 2025 MÔN TOÁN 10 - CÁN...ĐỀ KIỂM TRA CUỐI KÌ 2 BIÊN SOẠN THEO ĐỊNH HƯỚNG ĐỀ BGD 2025 MÔN TOÁN 10 - CÁN...
ĐỀ KIỂM TRA CUỐI KÌ 2 BIÊN SOẠN THEO ĐỊNH HƯỚNG ĐỀ BGD 2025 MÔN TOÁN 10 - CÁN...Nguyen Thanh Tu Collection
 
C6. Van de dan toc va ton giao ....pdf . Chu nghia xa hoi
C6. Van de dan toc va ton giao ....pdf . Chu nghia xa hoiC6. Van de dan toc va ton giao ....pdf . Chu nghia xa hoi
C6. Van de dan toc va ton giao ....pdf . Chu nghia xa hoidnghia2002
 
Access: Chuong III Thiet ke truy van Query.ppt
Access: Chuong III Thiet ke truy van Query.pptAccess: Chuong III Thiet ke truy van Query.ppt
Access: Chuong III Thiet ke truy van Query.pptPhamThiThuThuy1
 
30 ĐỀ PHÁT TRIỂN THEO CẤU TRÚC ĐỀ MINH HỌA BGD NGÀY 22-3-2024 KỲ THI TỐT NGHI...
30 ĐỀ PHÁT TRIỂN THEO CẤU TRÚC ĐỀ MINH HỌA BGD NGÀY 22-3-2024 KỲ THI TỐT NGHI...30 ĐỀ PHÁT TRIỂN THEO CẤU TRÚC ĐỀ MINH HỌA BGD NGÀY 22-3-2024 KỲ THI TỐT NGHI...
30 ĐỀ PHÁT TRIỂN THEO CẤU TRÚC ĐỀ MINH HỌA BGD NGÀY 22-3-2024 KỲ THI TỐT NGHI...Nguyen Thanh Tu Collection
 
xemsomenh.com-Vòng Lộc Tồn - Vòng Bác Sĩ và Cách An Trong Vòng Lộc Tồn.pdf
xemsomenh.com-Vòng Lộc Tồn - Vòng Bác Sĩ và Cách An Trong Vòng Lộc Tồn.pdfxemsomenh.com-Vòng Lộc Tồn - Vòng Bác Sĩ và Cách An Trong Vòng Lộc Tồn.pdf
xemsomenh.com-Vòng Lộc Tồn - Vòng Bác Sĩ và Cách An Trong Vòng Lộc Tồn.pdfXem Số Mệnh
 
Giáo trình nhập môn lập trình - Đặng Bình Phương
Giáo trình nhập môn lập trình - Đặng Bình PhươngGiáo trình nhập môn lập trình - Đặng Bình Phương
Giáo trình nhập môn lập trình - Đặng Bình Phươnghazzthuan
 

Recently uploaded (20)

Đề thi tin học HK2 lớp 3 Chân Trời Sáng Tạo
Đề thi tin học HK2 lớp 3 Chân Trời Sáng TạoĐề thi tin học HK2 lớp 3 Chân Trời Sáng Tạo
Đề thi tin học HK2 lớp 3 Chân Trời Sáng Tạo
 
TUYỂN TẬP ĐỀ THI GIỮA KÌ, CUỐI KÌ 2 MÔN VẬT LÍ LỚP 11 THEO HÌNH THỨC THI MỚI ...
TUYỂN TẬP ĐỀ THI GIỮA KÌ, CUỐI KÌ 2 MÔN VẬT LÍ LỚP 11 THEO HÌNH THỨC THI MỚI ...TUYỂN TẬP ĐỀ THI GIỮA KÌ, CUỐI KÌ 2 MÔN VẬT LÍ LỚP 11 THEO HÌNH THỨC THI MỚI ...
TUYỂN TẬP ĐỀ THI GIỮA KÌ, CUỐI KÌ 2 MÔN VẬT LÍ LỚP 11 THEO HÌNH THỨC THI MỚI ...
 
26 Truyện Ngắn Sơn Nam (Sơn Nam) thuviensach.vn.pdf
26 Truyện Ngắn Sơn Nam (Sơn Nam) thuviensach.vn.pdf26 Truyện Ngắn Sơn Nam (Sơn Nam) thuviensach.vn.pdf
26 Truyện Ngắn Sơn Nam (Sơn Nam) thuviensach.vn.pdf
 
Tử Vi Là Gì Học Luận Giải Tử Vi Và Luận Đoán Vận Hạn
Tử Vi Là Gì Học Luận Giải Tử Vi Và Luận Đoán Vận HạnTử Vi Là Gì Học Luận Giải Tử Vi Và Luận Đoán Vận Hạn
Tử Vi Là Gì Học Luận Giải Tử Vi Và Luận Đoán Vận Hạn
 
Giáo trình xây dựng thực đơn. Ths Hoang Ngoc Hien.pdf
Giáo trình xây dựng thực đơn. Ths Hoang Ngoc Hien.pdfGiáo trình xây dựng thực đơn. Ths Hoang Ngoc Hien.pdf
Giáo trình xây dựng thực đơn. Ths Hoang Ngoc Hien.pdf
 
60 CÂU HỎI ÔN TẬP LÝ LUẬN CHÍNH TRỊ NĂM 2024.docx
60 CÂU HỎI ÔN TẬP LÝ LUẬN CHÍNH TRỊ NĂM 2024.docx60 CÂU HỎI ÔN TẬP LÝ LUẬN CHÍNH TRỊ NĂM 2024.docx
60 CÂU HỎI ÔN TẬP LÝ LUẬN CHÍNH TRỊ NĂM 2024.docx
 
bài thi bảo vệ nền tảng tư tưởng của Đảng.docx
bài thi bảo vệ nền tảng tư tưởng của Đảng.docxbài thi bảo vệ nền tảng tư tưởng của Đảng.docx
bài thi bảo vệ nền tảng tư tưởng của Đảng.docx
 
Danh sách sinh viên tốt nghiệp Đại học - Cao đẳng Trường Đại học Phú Yên năm ...
Danh sách sinh viên tốt nghiệp Đại học - Cao đẳng Trường Đại học Phú Yên năm ...Danh sách sinh viên tốt nghiệp Đại học - Cao đẳng Trường Đại học Phú Yên năm ...
Danh sách sinh viên tốt nghiệp Đại học - Cao đẳng Trường Đại học Phú Yên năm ...
 
Trắc nghiệm CHƯƠNG 5 môn Chủ nghĩa xã hội
Trắc nghiệm CHƯƠNG 5 môn Chủ nghĩa xã hộiTrắc nghiệm CHƯƠNG 5 môn Chủ nghĩa xã hội
Trắc nghiệm CHƯƠNG 5 môn Chủ nghĩa xã hội
 
Giới Thiệu Về Kabala | Hành Trình Thấu Hiểu Bản Thân | Kabala.vn
Giới Thiệu Về Kabala | Hành Trình Thấu Hiểu Bản Thân | Kabala.vnGiới Thiệu Về Kabala | Hành Trình Thấu Hiểu Bản Thân | Kabala.vn
Giới Thiệu Về Kabala | Hành Trình Thấu Hiểu Bản Thân | Kabala.vn
 
Bài học phòng cháy chữa cháy - PCCC tại tòa nhà
Bài học phòng cháy chữa cháy - PCCC tại tòa nhàBài học phòng cháy chữa cháy - PCCC tại tòa nhà
Bài học phòng cháy chữa cháy - PCCC tại tòa nhà
 
các nội dung phòng chống xâm hại tình dục ở trẻ em
các nội dung phòng chống xâm hại tình dục ở trẻ emcác nội dung phòng chống xâm hại tình dục ở trẻ em
các nội dung phòng chống xâm hại tình dục ở trẻ em
 
xemsomenh.com-Vòng Thái Tuế và Ý Nghĩa Các Sao Tại Cung Mệnh.pdf
xemsomenh.com-Vòng Thái Tuế và Ý Nghĩa Các Sao Tại Cung Mệnh.pdfxemsomenh.com-Vòng Thái Tuế và Ý Nghĩa Các Sao Tại Cung Mệnh.pdf
xemsomenh.com-Vòng Thái Tuế và Ý Nghĩa Các Sao Tại Cung Mệnh.pdf
 
30 ĐỀ PHÁT TRIỂN THEO CẤU TRÚC ĐỀ MINH HỌA BGD NGÀY 22-3-2024 KỲ THI TỐT NGHI...
30 ĐỀ PHÁT TRIỂN THEO CẤU TRÚC ĐỀ MINH HỌA BGD NGÀY 22-3-2024 KỲ THI TỐT NGHI...30 ĐỀ PHÁT TRIỂN THEO CẤU TRÚC ĐỀ MINH HỌA BGD NGÀY 22-3-2024 KỲ THI TỐT NGHI...
30 ĐỀ PHÁT TRIỂN THEO CẤU TRÚC ĐỀ MINH HỌA BGD NGÀY 22-3-2024 KỲ THI TỐT NGHI...
 
ĐỀ KIỂM TRA CUỐI KÌ 2 BIÊN SOẠN THEO ĐỊNH HƯỚNG ĐỀ BGD 2025 MÔN TOÁN 10 - CÁN...
ĐỀ KIỂM TRA CUỐI KÌ 2 BIÊN SOẠN THEO ĐỊNH HƯỚNG ĐỀ BGD 2025 MÔN TOÁN 10 - CÁN...ĐỀ KIỂM TRA CUỐI KÌ 2 BIÊN SOẠN THEO ĐỊNH HƯỚNG ĐỀ BGD 2025 MÔN TOÁN 10 - CÁN...
ĐỀ KIỂM TRA CUỐI KÌ 2 BIÊN SOẠN THEO ĐỊNH HƯỚNG ĐỀ BGD 2025 MÔN TOÁN 10 - CÁN...
 
C6. Van de dan toc va ton giao ....pdf . Chu nghia xa hoi
C6. Van de dan toc va ton giao ....pdf . Chu nghia xa hoiC6. Van de dan toc va ton giao ....pdf . Chu nghia xa hoi
C6. Van de dan toc va ton giao ....pdf . Chu nghia xa hoi
 
Access: Chuong III Thiet ke truy van Query.ppt
Access: Chuong III Thiet ke truy van Query.pptAccess: Chuong III Thiet ke truy van Query.ppt
Access: Chuong III Thiet ke truy van Query.ppt
 
30 ĐỀ PHÁT TRIỂN THEO CẤU TRÚC ĐỀ MINH HỌA BGD NGÀY 22-3-2024 KỲ THI TỐT NGHI...
30 ĐỀ PHÁT TRIỂN THEO CẤU TRÚC ĐỀ MINH HỌA BGD NGÀY 22-3-2024 KỲ THI TỐT NGHI...30 ĐỀ PHÁT TRIỂN THEO CẤU TRÚC ĐỀ MINH HỌA BGD NGÀY 22-3-2024 KỲ THI TỐT NGHI...
30 ĐỀ PHÁT TRIỂN THEO CẤU TRÚC ĐỀ MINH HỌA BGD NGÀY 22-3-2024 KỲ THI TỐT NGHI...
 
xemsomenh.com-Vòng Lộc Tồn - Vòng Bác Sĩ và Cách An Trong Vòng Lộc Tồn.pdf
xemsomenh.com-Vòng Lộc Tồn - Vòng Bác Sĩ và Cách An Trong Vòng Lộc Tồn.pdfxemsomenh.com-Vòng Lộc Tồn - Vòng Bác Sĩ và Cách An Trong Vòng Lộc Tồn.pdf
xemsomenh.com-Vòng Lộc Tồn - Vòng Bác Sĩ và Cách An Trong Vòng Lộc Tồn.pdf
 
Giáo trình nhập môn lập trình - Đặng Bình Phương
Giáo trình nhập môn lập trình - Đặng Bình PhươngGiáo trình nhập môn lập trình - Đặng Bình Phương
Giáo trình nhập môn lập trình - Đặng Bình Phương
 

Luận văn: Lý thuyết đồ thị với các bài toán phổ thông, HAY, 9đ

  • 1. ĐẠI HỌC QUỐC GIA HÀ NỘI TRƯỜNG ĐẠI HỌC KHOA HỌC TỰ NHIÊN ———– NGUYỄN THỊ MINH THƯƠNG LÝ THUYẾT ĐỒ THỊ VỚI CÁC BÀI TOÁN PHỔ THÔNG LUẬN VĂN THẠC SĨ KHOA HỌC HÀ NỘI - 2015
  • 2. ĐẠI HỌC QUỐC GIA HÀ NỘI TRƯỜNG ĐẠI HỌC KHOA HỌC TỰ NHIÊN ———– NGUYỄN THỊ MINH THƯƠNG LÝ THUYẾT ĐỒ THỊ VỚI CÁC BÀI TOÁN PHỔ THÔNG Chuyên ngành: Phương pháp toán sơ cấp Mã số: 60.46.01.13 LUẬN VĂN THẠC SĨ KHOA HỌC Người hướng dẫn khoa học: GS.TS Đặng Huy Ruận HÀ NỘI - 2015
  • 3. Mục lục Lời nói đầu 3 1 Đại cương về đồ thị 4 1.1 Định nghĩa đồ thị . . . . . . . . . . . . . . . . . . . . . . 4 1.2 Một số dạng đồ thị đặc biệt . . . . . . . . . . . . . . . . 6 1.3 Bậc của đỉnh đồ thị . . . . . . . . . . . . . . . . . . . . . 8 1.3.1 Bậc của đỉnh . . . . . . . . . . . . . . . . . . . . 8 1.3.2 Nửa bậc . . . . . . . . . . . . . . . . . . . . . . . 8 1.3.3 Một số tính chất . . . . . . . . . . . . . . . . . . 9 1.4 Xích, chu trình, đường, vòng . . . . . . . . . . . . . . . . 13 1.4.1 Xích, chu trình . . . . . . . . . . . . . . . . . . . 13 1.4.2 Đường, vòng . . . . . . . . . . . . . . . . . . . . . 14 1.4.3 Một số tính chất . . . . . . . . . . . . . . . . . . 15 1.5 Đồ thị liên thông . . . . . . . . . . . . . . . . . . . . . . 16 1.5.1 Định nghĩa . . . . . . . . . . . . . . . . . . . . . 16 1.5.2 Tính chất . . . . . . . . . . . . . . . . . . . . . . 17 1.6 Số ổn định trong, số ổn định ngoài . . . . . . . . . . . . 18 1.6.1 Số ổn định trong . . . . . . . . . . . . . . . . . . 18 1.6.2 Số ổn định ngoài . . . . . . . . . . . . . . . . . . 19 1.6.3 Các thuật toán tìm số ổn định trong, số ổn định ngoài. . . . . . . . . . . . . . . . . . . . . . . . . 20 1.7 Nhân của đồ thị và ứng dụng vào trò chơi . . . . . . . . 21 1.7.1 Định nghĩa . . . . . . . . . . . . . . . . . . . . . 21 1.7.2 Tính chất . . . . . . . . . . . . . . . . . . . . . . 22 1.7.3 Trò chơi Nim . . . . . . . . . . . . . . . . . . . . 23 1.7.4 Trò chơi bốc các vật . . . . . . . . . . . . . . . . 24 1.8 Cây và bụi . . . . . . . . . . . . . . . . . . . . . . . . . . 29 1.8.1 Định nghĩa . . . . . . . . . . . . . . . . . . . . . 29 1.8.2 Đặc điểm của cây và bụi . . . . . . . . . . . . . . 30 1
  • 4. 2 Một số bài toán đồ thị cơ bản 33 2.1 Bài toán về đường đi . . . . . . . . . . . . . . . . . . . . 33 2.1.1 Đường đi Euler - Chu trình Euler. . . . . . . . . . 33 2.1.2 Đường đi Hamilton - Chu trình Hamilton. . . . . 40 2.2 Bài toán tô màu đồ thị . . . . . . . . . . . . . . . . . . . 43 2.2.1 Định nghĩa . . . . . . . . . . . . . . . . . . . . . 43 2.2.2 Một số tính chất . . . . . . . . . . . . . . . . . . 43 2.2.3 Thuật toán tô màu đỉnh. . . . . . . . . . . . . . . 53 3 Ứng dụng lý thuyết đồ thị vào giải toán phổ thông. 54 3.1 Quy trình giải bài toán bằng phương pháp đồ thị. . . . . 54 3.1.1 Xây dựng đồ thị G mô tả các quan hệ. . . . . . . 54 3.1.2 Dựa vào các kết quả của lý thuyết đồ thị hoặc lý luận trực tiếp suy ra đáp án của bài toán D. . . . 54 3.2 Bài toán về đỉnh - cạnh của đồ thị. . . . . . . . . . . . . 55 3.3 Bài toán về xích, chu trình, đường, vòng và tính liên thông của đồ thị. . . . . . . . . . . . . . . . . . . . . . . . . . 58 3.4 Bài toán về tô màu đồ thị. . . . . . . . . . . . . . . . . 63 3.5 Bài toán liên quan đến số ổn định trong, số ổn định ngoài. 74 3.6 Bài toán liên quan đến đường đi. . . . . . . . . . . . . . 76 3.6.1 Bài toán tìm đường đi trong mê cung . . . . . . . 76 3.6.2 Bài toán liên quan đến đường và chu trình Euler . 80 3.6.3 Bài toán liên quan đến đường và chu trình Hamilton 82 3.7 Bài toán liên quan đến cây. . . . . . . . . . . . . . . . . 84 Kết luận 89 Tài liệu tham khảo 90 2
  • 5. LỜI NÓI ĐẦU Lý thuyết đồ thị là một trong những ngành khoa học ra đời khá sớm. Lý thuyết đồ thị giúp mô tả hình học và giải quyết nhiều bài toán thực tế phức tạp. Khái niệm lý thuyết đồ thị được nhiều nhà khoa học độc lập nghiên cứu và có nhiều đóng góp trong lĩnh vực toán học ứng dụng. Năm 2001, Bộ Giáo Dục và Đào Tạo có quy định các chuyên đề bồi dưỡng học sinh giỏi thống nhất trên toàn quốc, trong đó có chuyên đề lý thuyết đồ thị. Như vậy, việc học chuyên đề Lý Thuyết Đồ Thị đối với học sinh khá và giỏi đang là nhu cầu thực tế trong dạy học toán ở phổ thông. Tuy nhiên, việc dạy học chuyên đề này còn tồn tại một số khó khăn vì những lý do khác nhau. Một trong các lý do đó là sự mới mẻ, độc đáo và khó của chủ đề kiến thức này. Luận văn "Lý thuyết đồ thị với các bài toán phổ thông" đưa đến sự sáng tạo trong cách nhìn nhận bài toán và lập luận cách giải dưới con mắt của lý thuyết đồ thị. Ngoài phần mở đầu và kết luận luận văn gồm 3 chương: Chương 1 Đại cương về đồ thị. Chương 2 Một số bài toán đồ thị cơ bản. Chương 3 Ứng dụng lý thuyết đồ thị vào giải toán phổ thông. Luận văn được hoàn thành dưới sự hướng dẫn, giúp đỡ tận tình của GS.TS Đặng Huy Ruận, tác giả xin bày tỏ sự kính trọng và lòng biết ơn sâu sắc tới thầy. Tác giả cũng xin gửi lời cảm ơn chân thành đến Ban giám hiệu cùng các thầy cô giáo khoa Toán - Cơ - Tin, Trường Đại học Khoa Học Tự Nhiên - Đại Học Quốc Gia Hà Nội đã tạo điều kiện, dạy bảo và dìu dắt tác giả trong những năm học vừa qua. Xin chân thành cảm ơn sự giúp đỡ của bạn bè, người thân trong thời gian học tập và làm luận văn. Do khả năng nhận thức của bản thân tác giả, luận văn còn nhiều hạn chế, thiếu sót. Tác giả kính mong các ý kiến chỉ bảo của quý thầy cô cùng sự đóng góp của các bạn đọc. Tác giả xin chân thành cảm ơn! Hà Nội, tháng 6 năm 2015 3
  • 6. Chương 1 Đại cương về đồ thị 1.1 Định nghĩa đồ thị Tập hợp X = ∅ các đối tượng và bộ E các cặp sắp thứ tự và không sắp thứ tự các phần tử của X được gọi là một đồ thị, đồng thời được ký hiệu bằng G(X, E) (hoặc G = (X, E) hoặc G(X)). Hình 1.1: Ví dụ về mô hình đồ thị Các phần tử của X được gọi là các đỉnh. Cặp đỉnh không sắp thứ tự được gọi là cạnh, cặp đỉnh sắp thứ tự được gọi là cạnh có hướng hay cung. Đồ thị chỉ chứa các cạnh được gọi là đồ thị vô hướng, còn đồ thị chỉ chứa các cung được gọi là đồ thị có hướng. Nếu đồ thị chứa cả cạnh lẫn cung thì nó được họi là đồ thị hỗn hợp hay đồ thị hỗn tạp. Một cặp đỉnh có thể được nối với nhau bằng hai hoặc nhiều hơn hai cạnh (hai hoặc nhiều hơn hai cung cùng một hướng). Các cạnh (cung) này được gọi là các cạnh (cung) bội. Một cung (hay một cạnh) có thể bắt đầu và kết thúc tại cùng một đỉnh. Cung (cạnh) loại này được gọi là khuyên hay nút. Cặp đỉnh x,y được nối với nhau bằng cạnh (cung) a và a được gọi là cạnh (cung) thuộc đỉnh x, đỉnh y. 4
  • 7. Nếu cung b xuất phát từ đỉnh u và đi vào đỉnh v thì u được gọi là đỉnh đầu, v được gọi là đỉnh cuối của cung b. Cặp đỉnh x, y được gọi là hai đỉnh kề nhau nếu x = y và là hai đầu của cùng một cạnh hay một cung. Đối với mọi đỉnh x dùng D(x) để chỉ tập đỉnh, mà mỗi đỉnh này được nối với x bằng ít nhất một cạnh; D+ (x) để chỉ tập đỉnh mà mỗi đỉnh này từ x có cung đi tới; D− (x) để chỉ tập đỉnh mà mỗi đỉnh này có cung đi tới x. Hai cạnh (cung) a,b được gọi là kề nhau, nếu: i) Chúng khác nhau. ii) Chúng có đỉnh chung (nếu a, b là cung, thì không phụ thuộc vào đỉnh chung đó là đỉnh đầu hay đỉnh cuối của cung a, đỉnh đầu hay đỉnh cuối của cung b). Ví dụ 1.1. Cho đồ thị hỗn hợp có khuyên G(X, E) với tập đỉnh X = {x1, x2, x3, x4, x5, x6, x7}, tập cạnh và cung E = {x1, x2; x2, x3; x4, x6; x5, x6; x3, x3; x1, x6; x5, x5} = {a1 a2 a3 a4 a5 b1 b2}, trong đó a1, a2, a3, a4, a5 là các cạnh; b1, b2 là các cung. Hình 1.2 5
  • 8. 1.2 Một số dạng đồ thị đặc biệt Trong những trường hợp không cần phân biệt giữa cạnh và cung ta quy ước dùng cạnh thay cho cả cung. Đồ thị G = (X, E) không có khuyên và mỗi cặp đỉnh được nối với nhau bằng không quá một cạnh, được gọi là đồ thị đơn hay đơn đồ thị và thông thường được gọi là đồ thị. Đồ thị G = (X, E) không có khuyên và có ít nhất một cặp đỉnh được nối với nhau bằng từ hai cạnh trở lên được gọi là đa đồ thị. Đồ thị G = (X, E) được gọi là vô hướng nếu các cạnh trong E là không định hướng. Đồ thị G = (X, E) được gọi là có hướng nếu các cạnh trong E là có định hướng. Hình 1.3 Đồ thị vô hướng (có hướng) G = (X, E) được gọi là đồ thị đầy đủ nếu mỗi cặp đỉnh được nối với nhau bằng đúng một cạnh (một cung với chiều tùy ý). Hình 1.4: Đồ thị đầy đủ Đa đồ thị vô hướng (có hướng) G = (X, E) được gọi là đồ thị k-đầy đủ nếu mỗi cặp đỉnh được nối với nhau bằng đúng k cạnh (k cung với 6
  • 9. chiều tùy ý). Đồ thị (đa đồ thị) G = (X, E) được gọi là đồ thị (đa đồ thị) hai mảng nếu tập đỉnh X của nó được phân thành hai tập con rời nhau X1, X2 (X1 X2 = X và X1 X2 = ∅) và mỗi cạnh đều có một đầu thuộc X1 còn đầu kia thuộc X2.Khi đó G = (X, E) còn được ký hiệu bằng G = (X1, X2, E). Hình 1.5: Đồ thị hai mảng Đồ thị (đa đồ thị) G = (X, E) được gọi là đồ thị (đa đồ thị) phẳng, nếu nó có ít nhất một dạng biểu diễn hình học trải trên một mặt phẳng nào đó, mà các cạnh của đồ thị chỉ cắt nhau ở đỉnh. Đồ thị (đa đồ thị) G = (X, E) được gọi là hữu hạn, nếu số đỉnh của nó hữu hạn, tức tập X có lực lượng hữu hạn. Đồ thị (đa đồ thị) G = (X, E) được gọi là vô hạn, nếu số đỉnh của nó là vô hạn. Đồ thị (đa đồ thị) với số cạnh thuộc mỗi đỉnh đều hữu hạn được gọi là đồ thị (đa đồ thị) hữu hạn địa phương. Một đồ thị hay đa đồ thị hữu hạn thì nó cũng hữu hạn địa phương. Cho Y ⊆ X, Y = ∅; H ⊆ E, F = E ∩ (Y × Y ) và V = (X × X)/E. Đồ thị G1(Y, F) được gọi là đồ thị con, còn G2(X, H) là đồ thị bộ phận của đồ thị G(X, E). Đồ thị G (X, V ) được gọi là đồ thị bù của đồ thị G(X, E). Đồ thị có hướng G(X, E) được gọi là đồ thị đối xứng nếu ∀x, y ∈ X, (x, y) ∈ E ⇒ (y, x) ∈ E Trong đồ thị đối xứng tùy ý, hai đỉnh kề nhau luôn luôn được nối bằng hai cung ngược chiều nhau. Để đơn giản, trong trường hợp này người ta quy ước thay hai cung nói trên bằng một cạnh nối giữa x và y. Đồ thị có hướng G(X, E) được gọi là đồ thị phản đối xứng nếu ∀x, y ∈ X, (x, y) ∈ E ⇒ (y, x) /∈ E 7
  • 10. 1.3 Bậc của đỉnh đồ thị 1.3.1 Bậc của đỉnh Giả sử G = (X, E) là một đồ thị hay đa đồ thị có hướng hoặc không có hướng. Số cạnh và cung thuộc đỉnh x được gọi là bậc của đỉnh x và ký hiệu bằng m(x). Đỉnh có bậc bằng 0 được gọi là đỉnh biệt lập. Đỉnh có bậc bằng 1 được gọi là đỉnh treo. Cạnh (cung) có ít nhất một đầu là đỉnh treo được gọi là cạnh (cung) treo. Hình 1.6 Ví dụ 1.2. Trong hình 1.6 ta có: m(1) = 2, m(2) = 2, m(3) = 3, m(4) = 3, m(5) = 3, m(6) = 1, m(7) = 0 Đỉnh 6 là đỉnh treo, đỉnh 7 là đỉnh cô lập, g là cạnh treo. 1.3.2 Nửa bậc Giả sử G = (X, E) là một đồ thị hay đa đồ thị có hướng. Số cung đi vào đỉnh x được gọi là nửa bậc vào của đỉnh x và ký hiệu bằng m (x) hoặc m− (x). Số cung đi ra khỏi đỉnh x được gọi là nửa bậc ra của đỉnh x và ký hiệu bằng m (x) hoặc m+ (x). Ký hiệu tập cung đi vào đỉnh x bằng E− (x), còn tập cung ra khỏi đỉnh x bằng E+ (x). 8
  • 11. Hình 1.7 Ví dụ 1.3. Trong hình 1.7 ta có: m (1) = 1, m (2) = 2, m (3) = 2, m (4) = 0, m (5) = 1, m (6) = 1; m (1) = 1, m (2) = 1, m (3) = 1, m (4) = 1, m (5) = 1, m (6) = 2; E− (4) = {∅}, E+ (4) = {g}; E− (6) = {f}, E+ (6) = {e, d}. 1.3.3 Một số tính chất Định lí 1.3.1. Trong đồ thị hay đa đồ thị tùy ý, tổng số bậc của tất cả các đỉnh bao giờ cũng gấp đôi số cạnh. Chứng minh. Thật vậy, khi tính bậc của các đỉnh mỗi cạnh vô hướng hặc có hướng đều được tính mỗi đầu đúng một lần, do đó tổng số bậc của tất cả các đỉnh bao giờ cũng gấp đôi số cạnh. Định lí 1.3.2. Trong đồ thị hay đa đồ thị tùy ý, số đỉnh bậc lẻ luôn luôn là số chẵn. Chứng minh. Giả sử đồ thị (đa đồ thị) G = (X, E) có n đỉnh, m cạnh X = {x1, x2, ..., xk, xk+1, ..., xn−1, xn}, Các đỉnh x1, x2, ..., xk bậc lẻ và xk+1, ..., xn−1, xn bậc chẵn. 9
  • 12. Theo định lý 1.1 có đẳng thức: m(x1) + m(x2) + ... + m(xk) A + m(xk+1) + ... + m(xn−1) + m(xn) B = 2m Vì B là tổng của các số chẵn nên B là số chẵn. Do đó, A = 2m − B phải là số chẵn. Số chẵn A là tổng của k số lẻ, nên k phải chẵn. Bởi vậy, số đỉnh bậc lẻ trong đồ thị (đa đồ thị) bất kỳ phải là một số chẵn. Định lí 1.3.3. Trong một đồ thị với n đỉnh (n ≥ 2) có ít nhất hai đỉnh cùng bậc. Chứng minh. Giả sử G = (X, E) là đồ thị tùy ý với |X| = n ≥ 2. Xét hai khả năng sau: 1) Nếu đồ thị có đỉnh bậc 0 thì trong đồ thị không có đỉnh nào kề với đỉnh này, nên mỗi đỉnh của đồ thị có bậc là một trong n − 1 số nguyên: 0, 1, 2, ..., n − 3, n − 2. 2) Nếu đồ thị có đỉnh bậc n − 1 thì đồ thị không có đỉnh bậc 0. Bởi vậy, bậc của mỗi đỉnh thuộc đồ thị là một trong n − 1 số nguyên: 1, 2, ..., n − 2, n − 1. Từ kết quả trên ta nhận thấy, đồ thị G = (X, E) với n đỉnh (n ≥ 2), nhưng chỉ có không quá n − 1 loại bậc. Do đó, phải có ít nhất hai đỉnh cùng bậc. Khẳng định được chứng minh. Định lí 1.3.4. Nếu đồ thị với n đỉnh (n ≥ 2) có đúng hai đỉnh cùng bậc, thì hai đỉnh này không thể đồng thời có bậc 0 hoặc bậc n − 1. Chứng minh. Giả sử x,y là hai đỉnh cùng bậc của đồ thị G = (X, E) và đều có bậc 0 hoặc bậc n − 1. Loại x, y và tất cả các cạnh thuộc chúng khỏi đồ thị G, ta được đồ thị con G1 có n − 2 đỉnh. Theo định lý 1.3 trong G1 có hai đỉnh cùng bậc, chẳng hạn u,v. 1) Nếu x, y cùng bậc 0, thì u,v trong G không kề với x,y nên u,v trong G đồng thời là hai đỉnh cùng bậc. Như vậy, đồ thị G phải có ít nhất hai cặp đỉnh cùng bậc. 10
  • 13. 2) Nếu x, y đều bậc n − 1. Khi đó, mỗi đỉnh u, v đều kề đồng thời với x, y nên trong đồ thị G các đỉnh u, v cũng cùng bậc. Như vậy, đồ thị G phải có ít nhất hai cặp đỉnh cùng bậc. Cả hai trường hợp có thể đều dẫn tới mâu thuẫn với tính chất: Đồ thị G có duy nhất một cặp đỉnh cùng bậc, nên x, y không thể cùng bậc 0 hặc cùng bậc n − 1 . Khẳng định được chứng minh. Định lí 1.3.5. Số đỉnh bậc n − 1 trong đồ thị G với n đỉnh (n ≥ 4), mà bốn đỉnh tùy ý có ít nhất một đỉnh kề với ba đỉnh còn lại, không nhỏ hơn n − 3. Chứng minh. 1) Nếu G là đồ thị đầy đủ, thì khẳng định là hiển nhiên. 2) Nếu G có cặp đỉnh duy nhất không kề nhau. Khi đó, trong G có n − 2 đỉnh bậc n − 1 3) Nếu G có hai cặp đỉnh không kề nhau, thì chúng phải có đỉnh chung. Thật vậy, giải sử A, B; I, D là hai cặp đỉnh không kề nhau. Nếu hai cặp đỉnh này không có đỉnh chung, thì trong 4 đỉnh A, B, I, D không có đỉnh nào kề với ba đỉnh còn lại. Như vậy, mâu thuẫn với giả thiết, nên hai cặp đỉnh A, B; I, D phải có hai đỉnh trùng nhau, chẳng hạn B ≡ I. Lấy đỉnh C tùy ý khác với A, B, D. Trong bộ bốn A, B, C, D đỉnh C là đỉnh kề với cả ba đỉnh A, B, D. Loại D ra khỏi bộ bốn và thay vào đó là đỉnh E tùy ý khác với A, B, C, D. Trong bộ bốn A, B, C, E hoặc C hoặc E phải kề với ba đỉnh còn lại. Nếu E kề với ba đỉnh còn lại, thì E cũng kề với C. Do đó C kề với tất cả ba đỉnh A, B, E. Do E là đỉnh tùy ý trong n − 4 đỉnh còn lại (khác với A, B, C) nên C có bậc n − 1 C là đỉnh tùy ý trong n − 3 đỉnh bậc n − 1 Khẳng định được chứng minh. Định lí 1.3.6. Với mọi số tự nhiên n (n > 2) luôn luôn tồn tại đồ thị n đỉnh, mà ba đỉnh tùy ý của đồ thị đều không cùng bậc. Chứng minh. 1) Với n = 3 đồ thị G3 gồm một đỉnh bậc 0 và hai đỉnh bậc 1. 11
  • 14. 2) Giả sử khẳng định đúng với đồ thị Gn có n đỉnh. Đồ thị Gn+1 có n + 1 đỉnh được xây dựng như sau: a. Nếu Gn có đỉnh bậc n − 1, thì không có đỉnh bậc 0, nên ta ghép vào Gn đỉnh x bậc 0 và được đồ thị Gn+1 gồm n + 1 đỉnh. Việc ghép thêm đỉnh x vẫn bảo toàn tính chất của Gn (tức là, ba đỉnh bất kỳ đều không cùng bậc). Mặt khác, đồ thị Gn không có đỉnh bậc 0, nên trong Gn+1 ba đỉnh bất kỳ đều không cùng bậc. b. Nếu Gn không có đỉnh bậc n − 1. Khi đó, tất cả các đỉnh của Gn đều có bậc không vượt quá n−2. Thêm vào Gn đỉnh x (không thuộc Gn) và nối x với từng đỉnh thuộc Gn bằng một cạnh được đồ thị Gn+1 gồm n + 1 đỉnh. Đỉnh x có bậc n, còn bậc của mỗi đỉnh thuộc Gn trong Gn+1 được tăng lên một đơn vị, nhưng đều không vượt quá n−1 và trong bậc mới ba đỉnh bất kỳ của Gn vẫn không cùng bậc. Khẳng định được chứng minh. Định lí 1.3.7. Trong đồ thị G = (X, E) với ít nhất kn + 1 đỉnh, mỗi đỉnh có bậc không nhỏ hơn (k − 1)n + 1 luôn tồn tại đồ thị con đầy đủ gồm k + 1 đỉnh. Chứng minh. Ta sẽ chứng minh định lý bằng phương pháp quy nạp theo k. 1) Với k = 1 khẳng định hiển nhiên đúng. 2) Với k = 2 có thể làm chặt chẽ hơn giả thiết. Nếu đồ thị 2n + 1 đỉnh mà mỗi đỉnh có bậc không nhỏ hơn n, thì nó có đồ thị con 3 đỉnh đầy đủ. Thật vậy, xét đỉnh x tùy ý, còn y là một trong các đỉnh kề với x. Tổng số đỉnh kề với x và y không nhỏ hơn 2n, nhưng số đỉnh khác x và y chỉ là 2n − 1. Bởi vậy, phải có ít nhất một đỉnh z được tính hai lần. Khi đó, x, y, z tạo thành một đồ thị con đầy đủ ba đỉnh. 3) Giả sử khẳng định trên đúng với k. Cần suy ra tính đúng đắn của khẳng định đối với k + 1. Theo giả thiết, thông đồ thị G gồm (k + 1)n + 1 đỉnh, số đỉnh kề với đỉnh x tùy ý không nhỏ hơn kn + 1, nên số đỉnh không kề với x sẽ không vượt quá n. Bởi vậy, mỗi đỉnh y kề với x thì nó kề với nhiều nhất n đỉnh không kề với đỉnh x. Do đó, đỉnh y phải kề với ít nhất kn + 1 − n = (k − 1)n + 1 đỉnh kề với đỉnh x. Xét đồ thị con G1 gồm các đỉnh kề với x. Đồ thị con G1 có ít nhất kn + 1 đỉnh và mỗi đỉnh của nó kề với ít nhất (k − 1)n + 1 đỉnh thuộc G1, nên theo giả thiết quy nạp, 12
  • 15. trong G1 có đồ thị con đầy đủ G2 gồm k + 1 đỉnh. Vì đỉnh x kề với từng đỉnh thuộc G2, nên đỉnh x kết hợp với các đỉnh thuộc G2 lập thành một đồ thị con đầy đủ gồm k + 2 đỉnh thong đồ thị G. Khẳng định được chứng minh. 1.4 Xích, chu trình, đường, vòng 1.4.1 Xích, chu trình Giả sử G(X, E) là một đồ thị hay đa đồ thị vô hướng: Dãy α các đỉnh của G(X, E): α = [x1, x2, ..., xi, xi+1, ..., xn−1, xn] được gọi là một xích hay một dây chuyền, nếu ∀i(1 ≤ i ≤ n − 1) cặp đỉnh xi, xi+1 kề nhau. Tổng số vị trí của tất cả các cạnh xuất hiện trong xích α được gọi là độ dài của xích α, ký hiệu |α|. Các đỉnh x1, xn được gọi là hai đỉnh đầu của xích α. Để chỉ rõ đỉnh đầu và đỉnh cuối ta còn ký hiệu α bằng α[x1, xn]. Một xích có hai đầu trùng nhau được gọi là một chu trình. Xích (chu trình) α được gọi là xích (chu trình) đơn (sơ cấp hay cơ bản), nếu nó đi qua mỗi cạnh (mỗi đỉnh) không quá một lần. Ví dụ 1.4. Cho đồ thị Hình 1.8 13
  • 16. α1 = [5, 1, 4, 2, 1] là một dây chuyền không sơ cấp. α2 = [1, 2, 3, 4] là một dây chuyền sơ cấp. α3 = [1, 5, 1] và α4 = [1, 2, 3, 4, 1] là các chu trình đơn và sơ cấp. α5 = [1, 2, 4, 3, 2, 1] là chu trình đơn nhưng không sơ cấp. 1.4.2 Đường, vòng Giả sử G(X, E) là một đồ thị hay đa đồ thị có hướng. Dãy đỉnh β của G(X, E) : β = [x1, x2, ..., xi, xi+1, ..., xm−1, xm] được gọi là một đường hay một đường đi nếu ∀i(1 ≤ i ≤ m − 1), đỉnh xi là đỉnh đầu, còn đỉnh xi+1 là đỉnh cuối của một cung nào đó. Tổng số vị trí của tất cả các cung xuất hiện trong β được gọi là đồ dài của đường β, ký hiệu: |β|. Đỉnh x1 được gọi là đỉnh đầu còn xm là đỉnh cuối của đường β. Người ta còn nói rằng, đường β xuất phát từ đỉnh x1 và đi tới xm. Đường β còn được ký hiệu bằng β[x1, xm]. Một đường có đỉnh đầu và đỉnh cuối trùng nhau được gọi là một vòng. Đường (vòng) β được gọi là đường (vòng) đơn (sơ cấp hay cơ bản), nếu nó đi qua mỗi cạnh (mỗi đỉnh) không quá một lần. Ví dụ 1.5. Cho đồ thị có hướng (hình 1.9): β1 = [1, 2, 4, 3, 5, 1] là một vòng đơn và sơ cấp. β2 = [1, 4, 3, 5] là một đường đơn và sơ cấp. β3 = [1, 4, 2, 5] không phải là đường. β4 = [1, 2, 4, 3, 2, 5] là một đường đơn nhưng không sơ cấp. β5 = [1, 4, 2, 5, 1, 2, 5] không phải là đường đơn và cũng không phải là đường sơ cấp. β6 = [1, 2, 4, 3, 2, 5, 1] là một vòng đơn nhưng không là vòng sơ cấp. 14
  • 17. Hình 1.9 1.4.3 Một số tính chất Định lí 1.4.1. Trong một đồ thị vô hướng với n đỉnh (n ≥ 3) và các đỉnh đều có bậc không nhỏ hơn 2 luôn tồn tại chu trình sơ cấp. Chứng minh. Vì đồ thị hữu hạn, mà xích sơ cấp qua từng đỉnh không quá một lần nên số xích sơ cấp trong đồ thị G = (X, E) là một số hữu hạn. Bởi vậy, luôn xác định được xích sơ cấp có độ dài cực đại trong đồ thị G = (X, E). Giả sử α = [x1, x2, ..., xk−1, xk] là một trong những xích sơ cấp có độ dài cực đại. Do bậc của mỗi đỉnh không nhỏ hơn 2, nên x1 phải kề với một đỉnh y nào đó khác với x2. Nếu y /∈ α, tức là y = xi, (3 ≤ i ≤ k), thì xích sơ cấp α = [y, x1, x2, ..., xk−1, xk] có độ dài |α | = |α| + 1 > |α|. Ta đã đi tới mâu thuẫn với tính chất độ dài cực đại của α. Bởi vậy, y ∈ α tức y ≡ xi, (3 ≤ i ≤ k), nên trong đồ thị G = (X, E) có chu trình sơ cấp β = [x1, x2, ..., xi, x1] Khẳng định được chứng minh. Định lí 1.4.2. Trong một đồ thị vô hướng với n đỉnh (n ≥ 4) và các đỉnh đều có bậc không nhỏ hơn 3 luôn tồn tại chu trình sơ cấp độ dài chẵn. 15
  • 18. Chứng minh. Giả sử α là một trong những xích sơ cấp có độ dài cực đại α = [x1, x2, ..., xi−1, xi, xi+1, ..., xj−1, xj, xj+1, ..., xk−1, xk] Vì α có độ dài cực đại, mà bậc của x1 không nhỏ hơn 3, nên x1 phải kề với hai đỉnh khác thuộc α: xi, (3 ≤ i ≤ k), xj, (3 ≤ j ≤ k). Khi đó có hai chu trình sơ cấp: α1 = [x1, x2, ..., xi−1, xi, x1] α2 = [x1, x2, ..., xi−1, xi, xi+1, ..., xj−1, xj, x1] 1) Nếu một trong hai chu trình α1, α2 có độ dài chẵn thì khẳng định được chứng minh. 2) Nếu ngược lại, cả hai chu trình α1, α2 đề có độ dài lẻ. Khi đó xích: α3 = [x1, x2, ..., xi−1, xi] có độ dài chẵn, còn xích α4 = [xi, xi+1, ..., xj−1, xj, x1] có độ dài lẻ, nên chu trình α5 = [x1, xi, xi+1, ..., xj−1, xj, x1] có độ dài chẵn. Khẳng định được chứng minh. 1.5 Đồ thị liên thông 1.5.1 Định nghĩa Hai đỉnh x, y của đồ thị G = (X, E) được gọi là cặp đỉnh liên thông nếu hoặc giữa x và y có ít nhất một xích nối với nhau , hoặc tồn tại ít nhất một đường đi từ x sang y hoặc từ y sang x. Hình 1.10 Trong hình 1.10 cặp đỉnh x,y là liên thông Đồ thị vô hướng G = (X, E) được gọi là đồ thị liên thông, nếu mọi cặp đỉnh của nó đều liên thông. Đồ thị có hướng G = (X, E) được gọi là đồ thị liên thông mạnh, nếu mọi cặp đỉnh của nó đều liên thông. 16
  • 19. Giả sử a là đỉnh bất kỳ của đồ thị G = (X, E). Dùng Ca để ký hiệu tập con của các đỉnh thuộc G, gồm đỉnh a và tất cả các đỉnh liên thông với a trong đồ thị G. Đồ thị con của G có tập đỉnh Ca được gọi là một thành phần liên thông của đồ thị G Ví dụ 1.6. Cho đồ thị G có bốn thành phần liên thông: Các đồ thị con G1, G3, G4 là liên thông Đồ thị con G2 liên thông mạnh Hình 1.11 1.5.2 Tính chất Định lí 1.5.1. Đồ thị vô hướng tùy ý với n đỉnh (n ≥ 2), mà tổng bậc của hai đỉnh tùy ý không nhỏ hơn n là đồ thị liên thông. Chứng minh. Giả sử đồ thị vô hướng G(X, E) có n đỉnh (n ≥ 2). Với mọi cặp đỉnh a, b của đồ thị ta đều có: m(a) + m(b) ≥ n (1) Nhưng a, b không liên thông. Khi đó trong đồ thị G tồn tại hai thành phần liên thông: G1 chứa a và có n1 đỉnh, còn G2 chứa b và có n2 đỉnh. 17
  • 20. Vì G1, G2 là các thành phần liên thông của G nên n1 + n2 ≤ n Khi đó m(a) + m(b) ≤ (n1 − 1) + (n2 − 1) = n1 + n2 − 2 ≤ n − 2 < n (2) Như vậy, (1) và (2) mâu thuẫn nhau, nên đồ thị G phải liên thông. Khẳng định được chứng minh. Từ định lý trên suy ra hệ quả sau: Hệ quả 1.5.1. Đồ thị, mà bậc của mỗi đỉnh không nhỏ hơn nửa số đỉnh, là đồ thị liên thông. Định lí 1.5.2. Nếu đồ thị có đúng hai đỉnh bậc lẻ, thì hai đỉnh này phải liên thông. Chứng minh. Giả sử đồ thị G(X, E) có đúng hai đỉnh bậc lẻ và hai đỉnh đó là a và b. Giả sử a, b không liên thông với nhau. Khi đó chúng phải thuộc hai thành phần liên thông khác nhau của đồ thị G. Chẳng hạn G1 chứa đỉnh a, còn G2 chứa đỉnh b. Bậc của đỉnh a trong G1 cũng chính là bậc của a trong G, nên trong G1 đỉnh a vẫn có bậc lẻ. Điều này mâu thuẫn với định lý 1.2. Bởi vậy a, b phải liên thông. Khẳng định được chứng minh. 1.6 Số ổn định trong, số ổn định ngoài 1.6.1 Số ổn định trong 1. Tập ổn định trong Giả sử có đồ thị G(X, E). Tập con A ⊆ X các đỉnh của đồ thị G được gọi là tập ổn định trong, nếu mọi cặp đỉnh thuộc A đều không kề nhau (không có cạnh hoặc cung nối với nhau). Tập con B ⊆ X các đỉnh của đồ thị G được gọi là tập ổn định trong cực đại, nếu B là tập ổn định trong và nếu thêm vào B một đỉnh tùy ý x ∈ X, thì tập con nhận được B ∪ {x} sẽ không ổn định trong. 2. Tính chất Nếu A là tập ổn định trong, thì mọi tập con của A đều phải ổn định trong. 18
  • 21. 3. Số ổn định trong Số phần tử của một trong những tập ổn định trong có lực lượng lớn nhất được gọi là số ổn định trong của đồ thị G, đồng thời được ký hiệu bằng α(G). 1.6.2 Số ổn định ngoài 1. Tập ổn định ngoài Giả sử có đồ thị G(X, E). Tập con B ⊆ X các đỉnh của đồ thị G được gọi là tập ổn định ngoài, nếu với mọi đỉnh x thuộc tập XB đều tồn tại đỉnh y ∈ B, để hoặc từ x sang y có cung hoặc cặp đỉnh x, y được nối bằng một cạnh. 2. Tính chất Nếu B là tập ổn định ngoài, thì mọi tập chứa B đều ổn định ngoài. 3. Số ổn định ngoài Số phần tử của một trong những tập ổn định ngoài có lực lượng bé nhất được gọi là số ổn định ngoài của đồ thị G, đồng thời được ký hiệu bằng β(G). Ví dụ 1.7. Cho đồ thị G như hình 1.12. Hãy tìm tất cả các tập ổn định trong,số ổn định trong và số ổn định ngoài của đồ thị G. Các tập ổn định trong Hình 1.12 19
  • 22. - Vì đồ thị không có khuyên, nên mỗi đỉnh lập thành một tập ổn định trong: M1 = {x1}, M2 = {x2}, M3 = {x3}, M4 = {x4}, M5 = {x5}, M6 = {x6}, M7 = {x7}. - Các tập ổn định trong gồm 2 đỉnh: M8 = {x1, x3}, M9 = {x1, x4}, M10 = {x1, x6}, M11 = {x2, x5}, M12 = {x2, x7}, M13 = {x3, x5}, M14 = {x3, x6}, M15 = {x3, x7}, M16 = {x4, x7}, M17 = {x5, x6}. - Các tập ổn định trong gồm 3 đỉnh: M18 = {x1, x3, x6}, M19 = {x3, x6, x5}. Số ổn định trong Ta nhận thấy M18 = {x1, x3, x6}, M19 = {x3, x6, x5} là các tập ổn định trong có lực lượng lớn nhất, nên số phần tử của nó chính là số ổn định trong. Tức là, α(G) = |{x1, x3, x6}| = |{x3, x6, x5}| = 3. Các tập ổn định ngoài - Tập ổn định ngoài một đỉnh không có. - Tập ổn định ngoài gồm 2 đỉnh, chẳng hạn N1 = {x1, x2}, N2 = {x1, x4}. Số ổn định ngoài β(G) = |{x1, x2}| = 2. 1.6.3 Các thuật toán tìm số ổn định trong, số ổn định ngoài. 1.6.3.1. Thuật toán tìm số ổn định trong. - Bước 1: Tìm các tập ổn định trong có 2 phần tử bằng cách xét tất cả tổ hợp chập 2 của n phần tử (n số các đỉnh), kiểm tra những tập nào mà phần tử tương ứng không kề nhau thì tập đó là ổn định trong; - Bước 2: Duyệt từng tập có 2 phần tử và bổ sung thêm phẩn tử thứ 3 và kiểm tra từng cặp như bước 1, tập nào thỏa mãn ta được tập ổn định trong 3 phần tử. ......... - Bước k: Giả sử ta đã tìm được m tập con ổn định trong có k+1 phần tử + Duyệt từng tập và bổ sung vào các tập đó thêm 1 phần tử. + Nếu không có tập nào bổ sung được nữa thì dừng. 20
  • 23. 1.6.3.2. Thuật toán tìm số ổn định ngoài. Xét G(X, E) với X = {x1, x2, ..., xn} - Bước 1: Xác định các tập ∆(xi), i = 1, 2, ..., n với ∆(xi) = {xi và các đỉnh kề với xi} - Bước 2: Từ các tập ∆(x1), ∆(x2), ..., ∆(xn) ta tìm tập B = {xk1, xk2, ..., xkm} sao cho ∆(xk1) ∪ ∆(xk2) ∪ ... ∪ ∆(xkm) = X. Khi đó B là tập ổn định ngoài cực tiểu. 1.7 Nhân của đồ thị và ứng dụng vào trò chơi 1.7.1 Định nghĩa Giả sử có đồ thị G(X, U). Tập đỉnh S ⊆ X được gọi là nhân của đồ thị G, nếu nó vừa là tập ổn định trong lại vừa là tập ổn định ngoài. Do S là tập ổn định trong nên nó không chứa khuyên. Mặt khác S ổn định ngoài nên nó phải chứa tất cả các đỉnh biệt lập và các đỉnh không có cung đi ra. Ví dụ 1.8. Cho hai đồ thị như hình 1.13. Đồ thị hình 1.13(a) có hai nhân là {1, 4} và {2, 3} Đồ thị hình 1.13(b) không có nhân vì các tập ổn định trong chỉ gồm 1 đỉnh, còn các tập ổn định ngoài phải gồm ít nhất hai đỉnh. Hình 1.13 21
  • 24. 1.7.2 Tính chất Định lí 1.7.1. Nếu đồ thị G(X, U) có số ổn định trong nhỏ hơn số ổn định ngoài thì nó không có nhân. Chứng minh. Giả sử trong đồ thị G(X, U), α(G) < β(G) (1) nhưng lại có nhân và S là một trong những nhân của đồ thị G. Khi đó, theo định nghĩa: α(G) = max{|A|A ∈ H(G)} ≥ |S| ≥ min{|B|B ∈ K(G)} = β(G) (2) trong đó, H(G) là tập gồm các tập ổn định trong, còn K(G) là tập gồm các tập ổn định ngoài của đồ thị G. So sánh (1) và (2) đi tới mâu thuẫn, nên G không thể có nhân. Định lý được chứng minh. Định lí 1.7.2. Nếu S là nhân của đồ thị G(X, U), thì nó cũng là tập ổn định trong cực đại. Chứng minh. Giả sử S là nhân của đồ thị G(X, U) và x là đỉnh tùy ý không thuộc S. Xét tập S ∪ {x}. Vì S là nhân và x /∈ S, nên ∃y ∈ S, để x, y được nối bằng một cạnh hoặc từ x sang y có cung. Bởi vậy, tập S ∪ {x} không ổn định trong, nên S là tập ổn định trong cực đại. Định lý được chứng minh. Định lí 1.7.3. Trong đồ thị vô hướng không có khuyên mọi tập ổn định trong cực đại đều là nhân. Chứng minh. Giả sử B là một tập ổn định trong cực đại của đồ thị vô hướng G(X, E). Khi đó ∀x ∈ (XB) đều ∃y ∈ B để x, y có cạnh nối với nhau, nên B đồng thời là tập ổn định ngoài. Định lý được chứng minh. Giả sử có đồ thị G(X, E) và A ⊆ X. Dùng D(A) để ký hiệu tập đỉnh, mà mỗi đỉnh này có cạnh nối với ít nhất một đỉnh thuộc A. Còn D+ (A) là tập đỉnh mà mỗi đỉnh này có ít nhất một đỉnh thuộc A có cung đi tới nó. D− (A) là tập đỉnh mà mỗi đỉnh này có cung đi tới ít nhất một đỉnh thuộc A. 22
  • 25. Hệ quả 1.7.1. Mọi đồ thị vô hướng không có khuyên đều có nhân. Chứng minh. Thật vậy, giả sử đồ thị vô hướng G(X, E) là đồ thị vô hướng không có khuyên. Khi đó mỗi đỉnh đều lập thành một tập ổn định trong. Xuất phát từ đỉnh tùy ý x0. Đặt S0 = {x0}, sau đó chọn đỉnh tùy ý x1 /∈ D(x0) và đặt S1 = S0 ∪ {x1} = {x0, x1}. Tiếp theo, chọn đỉnh tùy ý x2 /∈ D(S1)... Vì đồ thị G hữu hạn, nên sớm hay muộn quá trình phải dừng lại, tức là có số tự nhiên n để D(Sn) = XSn. Với cách chọn này Sn là tập ổn định trong cực đại, nên theo định lý 1.16, nó là nhân của đồ thị G. Hệ quả được chứng minh. 1.7.3 Trò chơi Nim Giữa hai đấu thủ, được ký hiệu là A và B, có một đồ thị G(X, E) cho phép xác định một trò chơi nào đó. Trong trò chơi này mỗi thế là một đỉnh của đồ thị. Đỉnh khởi đầu x0 được chọn bằng cách gắp thăm và các đấu thủ lần lượt đi: Đầu tiên đấu thủ A chọn đỉnh x1 trong tập D(x0) ∪ D+ (x0); sau đó đấu thủ B chọn đỉnh x2 trong tập D(x1) ∪ D+ (x1); tiếp theo đấu thủ A chọn đỉnh x3 trong tập D(x2) ∪ D+ (x2),...Nếu một trong hai đấu thủ chọn được đỉnh xk, mà D(xk) ∪ D+ (xk) = ∅, thì ván đó kết thúc. Đấu thủ nào chọn được đỉnh cuối cùng thì thắng cuộc và đấu thủ kia thua cuộc. Để kỷ niệm trò tiêu khiển quen thuộc mà Nim đã tổng quát hóa, người ta gọi trò chơi mô tả ở trên là trò chơi Nim và dùng ngay đồ thị G(X, E) xác định nó để ký hiệu cho trò chơi này. Định lí 1.7.4. Nếu đồ thị G(X, E) có nhân S và nếu một đấu thủ đã chọn được một đỉnh trong nhân S, thì việc chọn này bảo đảm cho đấu thủ đó thắng hoặc hòa. Chứng minh. Thật vậy, nếu đấu thủ A chọn được đỉnh x1 ∈ S, thì hoặc D(x1) ∪ D+ (x1) = ∅ tức là A thắng cuộc, hoặc D(x1) ∪ D+ (x1) = ∅, thì đối phương B buộc phải chọn đỉnh x2 ∈ (X − S). Khi đó đến lượt mình đấu thủ A lại có thể chọn x3 ∈ S và cứ như thế mãi. 23
  • 26. Vì đồ thị G có hữu hạn đỉnh, nên đến một lúc nào đó một trong hai đấu thủ bằng cách chọn dược đỉnh xk ∈ S, mà D(xk) ∪ D+ (xk) = ∅. Theo cách chọn trên, thì đấu thủ A đến lượt chọn xk, nên A chính là người thắng cuộc. Định lý được chứng minh. 1.7.4 Trò chơi bốc các vật 1. Trò chơi Trên bàn có một đống gồm m vật. Hai đấu thủ A, B thực hiện trò chơi bốc các vật theo nguyên tắc: 1) Người đi đầu xác định ngẫu nhiên (bằng gắp thăm hoặc gieo đòng tiền). 2) Với k(1 ≤ k < m) mỗi người đến lượt phải bốc ít nhất một vật và không được bốc quá k vật. 3) Người bốc được vật cuối cùng thắng(thua)cuộc. Khi tham gia cuộc chơi mỗi người đều phải tìm cách thực hiện để chiến thắng. Mỗi bước chơi đều có vai trò quyết định của nó. Song bước một có ý nghĩa quyết định hơn cả. Bởi thế người đi đầu có phần chủ động hơn. Nếu người đi đầu có thuật toán chơi đúng, thì nhất định chiến thắng. Bởi vậy cần đưa ra thuật toán chơi đúng cho người đi đầu. 2. Thuật toán chơi dựa vào nhân đồ thị a.Trường hợp bốc được vật cuối cùng thắng cuộc 1)Xây dựng đồ thị xác định trò chơi: Cần xác định đỉnh và cung của đồ thị tương ứng với số lượng vật có thể có là 0, 1, 2, ..., i, i + 1, ..., m − 1, m. Dùng ngay số lượng vật để ghi trên các điểm tương ứng. i) Đối với mỗi đỉnh x ≥ k có cung đi tới từng đỉnh thuộc tập D+ (x) = {x − 1, x − 2, ..., x − k + 1, x − k} . ii) Đối với mỗi đỉnh y(1 ≤ y < k) có cung đi tới từng đỉnh thuộc tập D+ (y) = {0, 1, 2, ..., y − 1} . 2)Xác định nhân của đồ thị: 24
  • 27. Vì từng cặp đỉnh thuộc tập M = {0, k + 1, 2(k + 1), ..., m k + 1 (k + 1)} không kề nhau và mỗi đỉnh i ∈ M đều có cung đi tới đỉnh i k + 1 (k + 1), nên tập M là nhân của đồ thị G. 3) Thuật toán: Giả sử A là người được đi đầu. Khi đó A bốc m − m k + 1 (k + 1) vật, tức đi theo cung m, m k + 1 (k + 1) để đến đỉnh m k + 1 (k + 1) ∈ M. Đến lượt mình, giả sử B bốc t(1 ≤ t ≤ k) vật. Tiếp theo A bốc k+1−t vật, tức xuất phát từ đỉnh m k + 1 (k + 1) − t đi theo cung m k + 1 (k + 1) − t, m k + 1 k để đến đỉnh m k + 1 k ∈ M. Cứ tiếp tục như vậy đấu thủ B chỉ có thể đạt được đỉnh ngoài nhân M, còn đấu thủ A lần lượt đạt được các đỉnh m k + 1 (k + 1), m k + 1 − 1 (k + 1), ... Cuối cùng A đạt được đỉnh 0, tức là A bốc được vật cuối cùng nên thắng cuộc. 25
  • 28. Ví dụ 1.9. ([4]Lý thuyết đồ thị và các bài toán không mẫu mực) Có 13 viên bi. Hai em A và B thực hiện trò chơi bốc bi theo nguyên tắc sau: 1) Mỗi người đến lượt chỉ có thể bốc từ 1 đến 3 viên. 2) Ai bốc được viên bi cuối cùng thì thắng cuộc. Nếu A được đi đầu, thì A phải có cách bốc bi như thế nào để đảm bảo thắng cuộc? Giải 1)Xây dựng đồ thị G xác định trò chơi (hình 1.14). Nhân của đồ thị M = {0, 4, 8, 12} Hình 1.14 2)Thuật toán: Để đảm bảo chiến thắng thì A phải bốc 1 viên bi để số bi còn lại là 12 (12 ∈ M). Tiếp sau, nếu B bốc s(1 ≤ s ≤ 3) viên bi, thì A phải bốc 4 − s viên bi để số bi còn lại là 8 (8 ∈ M),nếu B bốc t(1 ≤ t ≤ 3) viên bi, thì A phải bốc 4−t viên bi để số bi còn lại là 4 (4 ∈ M). Tiếp theo B chỉ có thể bốc từ 1 đến 3 viên bi, nên số bi còn lại sẽ không ít hơn 1 và không quá 3 viên nên A được bốc cả số bi còn lại (tức là đạt được đỉnh 0 thuộc M). Do đó A thắng. b.Trường hợp bốc được vật cuối cùng thua cuộc 1)Xây dựng đồ thị xác định trò chơi: Cần xác định đỉnh và cung của đồ thị tương ứng với số lượng vật có thể có là 0, 1, 2, ..., i, i + 1, ..., m − 1, m. Dùng ngay số lượng vật để ghi 26
  • 29. trên các điểm tương ứng. i) Đối với mỗi đỉnh x ≥ k có cung đi tới từng đỉnh thuộc tập D+ (x) = {x − 1, x − 2, ..., x − k + 1, x − k}. ii) Đối với mỗi đỉnh y(1 ≤ y < k) có cung đi tới từng đỉnh thuộc tập D+ (y) = {0, 1, 2, ..., y − 1}. 2)Xác định nhân của đồ thị: Vì từng cặp đỉnh thuộc tập N = {1, k + 2, 2(k + 1) + 1, ..., m k + 1 (k + 1) + 1} không kề nhau và mỗi đỉnh i /∈ N đều có cung đi tới đỉnh i k + 1 (k + 1) + 1, nên tập N là nhân của đồ thị con không chứa đỉnh 0. 3) Thuật toán: Giả sử A là người được đi đầu. Khi đó A bốc m − m k + 1 (k + 1) − 1 vật, tức đi theo cung m, m k + 1 (k + 1) + 1 để đến đỉnh m k + 1 (k + 1) + 1 ∈ N. Đến lượt mình, giả sử B bốc t(1 ≤ t ≤ k) vật. Tiếp theo A bốc k+1−t vật, tức xuất phát từ đỉnh m k + 1 (k + 1) + 1 − t đi theo cung m k + 1 (k + 1) + 1 − t, m k + 1 − 1 (k + 1) + 1 27
  • 30. để đến đỉnh m k + 1 − 1 (k + 1) + 1 ∈ N. Cứ tiếp tục như vậy đấu thủ B chỉ có thể đạt được đỉnh ngoài nhân N, còn đấu thủ A lần lượt đạt được các đỉnh ( m k + 1 (k + 1) + 1 − t, m k + 1 − 1 (k + 1) + 1, ... Cuối cùng A đạt được đỉnh 1 ∈ N, tức là sau khi đấu thủ A bốc lần cuối trên bàn còn đúng 1 vật. Khi đó, B phải bốc vật cuối cùng, nên thua cuộc. Ví dụ 1.10. ([4]Lý thuyết đồ thị và các bài toán không mẫu mực) Có 14 que diêm trên bàn. Hai em A và B thực hiện trò chơi bốc diêm theo nguyên tắc sau: 1) Mỗi người đến lượt chỉ có thể bốc từ 1 đến 3 que diêm. 2) Ai bốc phải que diêm cuối cùng thì thua cuộc. Nếu A được đi đầu, thì A phải có cách bốc diêm như thế nào để đảm bảo thắng cuộc? Giải Hình 1.15 1)Xây dựng đồ thị G xác định trò chơi (hình 1.15). Nhân của đồ thị N = {1, 5, 9, 13} 2)Thuật toán: 28
  • 31. Để đảm bảo chiến thắng thì A phải bốc 1 que diêm để số diêm còn lại là 13 (13 ∈ N). Tiếp sau, nếu B bốc s(1 ≤ s ≤ 3) que diêm, thì A phải bốc 4 − s que diêm để số diêm còn lại là 9 que (9 ∈ N), nếu B bốc t(1 ≤ t ≤ 3) que diêm, thì A phải bốc 4 − t que để số diêm còn lại là 5 (5 ∈ N). Nếu B bốc u(1 ≤ u ≤ 3) que diêm, thì A phải bốc 4 − u que diêm để số diêm còn lại là 1 (1 ∈ N). Chỉ còn lại một que diêm nên B phải bốc nốt. Do đó A thắng. 1.8 Cây và bụi 1.8.1 Định nghĩa Một đồ thị vô hướng liên thông, không có chu trình và có ít nhất hai đỉnh được gọi là một cây (hình 1.16) Hình 1.16 Đồ thị hữu hạn có hướng G = (X, U) là một bụi gốc x1 ∈ X, nếu nó có ít nhất hai đỉnh và thỏa mãn ba điều kiện sau: 1. Mỗi đỉnh khác x1 là điểm cuối của một cung duy nhất. 2. Đỉnh x1 không là điểm cuối của bất kỳ một cung nào. 3. Đồ thị G = (X, U) không có vòng. (Hình 1.17) Hình 1.17 29
  • 32. 1.8.2 Đặc điểm của cây và bụi Định lí 1.8.1. Giả sử H là một đồ thị vô hướng với n đỉnh n > 1. Để đặc trưng cho một cây thì sáu tính chất sau đây là tương đương: 1. H liên thông và không có chu trình; 2. H không có chu trình và có n − 1 cạnh; 3. H liên thông và có n − 1 cạnh; 4. H không có chu trình và nếu thêm một cạnh nối giữa hai đỉnh bất kì không kề nhau thì đồ thị nhận được H’ có một chu trình (và chỉ một mà thôi); 5. H liên thông và khi bớt một cạnh bất kì thì đồ thị mất tính liên thông; 6. Mọi cặp đỉnh của H đều được nối với nhau bằng một xích và chỉ một xích mà thôi. Chứng minh. Định lý được chứng minh theo phương pháp vòng tròn. Ký hiệu số cạnh của đồ thị H bằng m. Dùng p để ký hiệu số thành phần liên thông và V(H) là số chu trình của đồ thị H. (1) ⇒ (2): Theo tính chất (1): p = 1, V (H) = m − n + 1 = 0, nên m = n − 1. (2) ⇒ (3): Theo tính chất (2): m = n − 1, V (H) = 0, nên: V (H) = m − n + p = n − 1 − n + p = 0 ⇒ p = 1 Bởi vậy, H liên thông và có n − 1 cạnh. (3) ⇒ (4): Theo tính chất (3): p = 1, m = n − 1, nên V (H) = m − n + p = n − 1 − n + 1 = 0 Tức là H không có chu trình; ngoài ra, nếu thêm vào một cạnh nối giữa hai đỉnh không kề nhau, thì đồ thị H’ nhận được sẽ có chu số: V (H ) = m + 1 − n + 1 = n − 1 + 1 − n + 1 = 1 nên đồ thị H’ có một chu trình và chỉ một mà thôi. (4) ⇒ (5): Lấy hai đỉnh bất kỳ x, y của đồ thị H. Theo tính chất (4): nếu thêm vào cạnh (x,y) thì đồ thị mới nhận được H’ có chu trình, điều đó chứng tỏ giữa x, y đã có xích nối với nhau, tức H đã liên thông. Giả sử bớt đi một cạnh nào đó, chẳng hạn (u,v) mà đồ thị nhận được vẫn liên thông. Điều này chứng tỏ trong đồ thị H giữa các đỉnh u, v 30
  • 33. ngoài cạnh (u,v) còn một xích nữa nối giữa chúng, tức là trong H có ít nhất một chu trình đi qua u, v. Ta đi tới mâu thuẫn với tính chất (4): Đồ thị H không có chu trình. Bởi vậy, nếu bớt đi một cạnh tùy ý thì đồ thị nhận được từ H sẽ không liên thông. (5) ⇒ (6): Giả sử trong H tồn tại cặp đỉnh nào đó, chẳng hạn x, y được nối với nhau từ hai xích trở lên. Khi đó, nếu ta bỏ đi một cạnh nào đó thuộc một trong hai xích này, thì xích còn lại vẫn bảo đảm cho x, y liên thông. Như vậy, ta đã đi tới mâu thuẫn với tính chất (5). Do đó, mọi cặp đỉnh của H đều được nối với nhau bằng một xích và chỉ một mà thôi. (6) ⇒ (1): Giả sử H không liên thông. Khi đó có ít nhất một cặp đỉnh không có xích nối với nhau, nên mâu thuẫn với tính chất (6). Giả sử H có chu trình. Khi đó có ít nhất một cặp đỉnh nằm trên chu trình này được nối với nhau bằng ít nhất hai xích. Như vậy, ta cũng đi đến mâu thuẫn với tính chất (6). Bởi vậy, đồ thị H có tính chất (1). Định lý được chứng minh. Định lí 1.8.2. Một cây có ít nhất hai đỉnh treo. Chứng minh. Giả sử cây H chỉ có không quá một đỉnh treo. Ta tưởng tượng có một khách bộ hành đi theo đồ thị đó, xuất phát từ một đỉnh tùy ý (trong trường hợp đồ thị không có đỉnh treo) hay từ đỉnh treo (trong trường hợp đồ thị có một đỉnh treo): Nếu hành khách tự cấm mình không đi qua một cạnh hai lần, khi đó không thể gặp một đỉnh hai lần (do đồ thị không có chu trình). Mặt khác, khi tới một đỉnh người đó luôn luôn có thể đi ra bằng một cạnh mới (vì mỗi đỉnh khác đỉnh xuất phát đều có ít nhất hai cạnh). Như vậy khách bộ hành sẽ đi mãi không bao giờ dừng lại. Đó là điều không thể xảy ra, vì đồ thị H có hữu hạn đỉnh. Vậy đồ thị H không thể có ít hơn hai đỉnh treo. Định lý được chứng minh. Định lí 1.8.3. Mọi bụi khi bỏ định hướng các cạnh đều trở thành cây. Chứng minh. Giả sử bụi H = (X, U) có gốc là x1 và đồ thị vô hướng G = (X, E) nhận được từ bụi H sau khi bỏ định hướng các cung. 1. Đồ thị G liên thông: Do điều kiện (1) mỗi đỉnh x = x1 đều có đường từ x1 đi tới. Thật vậy, giả sử x = x1 và từ x1 không có đường đi tới x. 31
  • 34. Nếu x là đỉnh biệt lập, thì nó không thể là đỉnh cuối của một cung nào, còn nếu x không phải là đỉnh biệt lập, thì phải có đỉnh y là điểm xuất phát của một đường đi tới x. Nhưng do từ x1 không có đường đi tới x nên y = x1; mà nó cũng không là điểm cuối của bất kỳ cung nào. Như vậy, ta đã đi tới mâu thuẫn với điều kiện (1). Do đó mọi đỉnh x = x1 từ x1 đều có đường đi tới nó, nên trong G mọi đỉnh x đều có xích nối với đỉnh x1. Bởi vậy, G liên thông. 2. Đồ thị G không có chu trình. Thật vậy, giả sử G có chu trình, thì trong H dãy cung tương ứng với các cạnh thuộc chu trình này sẽ hoặc lập thành một vòng hoặc có ít nhất hai cung có chung điểm cuối. Như vậy, ta đã đi tới mâu thuẫn với điều kiện (1) hoặc điều kiện (3). Nên đồ thị G không có chu trình và liên thông. Do đó G là một cây. Định lý được chứng minh. 32
  • 35. Chương 2 Một số bài toán đồ thị cơ bản 2.1 Bài toán về đường đi 2.1.1 Đường đi Euler - Chu trình Euler. 2.1.1.1. Bài toán mở đầu : Bài toán 7 cây cầu ở K¨onigsberg: Thành phố K¨onigsberg thuộc Phổ (bây giờ gọi là Kaliningrad thuộc Cộng hòa Liên bang Nga) được chia thành bốn vùng bằng các nhánh sông Pregel. Các vùng này gồm 2 vùng bên bờ sông, đảo Kneiphof và một miền nằm giữa 2 nhánh của sông Pregel. Vào thế kỷ thứ XVIII, người ta đã xây 7 cây cầu nối các vùng lại với nhau như sơ đồ sau: Hình 2.1 Vào chủ nhật, người dân ở đây thường đi bộ dọc theo các vùng trong thành phố. Họ tự hỏi “Liệu có thể xuất phát tại một địa điểm nào đó trong thành phố, đi qua tất cả 7 cây cầu, qua mỗi cây một lần, rồi trở 33
  • 36. về điểm xuất phát được không?” Nhà toán học Thụy Sĩ Leonard Euler đã nghiên cứu giải bài toán này. Lời giải của ông được công bố năm 1736. Bài toán này có thể được coi là một trong những ứng dụng đầu tiên của lý thuyết đồ thị. Ta có thể xây dựng đồ thị G = (V, E) mô tả bài toán như sau: + Đỉnh: Lấy các điểm trên mặt phẳng hay trong không gian tương ứng với các vùng đất trong sơ đồ. Đối tượng của bài toán ở đây là một vùng đất trong sơ đồ. Vậy, mỗi đỉnh biểu diễn cho một vùng đất. Đồ thị G sẽ có 4 đỉnh A, B, C, D tương ứng với 4 vùng đất. + Cạnh: Trong đồ thị G các đỉnh được nối với nhau bằng các cạnh mà mỗi cạnh đại diện cho một chiếc cầu nối giữa hai vùng đất. Đồ thị G sẽ có 7 cạnh tương ứng với 7 chiếc cầu nối giữa các vùng đất trong sơ đồ. Euler đã nghiên cứu bài toán này, mô hình nó bằng một đa đồ thị, bốn vùng được biểu diễn bằng 4 đỉnh, các cầu là các cạnh như đồ thị sau: Hình 2.2 Bài toán tìm đường đi qua tất cả các cầu mỗi cầu không quá một lần có thể được phát biểu lại bằng ngôn ngữ đồ thị như sau: “Trong đa đồ thị G = (V, E) tồn tại hay không một chu trình đơn có chứa tất cả 7 cạnh?” 2.1.1.2. Định nghĩa 1. Chu trình Euler (Đồ thị Euler) Cho G = (V, E) là một đa đồ thị liên thông. Chu trình đơn chứa tất cả các cạnh của đồ thị G được gọi là chu trình Euler. Đồ thị có chứa một chu trình Euler được gọi là đồ thị Euler. 34
  • 37. 2. Đường đi Euler Cho G = (V, E) là một đa đồ thị liên thông. Đường đi Euler trong G là đường đi đơn chứa tất cả các cạnh của đồ thị G. Hình 2.3 Ví dụ 2.1. . Đồ thị ở hình 2.3a có chu trình Euler: a, b, e, d, c, e, a. Đồ thị ở hình 2.3b không có chu trình Euler nhưng có đường đi Euler: a, c, d, a, b, e, d, b. Đồ thị ở hình 2.3c không có chu trình Euler và đường đi Euler. 2.1.1.3. Chu trình và đường đi Euler trong đồ thị vô hướng Khi giải bài toán cầu K¨onigsberg, Euler đã phát hiện ra các tiêu chuẩn để khẳng định một đa đồ thị có chu trình và đường đi Euler hay không? 1. Định lý về chu trình Euler Một đa đồ thị liên thông G =(V, E) có chu trình Euler khi và chỉ khi mỗi đỉnh của nó đều có bậc chẵn. Chứng minh. (⇒) Ta sẽ chứng minh: Nếu đồ thị G có chu trình Euler, thì mọi đỉnh của G đều có bậc chẵn. Thật vậy, trước tiên giả sử G có chu trình Euler bắt đầu từ đỉnh a và tiếp tục bằng cạnh liên thuộc với a, chẳng hạn ab, khi đó cạnh ab góp một đơn vị vào deg (a). Mỗi lần khi chu trình đi qua mỗi đỉnh, nó tăng thêm 2 đơn vị cho bậc của đỉnh đó. Vì chu trình đi vào mỗi đỉnh bằng một cạnh liên thuộc và rời khỏi đỉnh này bằng một cạnh liên thuộc khác. Cuối cùng chu trình kết thúc ở đỉnh mà nó xuất 35
  • 38. phát. Do đó nó tăng thêm một đơn vị vào deg (a). Nghĩa là deg (a) phải là một số chẵn. Đỉnh khác a cũng có bậc chẵn vì chu trình góp 2 đơn vị vào bậc của nó mỗi lần đi qua đỉnh này. Vậy, mỗi đỉnh của G đều có bậc chẵn. (⇐) Giả sử mọi đỉnh của đa đồ thị liên thông G đều có bậc chẵn. Ta sẽ chứng minh tồn tại một chu trình Euler trong G. Thật vậy, ta sẽ xây dựng một chu trình đơn bắt đầu từ đỉnh a tùy ý của G. Gọi x0 = a; Trước tiên, ta chọn tùy ý cạnh x0x1, x1x2, ..., xn−1xn càng dài càng tốt. Ví dụ, trong đồ thị G sau: Hình 2.4 Ta bắt đầu tại a và chọn các cạnh liên tiếp ab, bc, cf, fa. Đường đi mà ta chọn sẽ kết thúc vì đồ thị có hữu hạn đỉnh. Đường đi này bắt đầu tại a và kết thúc tại a. Điều này luôn xảy ra vì mỗi lần đường đi qua một đỉnh bậc chẵn, nó chỉ dùng một cạnh để vào đỉnh này và còn ít nhất một đỉnh để ra khỏi đỉnh này. Đường đi vừa nói trên có thể đi qua tất cả các cạnh hoặc có thể không. Nếu tất cả các cạnh được sử dụng, thì ta nhận được chu trình Euler. Nếu không, ta gọi H là đồ thị con nhận được từ G bằng cách xóa các cạnh đã dùng và các đỉnh không liên thuộc với các cạnh còn lại. Chẳng hạn với đồ thị trên, khi xóa đi chu trình a, b, c, f, a khỏi đồ thị này, ta nhận được đồ thị con H. Vì G là liên thông nên H có ít nhất một đỉnh chung với chu trình vừa bị xóa. Gọi w là đỉnh đó (trong ví dụ trên là đỉnh c). Mỗi đỉnh của H có bậc chẵn vì tất cả các đỉnh của G có bậc chẵn và với mỗi đỉnh ta đã xóa đi từng cặp liên thuộc để tạo ra H. Lưu ý rằng H có thể không liên thông. Bắt đầu từ đỉnh w ta xây dựng một đường đi đơn bằng cách chọn càng nhiều đỉnh càng tốt như ta đã làm trong G. Đường này phải kết thúc tại w. Ví dụ trong đồ thị H nêu trên ta có chu trình con: c, d, e, c. Sau đó, ta tạo một chu trình trong G bằng cách ghép chu trình trong H 36
  • 39. với chu trình ban đầu trong G (điều này thực hiện được vì 2 chu trình có chung đỉnh w). Tiếp tục quá trình này cho đến khi tất cả các đỉnh được sử dụng. Quá trình này phải kết thúc vì đồ thị có hữu hạn đỉnh. Do đó, ta đã xây dựng được một chu trình Euler. Định lý được chứng minh. Bây giờ, trở lại bài toán 7 cây cầu ở K¨onigsberg: Ta đã thấy đa đồ thị G = (V, E) biểu diễn các cầu ở K¨onigsberg có 4 đỉnh bậc lẻ. Do đó, theo định lý trên sẽ không có chu trình Euler trong đa đồ thị này. Điều này cũng có nghĩa là bài toán 7 cây cầu ở K¨onigsberg không có lời giải. Hay nói cách khác, không thể từ một địa điểm nào đó đi qua cả 7 cây cầu, qua mỗi cầu đúng một lần rồi trở về được nơi xuất phát. 2. Thuật toán Fleury tìm chu trình Euler Để tìm một chu trình Euler trong một đa đồ thị có tất cả các đỉnh đều bậc chẵn, ta có thể sử dụng thuật toán Fleury như sau: Xuất phát từ một đỉnh bất kỳ của đồ thị G và tuân theo hai qui tắc sau: • Qui tắc 1: Mỗi khi đi qua một cạnh nào thì xóa cạnh đó đi, sau đó xóa đỉnh cô lập (nếu có). • Qui tắc 2: Không bao giờ đi qua một cầu (cạnh duy nhất nối giữa hai thành phần liên thông), trừ khi không còn cách đi nào khác để di chuyển. Ví dụ 2.2. Tìm một chu trình Euler trong đồ thị G (Hình 2.5) Hình 2.5 Xuất phát từ đỉnh A, giả sử ta chọn cạnh AB, BC, CF. Sau đó xóa 3 cạnh này, ta được đồ thị G1 37
  • 40. Đến đây, ta không thể chọn FG vì GF là một cầu, cho nên ta chọn FD, DC, CE, EF. Đến đây, ta được đồ thị G2. Ta không còn cách chọn nào khác, nên phải chọn FG, GH, HB, BG, GA. Như vậy, ta có chu trình Euler sau: A, B, C, F, D, C, E, F, G, H, B, G, A. 3. Định lý về đường đi Euler Đa đồ thị liên thông G = (V, E) có đường đi Euler, nhưng không có chu trình Euler khi và chỉ khi nó có đúng hai đỉnh bậc lẻ. Chứng minh. (⇒) Giả sử đa đồ thị G có đường đi Euler, nhưng không có chu trình Euler. Ta sẽ chứng minh G có đúng 2 đỉnh bậc lẻ. Thật vậy, giả sử G có đường đi Euler từ a đến b, nhưng không có chu trình Euler. Cạnh đầu tiên của đường đi góp một đơn vị vào deg (a). Sau đó mỗi lần đường đi qua a lại góp thêm 2 đơn vị vào deg (a). Chắc chắn đường đi không thể kết thúc tại a, cho nên deg(a) là số lẻ. Cạnh cuối cùng của đường đi góp một đơn vị vào deg(b) và mỗi lần đi qua b, nó cũng góp 2 đơn vị vào deg(b). Do đó, deg(b) là số lẻ. Các đỉnh trung gian đều có bậc chẵn vì mỗi lần đường đi tới rồi lại đi nên tăng hai đơn vị cho bậc của đỉnh đó. Vậy, đồ thị đã cho có đúng 2 đỉnh bậc lẻ. (⇐) Giả sử đa đồ thị liên thông G có đúng 2 đỉnh bậc lẻ. Ta sẽ chứng minh G có đường đi Euler. Thật vậy, giả sử G có đúng 2 đỉnh bậc lẻ là a và b. Thêm vào cạnh nối giữa a và b ta được đồ thị G’.Khi đó trong đồ thị mới G = G ∪ ab , tất cả các đỉnh đều có bậc chẵn. Do đó, theo định lý Euler, tồn tại một chu trình Euler trong G’. Trong chu trình này bỏ cạnh ab, ta được đường đi Euler trong G. Như vậy, trong một đa đồ thị liên thông có 2 đỉnh bậc lẻ thì đường đi Euler trong đồ thị đó sẽ nhận 2 đỉnh bậc lẻ làm các điểm đầu mút. Định lý được chứng minh. 38
  • 41. 2.1.1.4. Chu trình và đường đi Euler đối với đồ thị có hướng 1. Định lý về chu trình Euler Đồ thị G = (V, E) có chứa một chu trình Euler khi và chỉ khi G là liên thông và mọi đỉnh của nó đều có bậc chẵn. Ví dụ 2.3. Trên hình 2.6 ta có: Hình 2.6 Đồ thị G1 có chu trình Euler: a, b, c, a, d, c, a. Đồ thị G2 không có chu trình Euler. 2. Định lý về đường đi Euler Cho G = (V, E) là một đa đồ thị. G có một đường đi Euler từ A đến B khi và chỉ khi G là liên thông và mọi đỉnh của nó đều có bậc chẵn, chỉ trừ A và B có bậc lẻ. Ví dụ 2.4. Trên hình 2.7 Hình 2.7 Đồ thị G có đường đi Euler: a, b, c, a, d, c. 39
  • 42. 2.1.2 Đường đi Hamilton - Chu trình Hamilton. 2.1.2.1. Trò chơi Hamilton. Năm 1857 W. R. Hamilton đưa ra trò chơi sau đây: Trên mỗi đỉnh trong số 20 đỉnh của khối đa diện ngũ giác đều 12 mặt ghi tên một thành phố trên thế giới Hãy tìm cách đi bằng các cạnh của khối đa diện để qua tất cả các thành phố, mỗi thành phố đúng một lần. Hình 2.8 Để có được đáp án cho trò chơi như hình 2.8 ta cần nghiên cứu lý thuyết về chu trình Hamilton. 2.1.2.2. Định nghĩa. Đường đi trong đồ thị vô hướng G = (V, E) được gọi là đường đi Hamilton nếu nó đi qua tất cả các đỉnh của G và qua mỗi đỉnh đúng một lần. Một chu trình sơ cấp đi qua tất cả các đỉnh của đồ thị G = (V, E) (đi qua mỗi đỉnh đúng một lần) được gọi là chu trình Hamilton. Đồ thị G = (V, E) có chứa chu trình Hamilton được gọi là đồ thị Hamilton. Ví dụ 2.5. Cho hai đồ thị như trên hình 2.9 Đồ thị hình 2.9a có chu trình Hamilton là: a, b, c, d, e, a. Đồ thị hình 2.9b không có chu trình Hamilton vì mọi chu trình chứa mọi đỉnh của đồ thị đều phải đi qua cạnh ab 2 lần. 40
  • 43. Hình 2.9 2.1.2.3. Điều kiện tồn tại chu trình Hamilton. 1. Bổ đề 2.1.2.1. Đồ thị vô hướng n đỉnh liên thông (n ≥ 3), thuần nhất bậc 2 có chu trình Hamilton. Chứng minh. Giả sử đồ thị vô hướng G = (X, E)(|X| = n ≥ 3), liên thông thuần nhất bậc 2. Đồ thị G hữu hạn, nên số xích sơ cấp trong G cũng hữu hạn. Giả sử α = (x1, x2, ..., xk−1, xk) là một trong những xích sơ cấp có độ dài dài nhất (|α|max) Nếu k < n thì tồn tại đỉnh y /∈ α. Do G liên thông nên phải có ít nhất một đỉnh thuộc α kề với đỉnh y, chẳng hạn xi. Nếu i = 2, 3, ..., k − 1, thì m(xi) > 2 nên vô lý. Nếu i = 1 hoặc k, thì xích (y, x2, ..., xk−1, xk) hoặc xích (x1, x2, ..., xk−1, xk, y) có độ dài bằng |α| + 1 > |α|max, nên cũng dẫn đến mâu thuẫn. Do đó k = n, tức ta được xích Hamilton. Vì m(x1) = m(xn) = 2 nên x1 phải kề với xn (không thể kề với xi(3 ≤ i ≤ n−1) vì bậc của mỗi đỉnh này đã bằng 2). Ta được chu trình Hamilton. 2. Bổ đề 2.1.2.2. Đồ thị vô hướng G = (X, E) có chu trình Hamilton khi và chỉ khi nó có một đồ thị bộ phận liên thông và thuần nhất bậc 2. Chứng minh. . 1) Điều kiện cần là hiển nhiên. Bởi vì, nếu đồ thị G có chu trình Hamilton, thì mỗi chu trình Hamilton của G là một đồ thị bộ phận liên thông thuần nhất bậc 2. 41
  • 44. 2) Điều kiện đủ. Giả sử đồ thị G = (X, E) có đồ thị bộ phận G1 = (X, E1) liên thông và thuần nhất bậc 2. Theo bổ đề 2.1.2.1 đồ thị G1 có chu trình Hamilton α. Khi đó, α cũng chính là chu trình Hamilton trong đồ thị G 3. Định lý Rédei Trong đồ thị có hướng đầy đủ luôn luôn tồn tại đường Hamilton. Chứng minh. (Thuật toán tìm đường Hamilton trong đồ thị đầy đủ có hướng.) Giả sử G = (X, E) là đồ thị có hướng đầy đủ nào đó và: α = [x1, x2, ..., xi, xi+1, ...xk−1, xk] là một đường sơ cấp bất kì trong đồ thị G. 1) Nếu α đã đi qua tất cả các đỉnh của G, thì nó là một đường Hamilton của G. 2) Nếu trong G còn có đỉnh nằm ngoài α, thì ta có thể bổ sung dần dần các đỉnh này vào α và các đường sơ cấp nhận được tiếp theo để cuối cùng nhận được đường Hamilton. Thật vậy, giả sử x là đỉnh tùy ý không nằm trên α. a) Nếu cung nối x với x1 xuất phát từ x, thì bổ sung x và đầu của đường α, để được α1, mà |α1| = |α| + 1 α1 = [x, x1, x2, ..., xk−1, xk] b) Nếu tồn tại chỉ số i(1 ≤ i ≤ k − 1) mà từ xi có cung đi tới x và từ x có cung đi tới xi+1 thì ta chen x vào giữa xi và xi+1 để được đường sơ cấp α2 mà |α2| = |α| + 1, α2 = [x1, x2, ..., xi, x, xi+1, ..., xk−1, xk] c) Nếu cả hai khả năng trên đều không xảy ra nghĩa là ∀i(1 ≤ i ≤ k)xi đều có cung đi tới x. Khi đó bổ sung x vào cuối của đường α và được α3 mà |α3| = |α| + 1, α3 = [x1, x2, ..., xk−1, xk, x] Nếu đồ thị G có n đỉnh thì sau n − k bổ sung ta sẽ được đường Hamilton. 42
  • 45. 2.2 Bài toán tô màu đồ thị 2.2.1 Định nghĩa Tô màu đỉnh của một đồ thị là phép gán các màu cho các đỉnh, sao cho hai đỉnh kề nhau bất kỳ có màu khác nhau. Sắc số của đồ thị là số màu ít nhất cần dùng để tô trên các đỉnh của đồ thị, sao cho hai đỉnh kề nhau tùy ý được tô bằng hai màu khác nhau. Ví dụ 2.6. ([4]Lý thuyết đồ thị và các bài toán không mẫu mực) Đồ thị trên hình 2.10 có sắc số bằng 3: Hình 2.10 Sắc lớp là số màu ít nhất cần dùng để tô trên các cạnh của đồ thị, sao cho hai cạnh kề nhau (có đỉnh chung) tùy ý đều có màu khác nhau. 2.2.2 Một số tính chất Định lí 2.2.1. Một chu trình độ dài lẻ luôn có sắc số bằng 3. Chứng minh. Giả sử α là một chu trình độ dài lẻ tùy ý. Khi đó tồn tại số tự nhiên n để |α| = 2n + 1. Ký hiệu các đỉnh của α một cách trực tiếp bằng x1, x2, ..., x2n, x2n+1. Ta sẽ chứng minh khẳng định trên bằng phương pháp quy nạp theo n. 1) Với n = 1, chu trình α gồm 3 đỉnh x1, x1, x3. Do mỗi đỉnh xi, (1 ≤ i ≤ 3) đều kề với hai đỉnh còn lại, nên ta phải dùng đúng 3 màu khác nhau thì mới đủ tô trên mỗi đỉnh một màu, để hai đỉnh kề nhau tùy ý đều có màu khác nhau. 2) Giả sử khẳng định đã đúng với n ≤ k, tức là đối với chu trình α1 tùy ý với độ dài 2n + 1, (1 ≤ n ≤ k) đều có sắc số bằng 3. Cần chỉ ra rằng với n = k + 1 khẳng định vẫn đúng, tức là chu trình α tùy ý 43
  • 46. có độ dài bằng 2(k + 1) + 1 và có tập đỉnh được đánh số liên tiếp là x1, x2, ..., xk, xk+1, ..., x2k+2, x2k+3. Hình 2.11 Nối đỉnh x1 với đỉnh x2k+1 ta được chu trình α1 với độ dài 2k + 1. Theo giả thiết quy nạp, sắc số của α1 bằng 3, đồng thời x1 và x2k+1 có màu khác nhau. Chẳng hạn, x1 được tô màu M1 và x2k+1 được tô bằng màu M2. Khi đó để tô đỉnh x2k+2 ta có thể dùng lại màu M1 và tô đỉnh x2k+3 màu M2. Tức là không phải thêm màu mới. Vậy sắc số của α bằng 3. Khẳng định được chứng minh. Lớp đồ thị có chu trình tam giác cùng màu Để phục vụ cho việc giải quyết một số bài toán nào đó ta cần xét những dãy số đặc biệt và đưa ra các khẳng định thích hợp, chẳng hạn, để xây dựng những lớp đồ thị có chu trình tam giác cùng màu người ta đưa ra các dãy số nguyên dương: a1 = 2, a2 = 5, ..., an+1 = (n + 1)an + 1 u2 = 3, u3 = 6, ..., un+1 = (un − 1)n + 2 và có định lý sau: Định lí 2.2.2. a. Một đồ thị đầy đủ và vô hướng với an + 1 đỉnh, các cạnh được tô bằng n màu luôn có chu trình tam giác cùng màu. b. Một đồ thị đầy đủ vô hướng un+1 đỉnh, các cạnh được tô bằng n màu luôn có chu trình tam giác cùng màu. Chứng minh. 44
  • 47. a. Chứng minh bằng quy nạp theo n 1) Với n = 1 đồ thị đầy đủ tương ứng gồm a1 +1 = 2+1 = 3 đỉnh lập thành một chu trình tam giác. Các cạnh của đồ thị này được tô bằng một màu, nên chu trình tam giác lập nên G1 cùng màu. 2) Giả sử khẳng định đã đúng với n = k, tức là đồ thị đầy đủ bất kỳ Gk gồm ak + 1 đỉnh với các cạnh được tô bằng k màu đã có chu trình tam giác cùng màu. Cần chứng minh rằng khẳng định cũng đúng với n = k + 1. Xét đồ thị đầy đủ tùy ý Gk+1 với ak+1 + 1 đỉnh và các cạnh được tô bằng k + 1 màu, nên xuất phát từ đỉnh P phải có ít nhất ak + 1 cạnh được tô bằng cùng một màu. Giả sử màu này là màu đỏ và các cạnh PA1, PA2, ..., PAak , PAak+1 được tô bằng màu đỏ. Có hai khả năng xảy ra: i) Nếu một trong các cạnh nối giữa các đỉnh Ai, Aj, (1 ≤ i, j ≤ ak +1) được tô màu đỏ, chẳng hạn cạnh A1, A2 màu đỏ. Khi đó chu trình tam giác A1PA2 màu đỏ, nên đồ thị Gk+1 có chu trình tam giác màu đỏ. ii) Ngược lại, nếu không có cạnh nào trong các cạnh Ai, Aj, (1 ≤ i, j ≤ ak + 1) được tô màu đỏ. Khi đó đồ thị con đầy đủ Gk với tập đỉnh A1, A2, ..., Aak , Aak+1 có các cạnh được tô bằng không quá k màu, nên theo giả thiết quy nạp Gk đã có chu trình tam giác cùng màu, do đó Gk+1 có chu trình tam giác cùng màu. Khẳng định được chứng minh. b. Chứng minh tương tự như phần a. Định lí 2.2.3. Đồ thị đầy đủ có un+1 − 1 đỉnh (n ≥ 2) với n màu cạnh (các cạnh được tô bằng n màu), sao cho không có tam giác cùng màu nào, luôn luôn có 5 hình cạnh với các cạnh cùng màu và các đường chéo được tô bằng màu khác. Chứng minh. 1. Với n = 2 đồ thị tương ứng G2 đầy đủ có u3 − 1 = 5 đỉnh và 2 màu cạnh (xanh, đỏ) không có đồ thị con K3 cùng màu. Khi đó G2 có thể biểu diễn ở dạng hình 5 cạnh với cạnh cùng màu đỏ và đường chéo màu xanh. Thật vậy, do G2 đầy đủ nên mỗi đỉnh xuất phát đúng 4 cạnh được tô bằng 2 màu. Chính xác hơn, tại từng đỉnh, mỗi màu được tô trên đúng 2 cạnh. Giả sử ngược lại, tại đỉnh A màu đỏ được tô trên 3 cạnh là AB, AC và AD. Khi đó một trong 3 cạnh BC, BD, CD màu đỏ, đồ thị có tam giác đỏ. Ngược lại cả 3 cạnh đều màu xanh, đồ thị có tam giác xanh. Như vậy mâu thuẫn với giả thiết. 45
  • 48. Giả sử A có các cạnh đỏ là AB và AC (đường liền nét), còn AD và AE là xanh (đường nét đứt). Khi đó cạnh BC phải xanh và ED là đỏ (hình 2.12a). Hai cạnh BE và CE không thể cùng xanh. Giả sử BE đỏ, thì CE xanh. Suy ra CD đỏ và BD xanh. Vậy ta được hình 5 cạnh với cạnh màu đỏ và đường chéo màu xanh (hình 2.12b). Hình 2.12 2. Giả sử khẳng định đúng với n = k. Xét đồ thị Gk+1 đầy đủ với u(k+1)+1 − 1 đỉnh, k + 1 màu cạnh và không có đồ thị con K3 cùng màu. Mỗi đỉnh của Gk+1 xuất phát (u(k+1)+1 − 1)(k + 1) cạnh với k + 1 màu, nên phải có ít nhất uk+1 − 1 cạnh cùng màu. Giả sử tại đỉnh A có uk+1 − 1 cạnh cùng được tô bởi màu m1. Khi đó trong các đỉnh đối của A không có cặp đỉnh nào được nối với nhau bởi cạnh màu m1 (trái lại thì có K3 cùng màu m1). Xét đồ thị con đầy đủ Gk lập nên từ uk+1 − 1 đỉnh đối với A có cạnh chỉ tô bởi k màu (trừ màu m1) và không có K3 cùng màu, nên theo giả thiết quy nạp, trong Gk có hình 5 cạnh với cạnh cùng một màu và đường chéo là các màu khác (tất cả đều không là màu m1). Vậy trong Gk+1 có điều cần khẳng định. Định lí 2.2.4. Đồ thị đầy đủ gồm 6 đỉnh và được tô bằng không quá hai màu cạnh thì luôn có 2 chu trình tam giác cùng màu. Chứng minh. Trước hết ta chứng minh tồn tại tam giác cùng màu thứ nhất. Gọi các đỉnh của đồ thị là A1, A2, A3, A4, A5, A6. Giả sử các cạnh của đồ thị được tô bởi màu xanh hoặc đỏ. Xét 5 cạnh xuất phát từ đỉnh A1. Theo nghuyên lý Dirichlet tồn tại ít nhất 3 cạnh cùng màu, không giảm tính tổng quát nếu ta giả sử các cạnh đó là A1A2, A1A3, A1A4 và có màu đỏ (nét liền). 46
  • 49. Hình 2.13a Nếu tam giác A2A3A4 có các cạnh đều là màu xanh suy ra tồn tại tam giác cùng màu. Nếu tam giác A2A3A4 có ít nhất một cạnh màu đỏ, giả sử cạnh đó là A2A3 suy ra tam giác A1A2A3 là tam giác cùng màu. Chứng minh sự tồn tại của tam giác cùng màu thứ hai. Giả sử tam giác cùng màu thứ nhất là A1A2A3 và có màu đỏ. Xét các khả năng sau: • Trường hợp 1: Các cạnh A1A4, A1A5, A1A6 đều có màu đỏ. Xét tam giác A4A5A6: a. Nếu tam giác A4A5A6 có cả ba cạnh đều màu xanh thì ta có tam giác cùng màu thứ hai. b. Nếu tam giác A4A5A6 có ít nhất một cạnh màu đỏ, chẳng hạn A4A5 thì ta có tam giác cùng màu thứ hai là A1A4A5. • Trường hợp 2: Trong 3 cạnh A1A4, A1A5, A1A6 có đúng hai cạnh màu đỏ và một cạnh màu xanh. Không giảm tổng quát ta giả sử hai cạnh đỏ là A1A4, A1A5 và cạnh có màu xanh là A1A6. Xét tam giác A3A4A5: a. Nếu tam giác có cả 3 cạnh đều màu xanh thì ta có tam giác cùng màu thứ hai. b. Nếu tam giác này có ít nhất một cạnh đỏ, chẳng hạn A4A5 thì ta có tam giác cùng màu thứ hai là A1A4A5. • Trường hợp 3: Trong 3 cạnh A1A4, A1A5, A1A6 có đúng một cạnh màu đỏ và hai cạnh màu xanh. Giả sử cạnh màu đỏ là A1A4 và hai cạnh màu xanh là A1A5, A1A6. a. Nếu A2A4 hoặc A3A4 có màu đỏ hoặc A5A6 có màu xanh thì ta có tam giác cùng màu thứ hai. b. Nếu A2A4, A3A4 màu xanh còn A5A6 màu đỏ. Ta xét tam giác 47
  • 50. Hình 2.13b Hình 2.13c A4A5A6 : -) Nếu A4A5, A4A6 đều có màu đỏ thì ta có tam giác cùng màu thứ hai là A4A5A6. -) Nếu trong hai cạnh A4A5, A4A6 có ít nhất một cạnh có màu xanh. Giả sử A4A5 có màu xanh. Xét tam giác A2A3A5: +, Nếu tam giác này có A3A5, A2A5 cùng màu đỏ thì ta có tam giác cùng màu thứ hai. +) Nếu A3A5 có màu xanh thì ta cũng có tam giác cùng màu thứ hai là A3A4A5. +) Nếu A2A5 có màu xanh thì ta cũng có tam giác cùng màu thứ hai là A2A4A5. • Trường hợp 4: Cả 3 cạnh A1A4, A1A5, A1A6 cùng có màu xanh. Xét tam giác A4A5A6: -) Nếu cả ba cạnh của tam giác đều có màu đỏ thì ta có tam giác cùng màu thứ hai. 48
  • 51. Hình 2.13d -) Nếu trong 3 cạnh của tam giác có ít nhất một cạnh có màu xanh, giả sử cạnh đó là A5A6 thì ta có tam giác cùng màu thứ 2 là A1A5A6 Định lí 2.2.5. Đồ thị đầy đủ G gồm n đỉnh (n ≥ 6) và được tô bằng không quá hai màu cạnh luôn có ít nhất (n − 4) tam giác cùng màu. Chứng minh. Chứng minh bằng phương pháp quy nạp. Khẳng định đúng với n=6. Giả sử định lý đúng với n = k, k ≥ 6. Nghĩa là đồ thị đầy đủ Gk gồm có k đỉnh được tô bởi không quá hai màu cạnh đã có ít nhất (k − 4) tam giác cùng màu. Cần chứng minh đồ thị đầy đủ Gk+1 có k + 1 đỉnh và được tô bởi không quá hai màu cạnh thì có ít nhất (k − 3) tam giác cùng màu. Gọi các đỉnh của đồ thị Gk+1 là A1, A2, A3, ..., Ak+1. Vì Gk+1 có đồ thị con Gk nên luôn có tam giác cùng màu. Không giảm tổng quát ta gọi tam giác cùng màu đó là A1A2A3. Loại đỉnh A1 và các cạnh xuất phát từ nó ra khỏi đồ thị Gk+1 thì ta có đồ thị đầy đủ Gk có ít nhất (k − 4) tam giác cùng màu. Khôi phục lại đỉnh A1 và các cạnh của nó ta có thêm ít nhất một tam giác cùng màu nữa là A1A2A3. Vậy đồ thị đầy đủ Gk+1 có ít nhất k − 4 + 1 = k − 3 tam giác cùng màu. Định lí 2.2.6. Đồ thị đầy đủ G gồm 9 đỉnh được tô bằng hai màu cạnh xanh, đỏ thì luôn có đồ thị con đầy đủ K3 xanh hoặc đồ thị con đầy đủ K4 đỏ (hoặc ngược lại nếu ta đổi hai màu cho nhau) Chứng minh. Gọi các đỉnh của đồ thị là x1, x2, ..., x9. • Trường hợp 1: Đồ thị có đỉnh là đầu mút của ít nhất 4 cạnh màu xanh (nét đứt đoạn). Giả sử đỉnh x1 là đầu mút của ít nhất 4 cạnh màu xanh và các cạnh đó là x1x2, x1x3, x1x4, x1x5. Xét các cạnh nối hai đỉnh 49
  • 52. trong số các đỉnh x2, x3, x4, x5. Nếu có ít nhất một cạnh có màu xanh thì ta có tam giác xanh. Nếu không có cạnh nào có màu xanh, thì ta có tứ giác đỏ (hình 2.14a). Hình 2.14a • Trường hợp 2: Không có đỉnh nào là đầu mút của nhiều hơn 3 cạnh màu xanh. Khi đó có ít nhất một đỉnh là đầu mút của ít hơn 3 cạnh màu xanh. Thật vậy, nếu tất cả các đỉnh là đầu mút của đúng 3 cạnh màu xanh, thì tổng số cạnh có màu xanh là 3.9 2 . Điều này vô lý vì số cạnh phải nguyên. Hình 2.14b Giả sử x1 là mút của không quá 2 cạnh xanh suy ra x1 là mút của ít nhất 6 cạnh đỏ. 50
  • 53. Gọi các cạnh màu đỏ là x1x2, x1x3, x1x4, x1x5, x1x6, x1x7 (hình 2.14b). Xét đồ thị con đầy đủ G1 có 6 đỉnh x2, x3, x4, x5, x6, x7 với hai màu cạnh. Theo định lý 2.2.4 thì G1 có tam giác cùng màu. Nếu tam giác này có màu xanh thì định lý được chứng minh. Nếu tam giác này có màu đỏ thì tam giác này cùng với đỉnh x1 tạo thành tứ giác cùng màu đỏ. Định lí 2.2.7. Đồ thị đầy đủ K14 gồm 14 đỉnh được tô bằng hai màu cạnh xanh, đỏ, thì luôn có tam giác xanh hoặc ngũ giác đỏ (hoặc ngược lại nếu ta đổi hai màu cho nhau.) Chứng minh. Gọi các đỉnh của đồ thị là x1, x2, ..., x14. • Trường hợp 1: Đồ thị có đỉnh là đầu mút của ít nhất 5 cạnh màu xanh (nét đứt đoạn). Giả sử đỉnh x1 là đầu mút của ít nhất 5 cạnh màu xanh và các cạnh đó là x1x2, x1x3, x1x4, x1x5, x1x6. Nếu trong các cạnh có đầu mút là hai trong 5 điểm x2, x3, x4, x5, x6 có cạnh màu xanh. Chẳng hạn x2x5 thì ta có tam giác xanh x1x2x5. Nếu không ta có ngũ giác đỏ x2x3x4x5x6. Hình 2.15 • Trường hợp 2: Không có đỉnh nào là đầu mút của nhiều hơn 4 cạnh màu xanh. Suy ra đỉnh x1 là đầu mút của ít nhất 9 cạnh màu đỏ. Giả sử các cạnh đó là x1x2, x1x3, x1x4, x1x5, x1x6, x1x7, x1x8, x1x9, x1x10. Xét đồ thị con đầy đủ G1 gồm các đỉnh x2, x3, x4, x5, x6, x7, x8, x9, x10. Theo định lý 2.2.6 thì G1 có tam giác xanh hoặc tứ giác đỏ. Nếu tồn tại tam giác xanh ta có điều phải chứng minh. Nếu tồn tại tứ giác đỏ, giả sử tứ giác đó là x2x3x4x5 thì ta có ngũ giác đỏ x1x2x3x4x5. 51
  • 54. Nếu tam giác này có màu xanh thì định lý được chứng minh. Nếu tam giác này có màu đỏ thì tam giác này cùng với đỉnh x1 tạo thành tứ giác cùng màu đỏ. Định lí 2.2.8. Đồ thị 3 mảng G(X, E) với lực lượng mỗi mảng đều bằng n(n ≥ 1). Mỗi đỉnh được nối với từng đỉnh thuộc hai mảng còn lại bằng các cạnh tô màu xanh hoặc màu đỏ, sao cho số cạnh đỏ xuất phát từ mỗi đỉnh bằng đúng (n+1). Khi đó đồ thị G(X, E) có chu trình tam giác đỏ. Chứng minh. Trước hết chọn một đỉnh của G(X, E) có số cạnh đỏ nối với một trong hai mảng đỉnh còn lại lớn nhất. Giả sử đỉnh chọn được là A thuộc mảng thứ nhất và mảng có nhiều đỉnh nhất nối với A bằng một cạnh đỏ là mảng thứ hai gồm k đỉnh: B1, B2, ..., Bk . Khi đó số đỉnh thuộc mảng thứ 3 nối với A bằng một cạnh có màu đỏ là: n+k −1 đỉnh. Hình 2.16 Xét đỉnh C thuộc mảng thứ 3 và nối với đỉnh A bằng một cạnh có màu đỏ. Đỉnh C phải nối được với ít nhất một trong các đỉnh B1, B2, ..., Bk bằng một cạnh màu đỏ. Thật vậy, nếu ngược lại thì số đỉnh thuộc mảng thứ hai nối với C bằng một cạnh màu đỏ ít hơn n−k, suy ra số đỉnh thuộc mảng thứ nhất nối với C bằng một cạnh màu đỏ lớn hơn n+1−(n−k) = 52
  • 55. k + 1. Điều này mâu thuẫn với việc chọn điểm A, vậy tồn tại đỉnh Bi, sao cho CBi có màu đỏ. Suy ra ta có tam giác đỏ ABiC. 2.2.3 Thuật toán tô màu đỉnh. 1, Lập danh sách các đỉnh đồ thị theo thứ tự bậc giảm dần. Đặt i := 1 2, Tô màu i cho đỉnh đầu tiên trong danh sách. Duyệt lần lượt các đỉnh tiếp theo và tô màu i cho đỉnh không kề đỉnh đã được tô màu i. 3, Nếu tất cả các đỉnh đã được tô màu thì kết thúc: Đồ thị đã được tô bằng i màu. Ngược lại sang bước 4. 4, Loại khỏi tập đỉnh các đỉnh đã tô màu, đặt i := i + 1, và quay lại bước 2. 53
  • 56. Chương 3 Ứng dụng lý thuyết đồ thị vào giải toán phổ thông. 3.1 Quy trình giải bài toán bằng phương pháp đồ thị. Để giải bài toán T bằng phương pháp đồ thị ta cần thực hiện lần lượt hai bước sau: 3.1.1 Xây dựng đồ thị G mô tả các quan hệ. • Đỉnh: Lấy các điểm trên mặt phẳng hoặc trong không gian tương ứng với các đối tượng đã cho trong bài toán. Dùng ngay các ký hiệu đối tượng để ghi trên điểm tương ứng. • Cạnh: Hai đỉnh x, y tùy ý được nối với nhau bằng một cạnh (cung) với "đặc điểm t", khi và chỉ khi các đối tượng x, y có quan hệ "t" với nhau. Khi đó bài toán T đã được chuyển về bài toán D trên đồ thị. 3.1.2 Dựa vào các kết quả của lý thuyết đồ thị hoặc lý luận trực tiếp suy ra đáp án của bài toán D. Nếu đáp án của bài toán D còn dưới dạng "ngôn ngữ đồ thị", thì căn cứ vào phép đặt tương ứng khi xây dựng đồ thị mà diễn đạt thành đáp án bằng ngôn ngữ thông thường (tức là đáp án của bài toán T). 54
  • 57. 3.2 Bài toán về đỉnh - cạnh của đồ thị. Bài toán 3.2.1. (Olympic Toán Mỹ 1982) Sống trong một ký túc xá có 1982 người. Cứ bốn người trong đó bao giờ cũng chọn được ít nhất một người quen với cả ba người còn lại. Có ít nhất bao nhiêu người mà mỗi người quen với tất cả những người trong ký túc xá. Giải: 1. Xây dựng đồ thị G. Đỉnh: Lấy mỗi đỉnh trong đồ thị tương ứng với mỗi người sống trong ký túc xá. Cạnh: Trong đồ thị G hai đỉnh x,y bất kỳ có cạnh được nối với nhau khi và chỉ khi hai người x, y quen biết nhau. Đồ thị G mô tả toàn bộ quan hệ quen biết nhau trong ký túc xá mà ta xét. 2. Đáp án bài toán bằng ngôn ngữ đồ thị. Với cách xây dựng trên, bậc của mỗi đỉnh thuộc G bằng đúng số người quen của người tương ứng với đỉnh này. Theo định lý 1.3.5 ta suy ra có ít nhât 1982−3 = 1979 người mà mỗi người quen với tất cả những người trong ký túc xá. Bài toán 3.2.2. Trường THPT Tân Dân có 1101 học sinh. Biết rằng mỗi học sinh quen ít nhất 1001 học sinh. Chứng minh với mỗi học sinh của trường luôn tìm được 11 học sinh khác để tạo thành một nhóm gồm 12 học sinh, sao cho hai học sinh bất kỳ trong nhóm đều quen nhau. Giải: 1. Xây dựng đồ thị G. Đỉnh: Lấy 1101 đỉnh trên mặt phẳng tương ứng với 1101 học sinh. Cạnh: Trong đồ thị G hai đỉnh x,y bất kỳ được nối với nhau bằng một cạnh khi và chỉ khi hai em học sinh x, y quen nhau nhau. 2. Đáp án bài toán bằng ngôn ngữ đồ thị. Với cách xây dựng trên, bậc của mỗi đỉnh thuộc G bằng đúng số người quen của người tương ứng với đỉnh này. Theo định lý 1.3.7 số đỉnh của đồ thị G là kn + 1 = 1101 (∗) mỗi đỉnh có bậc không nhỏ hơn (k − 1)n + 1 = 1001 (∗∗) 55
  • 58. Do đó, luôn tồn tại đồ thị con đầy đủ gồm k + 1 đỉnh. Từ (∗), (∗∗) ta có n = 100, k = 11 . Như vậy, sẽ tồn tại một nhóm gồm k+1 = 11+1 = 12 em học sinh mà hai học sinh bất kỳ trong nhóm đều quen nhau. Bài toán 3.2.3. Liệu có thể có nhóm 9 người, mà trong đó mỗi người chỉ quen biết đúng 5 người khác được không? Giải: 1. Xây dựng đồ thị G. Đỉnh: Lấy 9 điểm tương ứng với 9 người trong nhóm làm đỉnh của đồ thị và đánh số từ 1 đến 9. Cạnh: Trong đồ thị G hai đỉnh x,y bất kỳ được nối với nhau bằng một cạnh khi và chỉ khi hai người x, y quen nhau. Đồ thị G mô tả quan hệ quen nhau của 9 người trong nhóm. Hình 3.1 2. Đáp án bài toán bằng ngôn ngữ đồ thị. Với cách xây dựng trên, bậc của mỗi đỉnh thuộc G bằng đúng số người quen của người tương ứng với đỉnh này. Nếu theo đúng giả thiết, tất cả các đỉnh của đồ thị đều có bậc là 5 thì theo định lý 1.3.1 số cạnh của đồ thị là: 9.5 2 = 22, 5 /∈ N 56
  • 59. không là số nguyên nên vô lý. Do đó, không thể có nhóm 9 người trong đó mỗi người chỉ quen biết 5 người khác. Bài toán 3.2.4. ([4]Lý thuyết đồ thị và các bài toán không mẫu mực) Chứng minh rằng: Nếu trong một tập số nguyên dương tùy ý M gồm ít nhất 3 số, mà có đúng 2 số có số số đồng dư bằng nhau, thì các số này không thể đồng thời không đồng dư với một số nào hoặc đồng thời đồng dư với tất cả các số còn lại thuộc tập M. Chứng minh. . 1. Xây dựng đồ thị G. Đỉnh: Lấy các điểm tương ứng với các số nguyên dương thuộc M làm đỉnh của đồ thị. Dùng ngay các số thuộc M để ghi trên các đỉnh tương ứng. Cạnh: Trong đồ thị G hai đỉnh x,y bất kỳ được nối với nhau bằng một cạnh khi và chỉ khi hai số x, y có số số đồng dư bằng nhau. Đồ thị G mô tả quan hệ của các số nguyên thuộc tập M, mà số số đồng dư của các số này là bằng nhau. 2. Đáp án bài toán bằng ngôn ngữ đồ thị. Theo định lý 1.3.4 đồ thị G với n đỉnh có đúng 2 đỉnh cùng bậc thì hai đỉnh này không thể đồng thời bậc 0 hoặc bậc n − 1 từ đó ta suy ra ngay điều phải chứng minh. Bài toán 3.2.5. Trong hội thi đấu cờ vua của khối các trường trung học phổ thông thuộc huyện Phú Xuyên có 10 em học sinh đại diện cho các trường tham gia thi đấu. Thể lệ cuộc thi là mỗi em phải đấu một trận với các em khác. Chứng minh rằng bất kỳ lúc nào cũng có 2 em đã đấu được một số trận như nhau. Giải: 1. Xây dựng đồ thị G. Đỉnh: Lấy mỗi điểm tương ứng với mỗi em học sinh tham gia thi đấu làm đỉnh của đồ thị. Dùng ngay tên của mỗi em để ghi trên đỉnh tương ứng. Cạnh: Trong đồ thị G hai đỉnh x,y bất kỳ được nối với nhau bằng một cạnh khi và chỉ khi hai 2 em học sinh x, y đã thi đấu cờ vua với nhau. Đồ thị G mô tả toàn bộ quan hệ thi đấu giữa các em học sinh. 2. Đáp án bài toán bằng ngôn ngữ đồ thị. 57
  • 60. Với cách xây dựng trên, bậc của mỗi đỉnh thuộc G bằng đúng số trận đấu mà em học sinh tương ứng với đỉnh này đã thi đấu. Theo định lý 1.3.3 đồ thị G luôn có ít nhất 2 đỉnh cùng bậc. Tức bất kỳ lúc nào cũng có 2 em đã đấu được một số trận như nhau. Bài toán 3.2.6. Cho một khối đa diện lồi A1, A2, ..., An. Gọi m1, m2, ..., mn lần lượt là số cạnh xuất phát từ các đỉnh A1, A2, ..., An và c là số cạnh của khối đa diện. Khi đó ta có m1 + m2 + ... + mn = 2c. Giải: 1. Xây dựng đồ thị G. Đỉnh: Lấy mỗi điểm trong đồ thị tương ứng với mỗi đỉnh của khối đa diện lồi. Cạnh: Trong đồ thị G hai đỉnh x,y bất kỳ được nối với nhau bằng một cạnh khi và chỉ khi cặp đỉnh x, y tạo thành cạnh của khối đa diện. 2. Đáp án bài toán bằng ngôn ngữ đồ thị. Với cách xây dựng trên, bậc của mỗi đỉnh thuộc G bằng đúng số cạnh xuất phát từ đỉnh tương ứng của khối đa diện. Theo định lý 1.3.1 tổng số bậc của tất cả các đỉnh luôn gấp đôi số cạnh. Do đó ta có m1 + m2 + ... + mn = 2c 3.3 Bài toán về xích, chu trình, đường, vòng và tính liên thông của đồ thị. Bài toán 3.3.1. ([4]Lý thuyết đồ thị và các bài toán không mẫu mực) Một thôn có ít nhất 4 gia đình, mỗi gia đình thân với ít nhất 3 gia đình khác. Chứng minh rằng có thể xếp một số chẵn gia đình làm nhà xung quanh một cái hồ để mỗi gia đình sống giữa hai gia đình mà họ thân. Giải: 1. Xây dựng đồ thị G. Đỉnh: Đối với mỗi gia đình trong thôn lấy một điểm tương ứng làm đỉnh của đồ thị và dùng ngay tên chủ hộ để ghi trên đỉnh tương ứng. Cạnh: Trong đồ thị G hai đỉnh x,y bất kỳ được nối với nhau bằng một cạnh khi và chỉ khi hai gia đình x, y thân nhau. Đồ thị G mô tả toàn bộ quan hệ thân nhau giữa các gia đình trong thôn. 2. Đáp án bài toán bằng ngôn ngữ đồ thị. 58
  • 61. Với cách xây dựng trên, bậc của mỗi đỉnh thuộc G bằng đúng số gia đình thân của gia đình tương ứng với đỉnh này. Theo cách xác định cạnh, mỗi đỉnh x có số cạnh đúng bằng số gia đình thân với gia đình x. Bởi vậy, mỗi đỉnh của đồ thị G đều có bậc không nhỏ hơn 3. Do đó theo định lý 1.4.2, trong đồ thị G tồn tại chu trình sơ cấp α độ dài chẵn. Khi đó dựa theo α mà sắp xếp chẵn gia đình tương ứng làm nhà xung quanh hồ theo thứ tự các đỉnh thì mỗi gia đình trong các gia đình này sẽ sống giữa hai gia đình mà họ thân. Bài toán 3.3.2. ([3]Lý thuyết đồ thị và ứng dụng) Một tập số nguyên dương M gồm ít nhất ba số. Mỗi số đều có ước chung với ít nhất hai số khác. Chứng minh rằng luôn luôn có thể ghi một nhóm gồm ít nhất ba số thuộc tập hợp lên một vòng tròn, để mỗi số đều đứng giữa hai số mà nó có ước chung. Giải: 1. Xây dựng đồ thị G. Đỉnh: Đối với mỗi số nguyên dương thuộc tập M lấy một điểm tương ứng làm đỉnh của đồ thị và ghi các số thuộc tập M lên các đỉnh tương ứng. Cạnh: Trong đồ thị G hai đỉnh x,y bất kỳ được nối với nhau bằng một cạnh khi và chỉ khi hai số x, y có ước chung. Đồ thị G mô tả toàn bộ quan hệ có chung ước với nhau giữa các số trong tập hợp M. 2. Đáp án bài toán bằng ngôn ngữ đồ thị. Vì mối số thuộc M có ước chung với ít nhất hai số khác, nên theo định lý 1.4.1 trong G có chu trình sơ cấp. Khi đó dựa theo một trong những chu trình sơ cấp của G mà ghi các số tương ứng lên một vòng tròn, thì mỗi số đều đứng giữa hai số mà nó có ước chung. Bài toán 3.3.3. (IMO 1991) Giả sử G là một đồ thị liên thông có k cạnh. Chứng minh rằng: Có thể đánh nhãn được các cạnh 1, 2, 3,...,k theo cách mà mỗi đỉnh thuộc vào hai hoặc nhiều hơn hai cạnh, ước số chung lớn nhất của các số nguyên đánh nhãn các cạnh này là 1. Giải: Ta bắt đầu tại một đỉnh v0 nào đó. Tưởng tượng rằng ta đang đi dọc theo các cạnh phân biệt của đồ thị, vừa đi vừa đánh số chúng như ta đang đếm: 1,2,3,... cho đến khi ta không thể đi xa hơn được nữa vì nếu muốn đi thêm phải dùng lại một cạnh đã đi qua. 59